VTNE practice - copy

Lakukan tugas rumah & ujian kamu dengan baik sekarang menggunakan Quizwiz!

In the typical life cycle of the flea (i.e. Ctenocephalides felis), where does the larva pupate and form into the adult flea? A) On the surface of the host animal B) In a tapeworm intermediate host C) In the environment (off of the host) D) In the gastrointestinal tract of the host animal

C) In the environment (off of the host) Explanation - In the flea life cycle, the adult female feeds, mates, and lays eggs on the host, but the eggs are smooth and usually fall off into the environment, often where the host typically lies down. After several days to weeks, the eggs hatch into larvae that feed on skin debris or other organic matter in the environment and then create a cocoon and pupate. The pupa then develops over 10 days or can remain in the environment considerably longer before emerging as an adult flea. This fact is important because this is why treating the host for fleas is not enough to eradicate a flea problem, as the environment is frequently contaminated and can re-infest the host after treatment

Which of the following can be easily changed by a technician to increase the amount of exposure on a radiograph without affecting the amount of contrast? A) Decrease time of exposure B) Decrease field size ("cone down") C) Increase milliamperage D) Increase kilovoltage

C) Increase milliamperage Explanation - The settings that affect exposure without changing contrast are milliamperage and exposure time. Increasing either will increase the exposure of the film. Changing kilovoltage changes the amount of contrast (higher kVp gives less contrast, more latitude and lower kVp gives greater contrast but less latitude). Decreasing field size will decrease the amount of scatter and may improve image quality but it will not increase exposure to the film

During an emergency you are assisting with CPCR. What is the best way listed to administer the drugs (atropine, epinephrine, and lidocaine)? You are unable to place an IV catheter despite multiple attempts. A) Subcutaneously B) Intraperitoneal C) Intratracheally D) Via stomach tube

C) Intratracheally Explanation - Drugs are rapidly absorbed via the intratracheal route. Establishing an airway is the second step (see below) in CPCR, so a polypropylene urinary catheter or red rubber feeding tube can be placed inside the endotracheal tube; the drug can then be injected through the tube followed with several mLs of sterile saline to help with distribution. The other methods listed are not appropriate for administration of emergency drugs as they would be absorbed at a very slow rate and absorbed very poorly due to poor circulation during cardiac arrest. *Establishing an airway was previously the first step in CPCR. However, the current guidlines put "Circulation" above "Airway", so that chest compressions are to be started immediately even prior to placing the endotracheal tube. (Obviously the endotracheal tube needs to also be placed ASAP)

A Yorkie is presenting for a bile acids test and has been fasted. Which of the following is true regarding a bile acids test? A) It is a measure of adrenal function B) It involves drawing a blood sample, giving a cortisol injection, and drawing another blood sample 1 hour after the injection C) It is a measure of liver function D) It is a measure of kidney function E) Fasting is not necessary for this test

C) It is a measure of liver function Explanation - A bile acids test is a measure of liver function. It requires fasting. A blood sample is drawn. The animal is then fed, and another sample is drawn 2 hours after feeding. The cortrosyn injection protocol describes an ACTH stimulation test which is a measure of adrenal gland function, testing for Cushing's (hyperadrenocorticism) or Addison's (hypoadrenocorticism)

Which of the following is TRUE regarding the presence of crystals in a dog or cat urine sample? A) It is always normal B) Crystals are not present in urine C) It is usually normal D) It is never normal

C) It is usually normal Explanation - Crystalluria is typically a normal finding. However, it is important for the technician to be able to identify the different types of crystals found in urine as some may be indicative of disease. An example of this is the finding of ammonium biurate crystals in small breed dogs. Finding this crystal is highly suggestive of a congenital portosystemic shunt

Which of the following is true of the liver? A) It does not have the ability to regenerate B) It produces amylase for digestion C) It makes and stores glucose D) It is the only source for alkaline phosphatase (ALP)

C) It makes and stores glucose Explanation - Glucose comes from liver production and from the GI tract when food is absorbed. The liver helps to break down fats, converts glucose to glycogen, produces urea (the main substance of urine), makes some amino acids, filters harmful substances from the blood, stores vitamins and minerals (vitamins A, D, K and B12) and maintains a proper level of glucose in the blood. The liver also produces cholesterol, albumin, bile, and coagulation factors. Amylase comes from the pancreas and also from saliva. The main sources of ALP are the liver and bone, but smaller amounts can be found in the intestine and kidney. Liver cells can regenerate

What is the most common location of an intravenous catheter in the horse? A) Auricular vein B) Saphenous vein C) Jugular vein D) Cephalic vein E) Femoral vein

C) Jugular vein Explanation - The jugular vein is very accessible and carries a low risk of injury to the person placing the catheter when compared to trying to place a catheter on a limb. Jugular veins are large and easy to find along the neck of the horse as well as easy to maintain

When assisting with a forelimb declaw, it is helpful to place a tourniquet to help control hemorrhage. Where should the tourniquet be placed? A) Directly around the digit upon which the surgeon is working B) Over the proximal humerus C) Just distal to the elbow D) Just proximal to the elbow

C) Just distal to the elbow Explanation - The optimal placement of the tourniquet is just below or distal to the elbow. Placing the tourniquet above the elbow can result in damage to the radial nerve, which will result in an inability for the patient to use the limb if the damage is permanent. Placing the tourniquet around the digit is unpractical and cumbersome

When using the ultrasonic cleaner, it is important to do which of the following? A) Add surgical milk to the solution B) Run the clean cycle for at least 27 minutes C) Lay the instruments in an open position D) Scrub the instruments while the ultrasonic cleaner is running

C) Lay the instruments in an open position Explanation - Laying the instruments in an open position will help ensure ultrasonic waves reach all areas of the instrument. Surgical milk should not be mixed in with the cleaner solution. The 10 minute run cycle is typically sufficient to clean the instruments. Scrubbing the instruments while in the ultrasonic cleaner is not necessary. Ideally, the instruments should be rinsed off of any major debris and residue or pre-cleaned prior to being placed in the ultrasonic cleaner.

When discussing teeth, what is the tooth surface closest to the tongue termed? A) Gingival B) Frontal C) Lingual D) Buccal

C) Lingual Explanation - The lingual side is closest the tongue. The buccal side is closest the cheek

A dog is having a mass removed from his side. What is the general term for this procedure? A) Buldgectomy B) Mastectomy C) Lumpectomy D) Tumorectomy E) Massectomy

C) Lumpectomy Explanation - A lumpectomy is the proper term for mass removal. A mastectomy is removal of a mammary gland. The other terms listed are not real terms used in veterinary medicine

Borrelia burgdorferi is the cause of which disease? A) Leptospirosis B) Cat scratch fever C) Lyme disease D) Ehrlichia canis

C) Lyme disease Explanation - Borrelia is a spirochete bacterium that causes Lyme disease. It is transmitted by the Ixodes tick (typically a deer tick). Ehrlichia is a rickettsial disease transmitted by the brown dog tick. Leptospirosis is also a spirochete bacteria transmitted by Leptospira. In veterinary medicine it most commonly transmitted through contact with urine from an infected animal. The causative agent for cat scratch disease is the bacteria Bartonella henselae

What personal protective equipment would be required for a dental cleaning? A) Booties, cap, mask, gloves B) Mask, eye protection, cap C) Mask, eye protection, gloves D) Eye protection, cap, booties, gloves

C) Mask, eye protection, gloves Explanation - For a dental, a face shield or eye protection is needed due to the splatter from dental cleaning. A mask is needed to prevent inhalation of this splatter or mist. Gloves are required to prevent the spread of infection. Dental cleanings are not sterile procedures, so a cap and booties would not be required

The 4th upper premolars are connected with which structure in dogs? A) Pharynx B) Nasal cavity C) Maxillary Sinus D) Frontal Sinus

C) Maxillary Sinus Explanation - The 4th upper premolars (carnassial teeth) communicate directly with the maxillary sinus. In the case of a carnassial tooth that becomes infected, an abscess builds up around the affected root just under the skin below and in front of the eye

Which of the following is a measure of red blood cell size on a complete blood count? A) Mean Corpuscular Hemoglobin (MCH) B) Mean Corpuscular Hemoglobin Concentration (MCHC) C) Mean Corpuscular Volume (MCV) D) Hematocrit

C) Mean Corpuscular Volume (MCV) Explanation - Red blood cell size (in femtoliters) is described by the MCV. This value can be determined electronically by automated analyzers or manually by dividing the PCV by the RBC concentration and multiplying by 10

Which of the following will cause a false positive fecal occult blood test? A) Dairy in the diet B) Steroid administration C) Meat based diet D) Diarrhea

C) Meat based diet Explanation - Meat but not dairy in the diet will result in a false positive fecal occult blood test. Therefore, the test should be administered after a patient has been on a meat-free diet such as cottage cheese and rice for at least 3 days

Which method is most useful for looking for Otodectes? A) They are visible with the naked eye B) Knott's method C) Mineral oil swab slide D) Hematoxylin and Eosin stain E) Sedi-stain

C) Mineral oil swab slide Explanation - Otodectes cynotis is the ear mite. It is most easily observed by taking debris from the ear from a swab and making a mineral oil smear on a slide for observation under the microscope. Knott's is used in looking for microfilariae

A degenerative left shift implies which of the following? A) More lymphocytes than neutrophils B) More eosinophils than lymphocytes C) More neutrophilic bands than normal neutrophils D) Less monocytes than lymphocytes

C) More neutrophilic bands than normal neutrophils Explanation - A degenerative left shift specifically refers to the neutrophil count. In this case, there are more neutrophilic bands present than neutrophils. In a regenerative shift there are more normal neutrophils present than neutrophilic bands. Bands have a smooth nucleus with parallel sides, while mature neutrophils have a twisted nucleus

A lab report has a note from the pathologist that says the "segs" were elevated. What blood cell type is a "seg"? A) Lymphocyte B) Platelet C) Neutrophil D) Red blood cell

C) Neutrophil Explanation - "Seg" is short for segmented neutrophil

Which of the following structures in the body is most sensitive to radiation? A) Bones of the hand B) Thyroid gland C) Ocular lens D) Cerebral neurons

C) Ocular lens Explanation - The lens of the eye is considered one of the most radiation-sensitive structures in the body. Even relatively low doses can predispose the lens to develop cataracts. The other most sensitive organs are ones with rapidly dividing cells such as the bone marrow and gonads

If a pulmonary metastatic lesion is visible on a right lateral radiograph but not on a left lateral radiograph, where is the lesion most likely to be? A) One of the cranial lung lobes B) One of the right lung lobes C) One of the left lung lobes D) One of the caudal lung lobes E) One of the dorsal lung lobes F) One of the ventral lung lobes

C) One of the left lung lobes Explanation - When a patient is placed into left lateral recumbency for a left lateral radiograph, the left lung fields are compressed and do not inflate fully. This can prevent resolution of small metastatic lung lesions in the left lung lobes due to the lack of contrast with air in the left lung. The same is true of the right lung fields with a right lateral radiograph. Therefore, for evaluation of pulmonary metastasis, both a left and right lateral radiograph should be taken. In this case, a lesion that can be seen on a right lateral but not a left lateral projection is most likely in the left lung lobe because, as explained above, the lesion may be obscured on the left lateral projection due to a lack of contrast in the uninflated lung

What represents depolarization from the SA node through the atria on an electrocardiogram (ECG)? A) T wave B) ST segment C) P wave D) QRS complex

C) P wave Explanation - The P wave represents depolarization spreading from the SA node through the atria. The PR segment on the ECG represents the delay at the AV node and partial atrial repolarization. The QRS complex represents ventricular depolarization, and the ventricular rate can be calculated by the time interval between the QRS complexes. The ST segment comes after the QRS and is when the ventricle is depolarized. The ST segment can help to diagnose ischemia or hypoxia because it will become either depressed or elevated. The T wave represents ventricular repolarization

Accumulations of desquamated cells, food particles, and bacteria along the teeth are known as which of the following? A) Caries B) Calculus C) Plaque D) Tartar

C) Plaque Explanation - A plaque is an accumulation of desquamated cells, food particles, and bacteria. Bacterial toxins damage the gingiva and oral mucosa. Dental calculus or tartar is mineralized plaque and can lead to persistent inflammation. Dental caries, also known as cavities or decay, are incidents of demineralization of the teeth

Which vessel transports blood from the intestines to the liver? A) Renal artery B) Vena cava C) Portal vein D) Hepatic duct

C) Portal vein Explanation - The portal vein is responsible for transporting the nutrient-dense blood from the intestines to the liver

Which of the following scenarios describes the proper care of surgical instruments? A) Rinse the instruments to remove the debris and residue. Place in an ultrasonic cleaner for 30 minutes. Then place the instruments in surgical milk. After removing instruments from the surgical milk rinse them again. B) Place instruments in surgical milk. Remove from surgical milk and rinse to remove and debris and residue. Place instruments in ultrasonic cleaner for 10 minutes. Let instruments dry. C) Pre-rinse the instruments immediately after surgery to remove residues. Place in ultrasonic cleaner for approximately 10 minutes. Place in surgical milk. Remove from surgical milk and let instruments dry. D) Place instruments in ultrasonic cleaner for approximately 10 minutes. Rinse instruments with distilled water and scrub as necessary. Place instruments in surgical milk. Remove from surgical milk and let instruments dry.

C) Pre-rinse the instruments immediately after surgery to remove residues. Place in ultrasonic cleaner for approximately 10 minutes. Place in surgical milk. Remove from surgical milk and let instruments dry. Explanation - The correct answer is to pre-rinse the instruments (using distilled water) immediately after surgery to remove residues. Place in ultrasonic cleaner for approximately 10 minutes. Place in surgical milk. Remove from surgical milk and let instruments dry.

The sacculus rotundus is part of the intestinal anatomy of which species? A) Horse B) Camel C) Rabbit D) Psittacine bird

C) Rabbit Explanation - Just proximal to the cecum is a gastrointestinal structure containing a large amount of lymphoid tissue called the sacculus rotundus which has some immune function. It is present only in lagomorphs

Which of these are used in the minor cross-match for blood products when looking for a compatible blood donor for a dog? A) Donor red blood cells, recipient plasma B) Recipient red blood cells, recipient plasma C) Recipient red blood cells, donor plasma D) Donor red blood cells, donor plasma

C) Recipient red blood cells, donor plasma Explanation - The correct answer is recipient red blood cells, donor plasma. In a minor crossmatch, you are looking to see if the factors in the plasma of the donor are going to react to the recipient's red blood cells

A patient with DIC (Disseminated Intravascular Coagulation) may show abnormal red blood cells called schistocytes. What are schistocytes? A) Large blue tinged red blood cells B) Red blood cells with nuclear fragments C) Red blood cell fragments D) Nucleated red blood cells

C) Red blood cell fragments Explanation - Patients with DIC frequently have schistocytes and decreased platelets on their blood smears. Schistocytes are red blood cell fragments

A red blood cell fragment, as might be seen from shearing of the cell by trauma in an animal with disseminated intravascular coagulopathy, is known as what? A) Spherocyte B) Keratocyte C) Schistocyte D) Acanthocyte E) Echinocyte

C) Schistocyte Explanation - A schistocyte is a red blood cell fragment that is usually formed due to shearing from intravascular trauma; they are seen in cases of DIC or vascular neoplasms such as hemangiosarcoma. An acanthocyte is a red blood cell with surface projections that are seen mainly in animals with altered lipid metabolism or liver disease. An echinocyte is a red blood cell with uniform blunt projections that are usually artifacts from blood drying but may also be seen with some diseases. A keratocyte is a red blood cell that appears to contain a vacuole; they are sometimes called blister cells and may occur from intravascular trauma. A spherocyte is a dark-staining red blood cell with no central pallor that usually occurs from partial phagocytosis of the cell seen with immune-mediated destruction

What kind of artifact can occur when ultrasound hits bladder calculi? (ie: uroliths, bladderstones) A) Reverberation B) Acoustic enchancment C) Shadowing D) Mirror image

C) Shadowing Shadowing occurs when ultrasound hits highly reflective structures like uroliths (bladderstones). Acoustic shadowing is a dark area distally or beyond the highly dense material. Basically, the stones (or bone, for example) are blocking the ultrasound beam.

You are asked to restrain a dog for a cephalic draw. How are you going to hold the dog? A) Lateral recumbency, holding off and stabilizing the lateral side of the pelvic limb B) Dorsal recumbency and stabilize the hindlimbs C) Sitting position, holding off and stabilizing a forelimb D) Sitting position, holding the head steady and pointing the muzzle upward

C) Sitting position, holding off and stabilizing a forelimb Explanation - The cephalic vein courses over the dorsal aspect of the radius and is a common place to draw blood. The position holding the head and pointing the muzzle upward is for jugular draws. The lateral aspect of a pelvic limb is used to draw from the lateral saphenous vein. The dorsal recumbency position is used sometimes for cystocentesis (inserting a needle in the bladder to collect urine)

Stimulation of the Vagus nerve results in which of the following? A) Diarrhea B) Hypersalivation C) Slowed heart rate D) Dilated pupils E) Tachycardia

C) Slowed heart rate Explanation - Vagal stimulation slows the heart rate

A 13-year old diabetic cat arrests during hospitalization during a blood glucose curve. During resuscitation efforts, an emergency blood panel is run which reveals a blood pH of 6.4. What drug may be requested by the veterinarian to help correct this abnormality but would not be used as a long-term therapy? A) Darbopoetin B) Methylprednisolone C) Sodium bicarbonate D) Insulin E) Potassium gluconate

C) Sodium bicarbonate Explanation - A blood pH of 6.4 is low, or acidic. Sodium bicarbonate can increase the cat's blood pH but ultimately the underlying problem (the diabetes) must be treated. Insulin may be used in both resuscitation efforts and long-term therapy. Darbopoetin causes an increase in red blood cell production and has no effect on blood pH. Potassium gluconate is administered orally to correct hypokalemia. Methylprednisolone can cause an increase in blood glucose levels and is not a drug used to correct pH. Corticosteroids such as this should be avoided in diabetic patients when possible

What type of muscle is the heart? A) Striated muscle, voluntary B) Smooth muscle, voluntary C) Striated muscle, involuntary D) Smooth muscle, involuntary

C) Striated muscle, involuntary Explanation - The three muscle types are cardiac, smooth, and skeletal. Cardiac muscle cells are located in the walls of the heart. This is striated muscle that is under involuntary control. Smooth muscle fibers are located in walls of hollow visceral organs, except the heart and are spindle-shaped. This type is also under involuntary control. Skeletal muscle fibers are striated and are in muscles attached to the skeleton. They are under voluntary control.

What is a ranula? A) Anal gland infection B) Cyst on the eyelid C) Sublingual mucocele D) Sludge accumulation in the gallbladder

C) Sublingual mucocele Explanation - A ranula is a fluctuant swelling of connective tissue consisting of collected mucin from a ruptured salivary gland duct. They are found under the tongue

Parrots have the unique ability to mimic sound. What anatomical structure in birds is responsible for producing sound? A) Larynx B) Cloaca C) Syrinx D) Choana

C) Syrinx Explanation - The syrinx is considered to be the voice box of parrots. They don't have a larynx, like mammels. The syrinx is located between the base of the tongue and the trachea. The choana is the slit-like opening in the roof of the bird's mouth that connects to the sinus cavity in the skull. The cloaca is the opening where the feces, urine, and reproductive activity exit the body

Which of the following drugs can cause discoloration of puppy teeth? A) Metronidazole B) Enrofloxacin C) Tetracycline D) Amoxicillin

C) Tetracycline Explanation - Tetracyclines can cause a discoloration of puppy teeth. Tetracycline binds to calcified tissues (bone, cementum, dentin), gets incorporated into the hydroxyapatite crystals with the calcium, and causes the discoloration. This occurs with puppy teeth because the teeth are still being actively formed. After the adult teeth are present, this should no longer occur

Which region of the blood smear should be used for the differential cell count? A) The center of the slide B) The feathered edge C) The monolayer region D) The thickest region of the smear

C) The monolayer region Explanation - When a blood smear is made, the feathered edge is where cells may be distorted or erratically distributed. It is appropriate to assess this area and the rest of the slide for platelet clumps, large irregular cells, and parasites. However, it is best to look in the monolayer region for the differential cell count because cells here should be randomly distributed with minimal distortion. Thicker regions or the center of the slide will frequently have so many red blood cells that they obscure the view of leukocytes

When taking a radiograph of the tibia and fibula, which of the following will occur if the x-ray beam is perpendicular to the tibia and fibula but not to the x-ray cassette and film? A) The tibia and fibula will appear foreshortened B) The tibia will appear foreshortened, and the fibula will appear elongated C) The tibia and fibula will appear elongated D) The tibia will appear elongated, and the fibula will appear foreshortened

C) The tibia and fibula will appear elongated Explanation - To avoid distortion (elongation or foreshortening), the x-ray beam should be perpendicular to the long axis of the bone and the x-ray cassette. If the bones are not perpendicular to the beam, they will appear foreshortened. If the cassette is not perpendicular to the beam, the bones will appear elongated. In dental radiography, it is frequently not possible for the long axis of the tooth and the film to be parallel to each other. In this case, the bisecting angle technique may be used, wherein the beam is angled half-way between the angle of the long axis of the tooth and the angle of the film in order to obtain a true image that is neither elongated nor foreshortened

Which of the following is considered part of the plantar surface? A) The top surface of the hindpaw B) The underside of the forepaw C) The underside of the hindpaw D) The top surface of the forepaw

C) The underside of the hindpaw Explanation - Plantar is the underside of the hindpaw. Palmar is the underside of the forepaw

Why are guinea pig sows prone to C-section if they are bred for the first time after 6 months of age? A) The brachycephalic skull of the babies will get stuck. B) You can't do a c-section on a guinea pig. C) Their pelvic bones fuse between 4 and 6 months. D) Guinea pigs are known to have low blood calcium. E) The older the sow is, the greater number of young they produce in one litter. F) The babies are too big to be born naturally.

C) Their pelvic bones fuse between 4 and 6 months. Explanation - Guinea pigs must be bred for the first time prior to 6 months of age because their pelvis must be flexible enough to allow the the fetus to pass through the birth canal. Their pelvic bones fuse at 4 - 6 months. If this occurs and the sow is bred, dystocia will occur

Asepsis implies which of the following? A) There are less than 1000 colonies of bacteria present B) There are less than 100 colonies of bacteria present C) There is no living organism present D) There are only symbiotic bacteria present

C) There is no living organism present Explanation - Asepsis means absolutely no living organisms present. The concept of aseptic technique is very important, and technicians should be very aware of sterility in the operating room. This includes walking within the room, passing instruments, prepping the patient, and working within the operating room. Adhering to strict aseptic principles decreases the likelihood a patient will develop a post-operative infection

Bacteria differ from animal cells in that they lack which of the following? A) They are unable to metabolize glucose B) They lack cell walls C) They lack a true membrane-bound nucleus D) They lack ribonucleic acid

C) They lack a true membrane-bound nucleus Explanation - Bacteria are referred to as prokaryotes because they lack a true membrane-bound nucleus. Animals, as well as all multi-cellular organisms, are known as eukaryotes since they have a membrane-bound nucleus. Bacteria do have ribonucleic acids, and several antibiotics target transcription of mRNA. Bacteria are able to metabolize glucose. The type of cell wall that a bacterium has helps us differentiate whether it is a Gram-positive or Gram-negative organism

A multi-lumen jugular catheter must be placed in a canine with renal disease for CVP monitoring. To what anatomical landmark should the catheter be inserted to? A) Thoracic inlet B) Aorta C) Third thoracic vertebrae D) Caudal vena cava E) The heart

C) Third thoracic vertebrae Explanation - Central venous pressure, or CVP, monitoring must be performed according to specific guidelines to reduce the possibility of inappropriate readings. The third thoracic vertebra is the anatomical landmark that the normal values are based on.

When performing anesthesia in the horse, which of the following should always be done prior to induction and intubation? A) Apply a small amount of lidocaine to the larynx B) Float the molars C) Throughly rinse the oral cavity D) Insert a tube into the esophagus so that you are able to place the endotracheal tube properly

C) Throughly rinse the oral cavity Explanation - The oral cavity should always be rinsed to remove food particles to prevent introducing foreign material into the trachea.

What is the purpose of polishing the teeth after a dental cleaning? A) To prevent cavity formation B) To strengthen the enamel C) To smooth the microscopic defects on the tooth surface D) To remove any remaining calculus on the teeth

C) To smooth the microscopic defects on the tooth surface Explanation - Scratches or pits on the tooth that can be created by the scaling procedure or from mastication increase the surface area on the tooth, allowing plaque/calculus to build up more quickly. Polishing helps to smooth out any defects. All surfaces of the tooth crown should be polished on slow rotational speed. The polisher should be kept in constant motion to prevent thermal damage to the tooth

If a disease is zoonotic, what does this imply? A) Transmissible between animals but not to people B) Host-specific C) Transmissible from animals to people D) Not contagious

C) Transmissible from animals to people Explanation - A zoonotic disease is an infectious disease in animals that can be transmitted to people

Rabbits are prone to hairballs. What is the medical term for a hairball? A) Phytobezoar B) Dermatophyte C) Trichobezoar D) Piloerection

C) Trichobezoar Explanation - Tricho refers to hair, bezoar is the compaction, "ball" that can result from different substances. Phytobezoar is a "plant - ball". Piloerection is used to describe when hair stands up. Like when a cat or dog is fearfull. Dermatophyte is the general term for skin fungus

When applying a bone plate to fix a fracture, which of the following instruments would be least useful? A) Drill guide B) Screw driver C) Trocar D) Tap E) Point-to-point forceps

C) Trocar Explanation - Trocars are typically used for penetrating into a space for arthroscopy, thoracoscopy, or laparoscopy. Once the trocar has been inserted, it is replaced by a cannula, which allows for insertion of an instrument or scoping device into the desired space, such as the knee joint. A drill with the appropriately sized drill bit is used to drill holes into bone. A drill guide is used to help protect the surrounding soft tissues and assist in aiming. A tap is then used to put a thread into the hole so that the screw can be placed securely. The screw is inserted with the aid of a screwdriver. Point-to-point forceps will help keep the fracture properly reduced while the bone plate is being applied.

The word "idiopathic" is commonly used in veterinary medicine. What does this word imply? A) Acquired in the hospital B) Inflicted by another C) Unknown cause D) Self-inflicted

C) Unknown cause Explanation - Idiopathic means that the cause or mechanism for a disease is unknown. Nosocomial means acquired in the hospital

Which of the following principles will always improve radiographic image quality? A) Use the lowest kilovoltage (kVp) setting and the highest milliampere (mA) that adequately exposes the film as possible B) Use the highest kilovoltage (kVp) setting and the lowest milliampere (mA) that adequately exposes the film as possible C) Use the highest milliampere (mA) setting and lowest time (s) setting possible to achieve the desired mAs D) Use the lowest milliampere (mA) setting and the highest time (s) setting possible to achieve the desired mAs

C) Use the highest milliampere (mA) setting and lowest time (s) setting possible to achieve the desired mAs Explanation - The product of multiplying the milliamperage (mA) and time (s) is the mAs, which determines the degree of film exposure for a given kVp. Minimizing the time of exposure decreases motion artifact that occurs from the patient moving. It is always preferable to use the highest milliamperage and lowest time setting for this reason. Depending on the type of radiographic study and the size of the animal, a certain kVp and mAs will produce a radiograph with a corresponding amount of contrast and latitude. Maximizing contrast at the expense of latitude or vice-versa may not yield the best radiograph

Which of the following is NOT considered good surgical instrument care? A) Using cleaning agents at a pH of approximately 10 B) Using an ultrasonic cleaner C) Using tap water D) Using a brush to scrub the instruments

C) Using tap water Explanation - The use of tap water for cleaning surgical instruments is not recommended since it carries a high likelihood of leaving mineral deposits on the instruments; distilled water is preferred. Cleaning agents that have a higher pH (between approximately 9 and 11) are used because these do not promote corrosion. An ultrasonic cleaner is highly recommended as it is much more effective than manual cleaning alone. The use of a brush to scrub the instruments is common practice and facilitates the removal of dry blood or residue.

A dog presenting with a head tilt and loss of balance most likely has disease associated with which nerve? A) Optic B) Trigeminal C) Vestibulocochlear D) Vagus

C) Vestibulocochlear Explanation - Head tilt and loss of balance are symptoms of vestibular syndrome, which is most often assoiated with damage to the vestibulocochlear nerve (CN VIII). Other symptoms may include nausea and nystagmus

Under which circumstances are pigs most likely to bite each other's tails? A) When they are fed a grain-only diet B) When a pig has an open wound C) When the facility is over-crowded D) When there is more than one male in a group

C) When the facility is over-crowded Explanation - Tail-biting is most likely to occur when a hog farm is over-crowded

What is the most commonly used method for fecal parasite testing for dogs and cats? A) Fecal cytology using H&E staining method B) Direct smear C) Zinc sulfate centrifugation D) Baermann funnel technique

C) Zinc sulfate centrifugation Explanation - Zinc sulfate centrifugation is the most common and best of these choices. Technique: Zinc sulfate (ZNSO4) solution is added to 2-3 grams of feces. It is mixed and then centrifuged for a few minutes on high speed. A loop can then skim the surface and place the sample on a slide for examination, or the tube can be filled to the top with more zinc solution and a coverslip set on top for a few minutes. Then the coverslip can be placed onto a slide for examination. Sometimes a drop of iodine is added to the sample to help identify the eggs more easily. Baermann is useful for detecting lungworm and some other nematodes. Direct smears can help detect trophozoites, but they are not useful for finding tapeworm or nematode eggs, or coccidia. Cytology of feces would mostly just demonstrate the bacterial flora

You are working in the critical care unit and learning to perform central venous pressure readings. What is a normal CVP reading? A) 15-20 cm H2O B) 10-15 cm H2O C) 90-120 cm H2O D) 0-10 cm H2O E) Less than 0 cm H2O

D) 0-10 cm H2O Explanation - The central venous pressure is the blood pressure of the thoracic vena cava. It is very helpful in monitoring the hydration status of a patient. A value less than zero would indicate dehydration and a value starting to trend upward around 5-8 indicates adequate fluids. Readings over 10 may indicate fluid overload or venous congestion. These values should always be interpreted in light of the patient's clinical status (i.e. body weight, skin turgor, heart rate, mucus membrane status, etc

Which is safe to put in the ears if the tympanum may be ruptured? A) Chlorhexidine B) Gentamicin C) Neomycin D) 0.9% NaCl

D) 0.9% NaCl Explanation - Saline (0.9% sodium chloride) is considered safe. The others listed (especially aminoglycoside antibiotics) could potentially be ototoxic if the eardrum is not intact

A 12-pound cat needs Clavamox drops at 14 mg/kg BID. The solution is 62.5 mg/mL. How many mLs should this cat receive twice daily? A) 0.8 mL B) 3.1 mL C) 2.4 mL D) 1.2 mL

D) 1.2 mL Explanation - 12 pounds/2.2= 5.4 kg 5.4 kg X 14 mg/kg dose = 76 mg 76 mg dose/ 62.5 mg/mL concentration = 1.2 mL

What is the maximum time that an ultrasonic scaler can be used on a tooth surface? A) 30 seconds B) 60 seconds C) 3-5 seconds D) 10-15 seconds

D) 10-15 seconds Explanation - The scaler must be kept in constant motion on the tooth surface and should not be on a tooth for more than 10 to 15 seconds. This is because heat can buildup and cause thermal damage to the tooth. The water from the scaler also helps to prevent heat buildup. The scaler should be held in a modified pen grasp during the procedure

How many pairs of cranial nerves are there? A) 15 B) 10 C) 8 D) 12

D) 12 Explanation - There are twelve pairs of cranial nerves. They are designated by Roman numerals as follows: I - Olfactory nerve II - Optic nerve III - Oculomotor nerve IV - Trochlear nerve V - Trigeminal nerve VI - Abducens nerve VII - Facial nerve VIII - Vestibulocochlear nerve IX - Glossopharyngeal nerve X - Vagus nerve XI - Accessory nerve XII - Hypoglossal nerve

Typically, when should a dog or cat be offered food following gastrointestinal surgery? A) 6-12 hours after surgery B) 3-5 days after surgery C) 48-72 hours after surgery D) 12-24 hours after surgery

D) 12-24 hours after surgery Explanation - Most animals undergoing intestinal surgery should be offered food and water within 12-24 hours after surgery. Further delaying the delivery of nutrition may result in slower healing. Trying to feed a patient within 6-12 hours after major surgery is usually unrewarding because the patient is still recovering from anesthesia and is frequently under the effects of strong pain medications which contribute to inappetence and nausea.

The heart rate of an adult cat should range between which of the following? A) 30-60 bpm B) 80-120 bpm C) 120-150 bpm D) 150-220 bpm

D) 150-220 bpm Explanation - An adult cat typically has a resting heart rate between 150-220 bpm. Keep in mind that if the cat is very excited the heart rate might be faster. A dog's heart rate ranges between 70-160 bpm. As with the cat, there may be variation. Remember that puppies and kittens will have faster heart rates.

What is the maximum time a fecal can be used for fecal flotation with zinc centrifugation to look for ova and parasites if kept in the refrigerator? A) 12 hours B) 2 hours C) 1 week D) 3 days

D) 3 days Explanation - Ideally, a fecal should be collected and performed within 24 hours. However, the maximum time that the feces can still be used if refrigerated is 3 days. Fresher is always better

A puppy presents for his first distemper-parvo puppy shot. He is currently 8 weeks of age. When should he receive his next set of vaccines? A) 1 week from now B) 8 weeks from now C) 4 months from now D) 3 weeks from now

D) 3 weeks from now Explanation - The puppy series starts at 6 to 8 weeks of age, and the distemper-parvo combo vaccine is given 3 weeks apart for a total of 3 to 4 vaccines, depending on the breed. Black-and-tan breeds such as Rottweilers are thought to be more susceptible to parvo-virus, so this breed may receive a series of 4 to 5 vaccines, depending on the age and status of the pet

For a veterinary X-ray technician, what is the maximum permissible dose of radiation they can receive in one year? A) 100 mrem B) 5 Sv C) 1 rem D) 50 mSv E) 10 rad F) 5 Gy

D) 50 mSv Explanation - There are many units of radiation exposure and dose that are used. Old terminology used the unit of the rad for measuring dose. For radiation safety, a "quality factor" is typically multiplied by the dose to give a "dose equivalent". The rem (radiation equivalent in man) is the dose in rad multiplied by a quality factor. Accepted terminology has changed over the years, and the standard (SI) unit of dose is the Gray (Gy). There are 100 rad per 1 Gy (1 rad = 1 cGy). The Sievert is the dose in Gy multiplied by the quality factor. The maximum permissible dose for a radiation worker as described in this question is 5 rem. (5,000 mrem). This converts to 50 mSv, which is the correct answer to this question

How many milliliters of 50% dextrose should be added to one liter of 0.9% saline in order to make it a 2.5% dextrose solution? A) 75 ml B) 5 ml C) 100 ml D) 50 ml E) 25 ml

D) 50 ml Explanation - To answer this question, the following formula should be implemented: C(1)V(1) = C(2)V(2) C is the concentration V is the volume C(1) =0.5 V(1) = x C(2) = 0.025 V(2) = 1000 ml 0.5x = 0.025(1000) x = 50 ml 50 ml of 50% dextrose should be added to one liter in order to make a 2.5% solution.

The respiratory rate for an adult horse should generally be: A) 40-50 breaths per minute B) 30-40 breaths per minute C) 5-8 breaths per minute D) 8-20 breaths per minute

D) 8-20 breaths per minute Explanation - Horses breathe at a rate of 8 to 20 times per minute. Dogs breathe 10-30 times per minute at rest; cats breathe 24-42 times per minute at rest

Some orphaned kittens are left on the doorstep of the clinic where you work. You volunteer to take care of these kittens. What volume can the stomach of a kitten or puppy hold on average? A) 15 mL/kg B) 30 mL/kg C) 2 mL/kg D) 9 mL/kg

D) 9 mL/kg Explanation - The stomach volume of a kitten is around 9 mL/kg. Giving more than this at a time can lead to regurgitation and causes increased risk of aspiration

A 3-year old male neutered Labrador Retriever presents for lethargy and inappetence of a few days in duration. The dog is approximately 7% dehydrated with pale and dry mucous membranes, a capillary refill time > 2 sec, heart rate > 170 bpm and weak femoral pulses. The veterinarian diagnoses the dog with hypovolemic shock and orders isotonic crystalloid fluid therapy. What is the isotonic crystalloid total "shock dose" in dogs? A) 5 ml/kg B) 10-20 ml/kg C) 180 ml/kg D) 90 ml/kg E) 40-60 ml/kg

D) 90 ml/kg Explanation - The isotonic crystalloids shock dose in dogs is 90 ml/kg. 1/4 to 1/3 of this volume should be administered initially and the patient reassessed. In cats, the isotonic crystalloids shock dose is 40-60 ml/kg. Synthetic colloids can be administered at a dose of 10-20 ml/kg in dogs and 5-10 ml/kg in cats; hypertonic saline can be administered at a dose of 5 ml/kg.

What is the test of choice for diagnosing Addison's disease in dogs? A) Pre- and post-prandial bile acids test B) Estrogen level C) Insulin:glucose ratio D) ACTH stimulation test E) Urine cortisol:creatinine ratio

D) ACTH stimulation test Explanation - Addison's disease, or hypoadrenocorticism, is a condition where the adrenal glands do not produce sufficient cortisol. ACTH should stimulate cortisol secretion from the adrenal glands, but patients with Addison's disease do not have the ability to secrete normal cortisol levels, even after stimulation

How can a technician determine if a pack was sterilized appropriately? A) The tape on the outside of the pack is not broken B) The pack is steaming when removed from the autoclave C) The pack is double wrapped prior to autoclaving, ensuring sterility D) An indicator strip inside the pack changes color

D) An indicator strip inside the pack changes color Explanation - The only way to know for sure if a pack has received appropriate sterilization is if the indicator strip changes color inside the pack

One of the most common incisional complications encountered in veterinary surgery is the formation of a seroma. Which of the following is a poor treatment choice for an incision diagnosed with a seroma? A) Exercise restriction B) Placement of a drain C) Warm compress D) Antibiotics

D) Antibiotics Explanation - Seromas are not infections and do not need to be treated with antibiotics. They consist of serous fluid, which is usually secondary to excessive dissection or "dead-space" that was created during surgery. Excessive activity and motion at the surgical site may predispose to the formation of a seroma. A warm compress will help increase blood flow to the region and thus help the body reabsorb the fluid. Nonetheless, it may take days to weeks for a seroma to completely resolve. Placing a drain at the site of a seroma may help speed up its resolution, as this helps keep the tissues adhered to each other.

A patient who has stopped breathing after administration of a short-acting anesthetic can be described as being which of the following? A) Eupneic B) Atelectic C) Dyspneic D) Apneic

D) Apneic Explanation - This patient would be apneic, which means the patient is not breathing. Dyspnea implies difficulty breathing. Eupnea is normal respiration. Atelectasis is the description given to collapsed lungs

What is the average gestation length of a cow? A) Approximately 350 days B) Approximately 200 days C) Approximately 120 days D) Approximately 285 days

D) Approximately 285 days Explanation - Cows have an average gestation length of 285 days (about 9 months). The average gestation length of dogs and cats is 63 days. Horses have a gestation length of approximately 340 days. Although the average for horses is approximately 11 months, it can vary by as much as 30 days.

What is the average gestation length of a horse? A) Approximately 400 days B) Approximately 230 days C) Approximately 290 days D) Approximately 340 days

D) Approximately 340 days Explanation - Horses have a gestation length of approximately 340 days. Although the average is approximately 11 months, it can vary by as much as 30 days. Cows have an average gestation length of 285 days (about 9 months). The average gestation length of dogs and cats is 63 days

You are examining a blood agar plate from a submitted bacterial culture and see an area of complete hemolysis, leaving a clear zone around a bacterial colony. What type of hemolysis is this? A) Gamma hemolysis B) Delta hemolysis C) Alpha hemolysis D) Beta hemolysis

D) Beta hemolysis Explanation - Alpha hemolysis is partial hemolysis, creating a band of slimy discoloration around a bacterial colony. Beta hemolysis is complete hemolysis, leaving a clear zone around a bacterial colony. Gamma hemolysis causes no change in the medium's appearance. Delta hemolysis creates a double zone of hemolysis wherein a narrow area around the colony is hemolyzed and surrounded by a larger region with some hemolysis

You are asked to run an Azostix strip on a dog prior to anesthesia. What does this test for? A) Alanine aminotransferase B) Creatinine C) Amylase D) Blood urea nitrogen E) Blood glucose

D) Blood urea nitrogen Explanation - The Azostix tests for BUN. It is a small strip to which a drop of blood is applied and gives a result in one minute

What part of the brain controls heartbeat, respiration, and blood pressure? A) Cerebral cortex B) Cerebellum C) Hypothalamus D) Brain stem

D) Brain stem Explanation - The brain stem is made of the midbrain, pons, and medulla. It controls basic vital functions such as breathing, heartbeat, and blood pressure.

For which very contagious and rapidly fatal disease are ferrets vaccinated against? A) Feline panleukopenia B) Canine adenovirus C) Feline rhinotracheitis D) Canine distemper E) Canine bordetella F) Canine parvovirus

D) Canine distemper Explanation - Ferrets are very susceptible to the canine distemper virus. It is very contagious between ferrets and thus has a very high morbidity and mortality rate

The rear end of an animal is referred to as which of the following regions? A) Dorsal region B) Ventral region C) Cranial region D) Caudal region

D) Caudal region Explanation - Cranial is toward the head. Caudal is toward the tail. Ventral is toward the abdomen; dorsal is toward the backbone. Lateral is away from midline and medial is toward midline. Proximal is close to the spine or body while distal is away; these are typically used when describing limbs (e.g. fracture of the proximal femur vs. fracture of the distal femur). Palmar is the bottom of the paw of the forelimb; plantar is the bottom of the paw of the hind limb. Anterior is toward the head and posterior is away from the head. In veterinary medicine, anterior and posterior are usually used only to describe distal extremities or areas of the head.

What is the best construction form for a tortoise outdoor enclosure? A) Pressure treated wood B) Chain link fence C) Stacked rocks or pavers D) Cement blocks E) Chicken wire

D) Cement blocks Explanation - Cement blocks mortared together will prevent falling as tortoise digs, and is sturdy enough for even the biggest tortoise. Chain link fence is fine if the bottom of the fence is covered. If a tortoise can see through the fence, it will try to escape. Pressure treated wood may have chemicals in the wood that is bad for the tortoise. Chicken wire is not strong enough to hold a large tortoise. The rocks could cause injury if they fell on the tortoise while it was digging, or if it tried to climb.

Which of the following tests can be used to evaluate a patient for immune-mediated hemolytic anemia? A) Agarose immunodiffusion B) Latex agglutination C) Coggin's test D) Coomb's test

D) Coomb's test Explanation - A Coomb's test is used to detect the presence of immunoglobulins against the patient's red blood cells. The Coggin's test is used to diagnose equine infectious anemia. Latex agglutination tests and agarose immunodiffusions are tests that are used to detect the presence of specific patient antibodies

What is the best way to restrain a foal? A) It is too dangerous to restrain a foal, it is best just to corner them and not use restraint B) Remove the foal from its mother and cradle one hand under the neck and the other under the abdomen C) Use a halter D) Cradle one hand under the neck and grasp the base of the tail with the other hand

D) Cradle one hand under the neck and grasp the base of the tail with the other hand Explanation - It is best to keep the foal with the mother. Foals are unpredictable and not trained to use a halter. Best control is achieved when cradling one hand under the neck and grasping the tail base with the other hand

Ancylostoma causes which zoonotic disease in humans? A) Ocular larva migrans B) Crohn's disease C) Visceral larva migrans D) Cutaneous larva migrans

D) Cutaneous larva migrans Explanation - Ancylostoma is a hookworm and is the most common cause of Cutaneous larva migrans. This is an intestinal parasite of dogs and cats. In this zoonotic disease, the parasite burrows under the skin in humans. Ocular and Visceral larva migrans are caused by Toxocara canis (roundworm). Crohn's disease is a type of severe inflammatory bowel disease in people with no correlation to our pet patients

Which breed is over-represented and prone to intervertebral disk disease? A) St. Bernards B) Schnauzers C) Chihuahuas D) Dachshunds E) Labradors

D) Dachshunds Explanation - Due to their anatomy, Dachshunds' long backs make them more prone to having a disk protrusion that can compress the spinal cord. This is why it is necessary to recommend to owners of Dachshunds (and other breeds with this type of conformation - corgis, bassets, etc.) that the dogs should not jump on and off furniture or beds; this is one measure that may help to prevent such an injury

An 11-year old male German shepherd dog with previously diagnosed dilated cardiomyopathy (DCM) presents to your clinic following a dog fight. The dog is significantly aggressive and resistant to handling. What medication should NOT be used to sedate this patient? A) Butorphanol B) Atropine C) Midazolam D) Dexmedetomidine E) Buprenorphine

D) Dexmedetomidine Explanation - Dexmedetomidine should only be used in patients with normal cardiac function as it can cause a significant reduction in cardiac output due to reduced heart rate. It should be avoided whenever possible in patients that have compromised cardiac function. Butorphanol and buprenorphine would be better options for sedation as these drugs would not have as much effect on cardiac output and would provide some pain relief. Midazolam can be used to sedate the patient via IM administration and, although it would not provide any pain control, it would allow for safer handling of the patient. Atropine is not a sedative, but an anticholinergic used most commonly to increase heart rate

You are asked to take chest radiographs on a dog that has been hit by a car. You are reviewing the radiographs for technique and see that it appears the dog has intestinal contents in the chest cavity. What is this evidence of? A) Ruptured pericardial sac B) Broken ribs C) A rotated stomach D) Diaphragmatic hernia

D) Diaphragmatic hernia Explanation - A diaphragmatic hernia is a hole in the diaphragm. Through this hole, abdominal contents can protrude into the chest cavity. It can be helpful to hold an animal like this in an upright position to allow gravity to keep the abdominal contents out of the chest. The veterinarian should be notified immediately

The modified Knott's technique can be used to isolate the diagnostic stage of which of the following parasites? A) Giardia lamblia B) Isospora canis C) Strongyloides stercoralis D) Dirofilaria immitis E) Oxyuris equi

D) Dirofilaria immitis Explanation - The modified Knott's technique allows differentiation of microfilariae circulating in the bloodstream. It involves mixing blood with formalin, centrifuging, and analyzing the sediment under a microscope. Dirofilaria immitis is the only blood-borne parasite of the answer choices. The other parasite that can be recovered in this technique is Dipetalonema reconditum

In what position is colic surgery most commonly performed? A) Ventral recumbency B) Lateral recumbency C) Standing D) Dorsal recumbency

D) Dorsal recumbency Explanation - The correct answer is dorsal recumbency (horse is laying on his/her back). Dorsal recumbency usually provides the best access to the horse's organs when performing colic surgery. Placing a horse in dorsal recumbency requires a skilled team which is able to support all of the horse's needs. Padding will be extremely important to prevent any damage to muscle groups. A standing flank approach may be performed for select causes of colic but is not as common of a surgical position as dorsal. The patient is given enough sedation to be able to stand; also, a local anesthetic block along the skin and abdominal musculature is typically administered to prevent the horse from feeling pain

You are preparing to collect abdominal fluid from a horse presented for signs of colic. Collection of abdominal fluid for cytologic examination should be collected in what type of sample tube? A) Sodium citrate (blue top) B) Sodium heparin (green top) C) Clot tube (red top) D) EDTA (purple top)

D) EDTA (purple top) Explanation - Cells from fluid samples are best preserved and prevented from clotting in an EDTA tube

The doctor is going to spay a 6 pound Chihuahua and asks you to make sure the patient stays warm during and after the procedure. Which of the following methods is NOT appropriate to use as a heating source for anesthetized patients? A) Warmed 1 liter fluid bags B) Bair Hugger C) Warming bean bags D) Electric heating pad E) Water Circulating blanket

D) Electric heating pad Explanation - Severe burns may result from electric heating pads even when using a barrier between the pad and patient. In addition, the electric heating pads are not recommended for conscious animals because of the danger of electrocution from the patient bithing the cord

The best locations to administer intramuscular injections, such as penicillin, to a horse include all of the following EXCEPT? A) Pectorals B) Semitendinosus C) Semimembranosus D) Epaxial muscles E) Muscles of the neck

D) Epaxial muscles Explanation - The epaxial muscles, which are at the dorsum of the horse surrounding the vertebrae, are not an acceptable location for an intramuscular injection due to their relatively small size and location near the spinal cord. All the other injection sites are acceptable

Which type of cell should not be found in normal joint fluid? A) Neutrophils B) Lymphocytes C) Macrophages D) Erythrocytes

D) Erythrocytes Explanation - Normal synovial fluid should contain no erythrocytes and moderate numbers of nucleated cells (<3000/ul). The cells are normally about 95% mononuclear cells, with approximately equal numbers of lymphocytes and macrophages. There should be <10% neutrophils and very few eosinophils. Blood contamination can make it difficult to interpret synovial cytology

How often should an IV peripheral vein catheter be replaced as a standard? A) Every 5-7 days B) Don't replace as long as it is working fine C) Every 24 hours D) Every 3 days

D) Every 3 days Explanation - It is recommended to replace a peripheral catheter every 72 hours (3 days). The risk of catheter-induced sepsis goes up after this time period. Sometimes they have been left in for up to 4-5 days. Obviously, the site should be watched closely for any discharge, pain, and swelling and if these occur the catheter should be removed and another placed at a different site.

In which situation would perioperative antibiotics NOT be indicated? A) Surgery on a patient with hyperadrenocorticism B) Surgery on a patient treated with glucocorticoids C) An orthopedic procedure lasting 1 hour D) Excisional biopsy of a 2cm mass E) Surgery on a patient with diabetes mellitus

D) Excisional biopsy of a 2cm mass Explanation - Diabetes mellitus, hyperadrenocorticism, and chronic steroid therapy can all result in immunosuppression; these patients should be treated with perioperative antibiotics unless the surgeon specifically requested that an antibiotic not be used in the event that an intraoperative sample is needed for bacterial culture. Any patient having an orthopedic procedure involving the placement of implants should have perioperative antibiotics. A clean procedure performed under 90 minutes in an otherwise healthy and immunocompetent patient does not require perioperative antibiotics. An excisional biopsy of a 2cm mass would fall under this category.

Harderian glands are associated with which part of the anatomy? A) Anus B) Ears C) Bladder D) Eyes

D) Eyes Explanation - The Harderian gland is an accessory lacrimal gland on the inner side of the orbit in reptiles and birds; it is also present but is usually degenerate in mammals. Ears have ceruminous glands. There are perianal glands and apocrine glands associated with the anus of dogs and cats. The bladder is made up of transitional epithelium and is not glandular

A thoracic radiograph should be taken at which time? A) Full exhalation B) Systole C) Diastole D) Full inspiration

D) Full inspiration Explanation - Thoracic radiographs should be taken at maximal inspiration to obtain the clearest image of the lung fields. When the lungs are filled with air, the air provides maximal contrast between the normal lung and pathologic processes such as fluid accumulation or the presence of a mass. It is not usually possible to time a radiographic exposure based on the cardiac cycle

A Great Dane is presented for her ovariohysterectomy. The owner would also like to have a procedure done at the same time to secure her stomach to the body wall to prevent her from a gastric dilatation volvulus in the future. What is this procedure called? A) Gastric bypass B) Gastrotomy C) Gastrectomy D) Gastropexy

D) Gastropexy Explanation - "Pexy" is the term for surgical fixation. A gastropexy is attachment of the stomach to the abdominal wall to prevent the stomach from torsion or malposition. Gastrotomy is an incision into the stomach. Gastrectomy is the removal of the stomach. Gastric bypass procedure typically reduces the size of the stomach and allows food to bypass part of the small intestine. (This is mostly performed in people as a weight loss procedure.)

Which blood chemistry value may be artificially lowered if analysis of a blood sample is delayed? A) Sodium B) Alanine aminotransferase C) Creatinine D) Glucose

D) Glucose Explanation - Even after sample collection, glucose continues to be metabolized and, as a result, an artificially low blood glucose value can be expected if analysis is delayed

Anticholinergics can be used in small mammals as part of a preanesthetic protocol. Of the following, which anticholinergic is the best choice for use in rabbits? A) Midazolam B) Atropine C) Diazepam D) Glycopyrrolate

D) Glycopyrrolate Explanation - Both atropine and glycopyrrolate can be used in small mammals, but glycopyrrolate is preferred for use in rabbits. This is because the effect of atropine is less effective in this species because many animals have high levels of atropinesterase. Both diazepam and midazolam are benzodiazapines, not anticholinergics

Bacteria that stain red by safranin or basic fuchsin on a Gram stain are known as which of the following? A) Gram-red B) Gram-neutral C) Gram-positive D) Gram-negative

D) Gram-negative Explanation - Gram-positive bacteria retain the crystal violet-iodine stain at the beginning of a Gram stain. Gram-negative bacteria lose the violet color and stain red from safranin or basic fuchsin

A 2-year old Dalmatian presents after being hit by a car. Anisocoria is noted during the initial physical examination. What type of trauma would be suspected? A) Splenic torsion B) Ruptured bladder C) Pneumothorax D) Head trauma E) Orthopedic injury

D) Head trauma Explanation - Anisocoria is defined as unequal or asymmetrical pupil size and results most often from CNS trauma. Although other systems may have sustained damage, the most likely cause of anisocoria in a patient that was hit by a car is head trauma

What does Eimeria stiedai cause? A) Intestinal coccidiosis in dogs and cats B) It is a non-pathogenic coccidia in horses C) Cloacal coccidiosis in chickens D) Hepatic coccidiosis in rabbits

D) Hepatic coccidiosis in rabbits Explanation - Eimeria stiedai is the species of Eimeria that causes hepatic coccidiosis in rabbits. The rabbits ingest sporulated oocysts, which contain four sporozoites that hatch and travel via the hepatic portal vein to the liver and penetrate the bile duct epithelium. The schizonts in the bile duct cause bile duct hyperplasia; they block the bile duct, causing hepatomegaly, icterus, and liver failure.

Which of the following statements about ultrasound probe transducers is true? A) Higher frequency probes have decreased image resolution and decreased penetrating ability relative to lower frequency probes B) Higher frequency probes have decreased image resolution and increased penetrating ability relative to lower frequency probes C) Higher frequency probes have increased image resolution and increased penetrating ability relative to lower frequency probes D) Higher frequency probes have increased image resolution and decreased penetrating ability relative to lower frequency probes

D) Higher frequency probes have increased image resolution and decreased penetrating ability relative to lower frequency probes Explanation - The frequency of an ultrasound probe is usually measured in MHz (megahertz). Higher frequency probes (i.e. 7.5 MHz or 10 MHz) probes have increased image resolution but sacrifice penetration. Lower frequency probes (i.e. 5 MHz) are better for imaging deeper structures or cavities but do not provide as much image resolution

Phenylpropanolamine is used to treat urinary incontinence in dogs. It can commonly cause which side effect that should be discussed with the owner? A) Hypocalcemia B) Renal failure C) Profound hyperglycemia D) Hypertension

D) Hypertension Explanation - Phenylpropanolamine (Proin) can cause a high blood pressure (hypertension). If this occurs with the drug, a different treatment should be considered.

Yorkshire Terrier who has given birth 2 weeks ago presents with a history of tremoring. She is febrile and not eating well. On her electrolyte panel which of the following would you expect to see? A) Hypokalemia B) Hyponatremia C) Hypomagnesemia D) Hypocalcemia

D) Hypocalcemia Explanation - Eclampsia, also known as "milk fever" is caused by a low calcium level. It most often occurs around 3 weeks after whelping due to losing calcium in the milk and the inability of the body to keep up with demand. It is most common in small-breed dogs with larger litters. It is treated with calcium gluconate IV in an acute crisis and oral supplementation is then required. Milk fever is also seen more commonly in dairy cows that are close to calving. It typically causes a LOW temperature and often causes staggering and recumbency. It can also cause tremors and dilated pupils. It is also treated with injectable calcium. When cows are dry (not lactating), they should be kept on a LOW calcium diet so that calcium can be mobilized more rapidly from the bone when it is needed during calving and lactation

When looking under the microscope at a blood sample of a patient with iron deficiency, which of the following terms decribes what you would most likely see? A) Hypovolemia B) Hyponatremia C) Hyperchromasia D) Hypochromasia

D) Hypochromasia Explanation - Hypochromasia means lack of color or deminished color in the red blood cell. Iron is important in the formation of hemoglobin, and hemoglobin is responsible for the majority of color in the RBC. Therefore, a lack of iron would result in a pale red blood cell (hypochromasia). Hypovolemia is lack of volume of blood in the body. Hyperchromasia is inhanced or increased color in the red blood cells. Hyponatremia is low blood sodium

A urine sample with a specific gravity of less than 1.008 is considered to be which of the following? A) Isosthenuric B) Normosthenuric C) Hypersthenuric D) Hyposthenuric

D) Hyposthenuric Explanation - The correct answer is hyposthenuric. Urine samples between 1.008-1.012 are considered isosthenuric. Samples greater than 1.012 are hypersthenuric. Normal canine urine ranges from approximately 1.015-1.045

Where would you find pia mater? A) Inside the ear canals B) In the lining of the urinary bladder C) Under the tongue D) In the brain

D) In the brain Explanation - Pia mater, arachnoid, and dura mater are the three layers of the meninges of the brain. The meninges cover the brain and spinal cord and thus are found in the central nervous system

You are reading a report from an oncologist and read that the popliteal lymph nodes were enlarged. Where are these nodes located? A) Inside each thigh at the inguinal area B) At the chest area over the pectoral muscles C) On either side of the neck just under the jaw rostral to the salivary glands D) In the caudal portion of the hindlimbs at the level of the stifle and back of the thigh

D) In the caudal portion of the hindlimbs at the level of the stifle and back of the thigh Explanation - The popliteal nodes are located on the back of the thigh just caudal to the stifle. The pre-scapular nodes are located at the chest over the pectoral muscles. The mandibular nodes are located under the jaw rostral to the salivary glands. The inguinal nodes are located inside each thigh at the inguinal region

Which of the following is expected from a positive inotropic drug? A) Dramatic decrease blood pressure B) Decrease in adrenal gland stimulation C) Prevention of arrhythmias D) Increase in cardiac

D) Increase in cardiac Explanation - Positive inotropes mostly work by making more calcium available to the muscle and thus increase cardiac contractility. Digoxin is a positive inotrope and acts by inhibiting the sodium-potassium ATPase pump of the cells. Toxicity can cause arrhythmias and a slow heart rate. It should be avoided or used with extreme caution in patients with renal disease or hypokalemia because it increases the risk of toxicosis

A "UMIC" would be performed on a urine sample to check for which of the following? A) Diabetic regulation level B) Crystalluria C) Ketonuria D) Infection and antibiotic susceptibility

D) Infection and antibiotic susceptibility Explanation - A UMIC is a urine minimum inhibitory concentration. This is the lowest concentration of an antimicrobial that will inhibit the visible growth of a microorganism after incubation. It basically indicates which bacteria are present and to which antibiotics those organisms are susceptible

Where are the anal sacs located? A) Inside the anus at the 12 and 6 o'clock positions B) The dorsal external anus region C) In the large intestine just caudal to the cecum D) Inside the anus at the 4 and 8 o'clock positions

D) Inside the anus at the 4 and 8 o'clock positions Explanation - Anal sacs (also referred to as anal glands) are located just inside the anus at the 4 and 8 o'clock positions. Many dogs have difficulty expressing these sacs, which should normally occur during a bowel movement. Thus, manual expression is needed and is a procedure frequently provided by technicians. A symptom of full or irritated anal sacs is scooting or licking the rear end.

In which part of the intestinal tract is nutrient absorption the greatest? A) Colon B)Duodenum C) Ileum D) Jejunum

D) Jejunum Explanation - The jejunum is the longest portion of the small intestine and is the site of the most nutrient absorption

A toggle-pin fixation for a luxated hip is going to be performed this afternoon. How will the patient be ultimately positioned for surgery after it is has surgically prepped? A) Ventral recumbency B) Dorsal recumbency C) Hanging leg technique D) Lateral recumbency

D) Lateral recumbency Explanation - A toggle-pin fixation is performed to reduce a luxated hip. This is the most common surgical procedure performed for repair of a luxated hip. The approach is made with a curvilinear incision over the lateral aspect of the coxofemoral joint; it is therefore best to have the patient in lateral recumbency. A hanging leg technique would be performed to aseptically prep the limb prior to surgery, but the limb would not remain in that position for the duration of the surgery and is therefore not the best answer choice

What structure in the eye is responsible for focus? A) Cornea B) Iris C) Retina D) Lens E) Sclera

D) Lens Explanation - The lens helps to refract light to be focused on the retina. By changing its shape, it changes the focal distance. The lens is a transparent biconvex disc located in the anterior chamber of the eye. The iris regulates the amount of light entering the eye by controlling the pupil size. The iris is the colored part of the eye. The retina contains photoreceptors (rods and cones) and is the location of the optic nerve. The sclera is a fibrous tunic and is the white part of the eye. The cornea is transparent and covers the front part of the eye. It also helps to refract light.

Which setting is better for an x-ray of a part of the body with high contrast between tissues, like a hip x-ray of a young dog? A) Low kilovoltage (kVp); Low milliamp-seconds (mAs) B) High kilovoltage (kVp); Low milliamp-seconds (mAs) C) High kilovoltage (kVp); High milliamp-seconds (mAs) D) Low kilovoltage (kVp); High milliamp-seconds (mAs)

D) Low kilovoltage (kVp); High milliamp-seconds (mAs) For bone, which has a lot more natural contrast because it is so hard, use a lower kVp and a higher mAs. For soft tissues (like the abdomen) use a higher kVp and lower mAs settings. Kilovoltage (kVp) affects the scale of contrast on an x-ray. The scale of contrast refers to the number of shades of gray that you can see. Generally, increasing kVp increases film blackness and contrast.

In normal occlusion, what is the proper position of the incisors and canine teeth when the mouth is closed? A) Mandibular incisors are palatal to the maxillary incisors, and the mandibular canine is distal to the maxillary canine B) Mandibular incisors are buccal to the maxillary incisors, and the mandibular canine is distal to the maxillary canine C) Mandibular incisors are buccal to the maxillary incisors, and the mandibular canine is mesial to the maxillary canine D) Mandibular incisors are palatal to the maxillary incisors, and the mandibular canine is mesial to the maxillary canine

D) Mandibular incisors are palatal to the maxillary incisors, and the mandibular canine is mesial to the maxillary canine Explanation - In normal occlusion, the mandibular incisors are palatal (behind) the maxillary incisors, and the coronal third of the mandibular incisors rests on a smooth convex bulge on the maxillary incisors known as the cingulum. The mandibular canines should be centered between the maxillary third incisor and the maxillary canine (mesial to the maxillary canine), without touching either tooth

Which type of gas is formed due to rumenal fermentation? A) Sevoflurane B) Octane C) Oxygen D) Methane

D) Methane Explanation - Microbial fermentation of feed leads to production of methane gas. Carbohydrate digestion in the rumen (anaerobic methanogenesis) causes the highest amount of methane gas to be released.

An animal with inflammation of the uterine lining is said to have which of the following conditions? A) Pyometra B) Uterinitis C) Cystic ovaries D) Metritis

D) Metritis Explanation - Metritis is inflammation of the lining of the uterus (the endometrium). Pyometra is pus in the uterus

72 hours have passed since performing and exploratory laparotomy. It is important to evaluate the incision on a daily basis to help detect any complications early. Which of the following would you not expect to find on a 3-day-old incision? A) Moderate pain B) Moderate swelling C) Moderate redness D) Mild serosanguinous discharge

D) Mild serosanguinous discharge Explanation - An incision will demonstrate all the classic signs of inflammation (redness, swelling, pain, and heat). Occasionally, there may be mild serosanguinous discharge from an incision, but this usually resolves in 24-48 hours. Persistence of discharge is worrisome as this may perpetuate bacterial colonization and subsequent incisional failure

What is the vector for transmission of heartworm disease? A) Fly B) Tick C) Flea D) Mosquito

D) Mosquito Explanation - The heartworm, Dirofilaria immitis, is transmitted by infected mosquitoes (the mosquito is the intermediate host). The mosquito ingests microfilariae from the blood of an infected dog (an L1). Then the microfilariae mature in the mosquito to an infective state (L3). The mosquito punctures the skin, and the L3 enter into the new host where they begin migrating and maturing. The adults arrive in the heart about 110 days after infection. The females begin to produce the microfilariae about 6 months after infection, at which time the cycle can begin again via a mosquito bite

What is the basic structural and functional unit of the kidney? A) Glomerulus B) Neuron C) Hepatocyte D) Nephron

D) Nephron Explanation - The nephron's job is to regulate the concentration of water and salts by filtering the blood, absorbing what is needed, and excreting the rest as urine. It eliminates waste from the body, regulates blood volume and blood pressure, controls electrolyte and metabolite levels, and regulates blood pH. The glomerulus is a cluster of vessels located within Bowman's capsule and is the main filter of the nephron. Hepatocytes are liver cells.

What is the preferred stain to identify reticulocytes and Heinz bodies in erythrocytes? A) Wright's stain B) Romanowsky stain C) Wright's-Giemsa stain D) New methylene blue

D) New methylene blue Explanation - Reticulocytes and Heinz bodies are most reliably identified using a new methylene blue stain. Since this is not routine on a CBC, a separate new methylene blue stain should be performed when a reticulocyte count is indicated or when oxidative damage is suspected. Brilliant cresyl blue stain is another option for visualizing Heinz bodies

You are restraining a horse with a harness while the veterinarian is examining a sore on the lateral side of the left pelvic limb. Where should you stand while holding this horse? A) On the right side, hold lead rope with little slack B) On the left side, hold lead rope and leave enough slack so that the horse may turn his head around to see what the veterinarian is doing C) The horse should be tied up and you should stand to the right of the veterinarian at the hind end of the horse to prevent the horse from kicking D) On the left side, hold lead rope with little slack

D) On the left side, hold lead rope with little slack Explanation - When handling a horse, the holder and the person working on the horse should be standing on the same side. If the exam starts on the left, stand on the left; as the examiner moves to the right side, you should also move to the right side. There should be little slack in the lead rope to prevent the horse from moving

Bone is made by which type of cell? A) Chondrocytes B) Osteocytes C) Osteoclasts D) Osteoblasts

D) Osteoblasts Explanation - Osteoblasts are mononucleate cells that are responsible for bone formation and come from the bone marrow. Osteocytes are actually cells inside of the bone. Osteoclasts are large cells that dissolve the bone. Chondrocytes are cartilage cells.

All of the following pieces of information legally must be present on a radiographic film EXCEPT for which of the following? A) Name of practice B) Date of radiograph C) Name of patient and owner D) Patient breed

D) Patient breed Explanation - A radiograph is part of the legal medical record. As such, it is necessary to identify the patient it was taken from, the date of the exam, and the name of the practice. It is frequently helpful to use a marker to orient the radiograph or to identify which extremity is being examined. For film radiographs, frequently patient information is imprinted by the use of special graphite-impregnated tape where the information can be written and placed on the radiograph cassette or by a light flasher system that exposes printed patient information to the film. For digital radiographs, the patient information must be entered into the computer so that it is saved along with the digital radiograph. Additional patient information such as breed can be helpful but is not required

Which medication given for diarrhea can cause the stools to appear dark in color, like melena? A) Tylosin B) Sucralfate C) Metronidazole D) Pepto Bismol E) Forti-flora

D) Pepto Bismol Explanation - Pepto Bismol and some formulations of KaoPectate contain bismuth subsalicylate, which causes darkening of the stool and acts by anti-secretory effects on the intestine. Sucralfate is given to help sooth esophageal and gastric ulcers. Metronidazole and Tylosin are antibiotics. Forti-flora is given to help restore the normal gastrointestinal flora.

Which of the following would most commonly be enlarged on chest films of a heartworm positive dog? A) Aorta B) Lymph nodes C) Left atrium D) Pulmonary arteries

D) Pulmonary arteries Explanation - Heartworm disease causes right heart enlargement and enlargement of the pulmonary arteries, which are often seen on chest radiographs in pets with heartworm disease. Right heart enlargement gives a reverse-D appearance to the heart

A urine protein:creatinine ratio may be requested for which purpose? A) Test for bilirubin in the urine B) Look for urinary tract infection C) Measure the glomerular filtration rate D) Quantify urine protein

D) Quantify urine protein Explanation - Urine protein:creatinine ratio is helpful when quantifying the severity of proteinuria. Urine reagent test strips are not sufficiently quantitative, and the sulfosalicylic acid turbidity test merely provides a measure of the amount of protein in a single urine sample

Which of the following animals develops dental caries? A) Horses and cats B) Pigs and reptiles C) Goats and camelids D) Rabbits and chinchillas E) Cattle and sheep

D) Rabbits and chinchillas Explanation - Rabbits, chinchillas, and dogs develop dental caries. Caries are very rare in the other species listed. Caries are cavities

A dog sustains a humeral fracture. Which nerve would most likely be damaged? A) Sciatic nerve B) Ulnar nerve C) Suprascapular nerve D) Radial nerve

D) Radial nerve Explanation - The radial nerve innervates the extensor and supinator muscles located in the forelimb and provides distal sensation. This is the most commonly injured nerve in a forelimb fracture. The sciatic nerve runs along the hind limbs and can be immediately ruled out. The suprascapular nerve is higher up the forelimb and less likely to be injured. The ulnar nerve runs along the medial aspect of the humerus and is thought to be more protected than the radial nerve which courses along the lateral aspect of the humerus

What do spherocytes look like under the microscope? A) White blood cells with solid round nuclei B) Red blood cells with blue round inclusion bodies C) Red blood cells that are spiculated D) Red blood cells that are small, very dark, and perfectly round E) White blood cells with blue round inclusion bodies

D) Red blood cells that are small, very dark, and perfectly round Explanation - The spherocytes are smaller than normal red blood cells due to their loss of membrane, and there is no pale area in the center. The most specific disease showing this change in the RBCs is Immune Mediated Hemolytic Anemia. Other hemolytic conditions can also cause this change to occur. Echinocytes are spiculated red blood cells that are sometimes seen in snake bite cases

A 3-year old castrated male Doberman Pinscher comes in to the emergency clinic with signs consistent with gastric dilation and volvulus (GDV). Following standard initial emergency treatment for suspected gastric dilation and volvulus, what imaging procedure would be most helpful in confirming the diagnosis? A) Abdominal ultrasound exam B) Barium contrast abdominal radiograph C) Dorsal recumbency abdominal radiograph D) Right lateral recumbency abdominal radiograph E) Ventral recumbency abdominal radiograph F)Ventral dorsal abdominal radiograph

D) Right lateral recumbency abdominal radiograph Explanation - A typical GDV occurs with repositioning of the pylorus to the left dorsal abdomen. A right lateral radiographic image is the best perspective position for revealing a-gas filled right dorsally-displaced pylorus with a gas-filled ventral fundus separated by a soft tissue band (compartmentalized stomach). Although the features of a malpositioned stomach may be observed on the other radiographic views, they are often more difficult to interpret than that represented by the right lateral recumbency view. The severe amount of gas within the GI tract that is associated with GDV would make ultrasound exam interpretation extremely difficult at best

Name the heart chamber responsible for pumping blood toward the lungs. A) Right atrium B) Left atrium C) Left ventricle D) Right ventricle

D) Right ventricle Explanation - The right atrium receives deoxygenated blood from the vena cava. This blood is then pumped into the right ventricle (passing through the tricuspid valve) which then gets pumped into the lungs (passing through the pulmonary valve) via the pulmonary artery for oxygenation. The newly oxygenated blood returns to the left atrium via the pulmonary vein. Once in the left atrium, the blood is pumped into the left ventricle (passing through the atrioventricular valve) from which it is then pumped into the circulation via the aorta (passing through the aortic valve

Which of the following is an example of an endodontic treatment? A) Soft palate resection surgery B) Dental scaling C) Tumor removal from the tongue D) Root canal

D) Root canal Explanation - Endodontics deals with the tooth pulp and the tissues surrounding the root of a tooth. The root canal is the most common endodontic procedure

The shoulder joint links which two bones? A) Humerus and ulna B) Humerus and radius C) Radius and ulna D) Scapula and humerus

D) Scapula and humerus Explanation - The shoulder joint connects the scapula and the humerus. The head of the humerus sits at glenoid cavity of the scapula to form the shoulder joint. The elbow would connect the humerus to the radius and ulna.

What is the main effect of Dopram (doxopram hydrochloride)? A) Slows heart rate B) Stops seizures C) Induces emesis D) Stimulates respiration

D) Stimulates respiration Explanation - Dopram (doxopram hydrochloride) is a potent respiratory stimulant. In veterinary medicine, it is used to aid in diagnosis of laryngeal paralysis

You find coccidia (Isospora) on a fecal float for a puppy that presents with mucoid diarrhea with streaks of blood. What medication is used to treat coccidia? A) Praziquantel B) Fenbendazole C) Metronidazole D) Sulfadimethoxine E) Pyrantel

D) Sulfadimethoxine Explanation - Sulfadimethoxine (Albon) is used to treat coccidia in dogs and cats

A feline patient presents for onychectomy. What does this entail? A) Anal gland expression B) Nail trim C) Neuter D) Surgical nail removal E) Bladder stone removal

D) Surgical nail removal Explanation - Onychectomy, mostly known as declawing, is an operation to surgically remove an animal's nails by means of amputating the distal phalanx

What does bog spavin describe? A) Abscessation of the hoof wall B) Rotation of the coffin joint C) Inflammation of the superficial digital flexor tendon D) Tarsocrural effusion

D) Tarsocrural effusion Explanation - Bog spavin is a term used in the equine field. It describes the accumulation of synovial fluid in the tarsocrural joint. Rotation of the coffin joint occurs with laminitis. "Bowed tendon" and tendonitis are the descriptive terms for inflammation of a tendon. If in the correct location the abscessed hoof wall may be termed a subsolar abscess. These result in extreme pain and lameness and must be distinguished from laminitis

Which of the following conditions causes muscle rigidity and a "saw horse"-like stance? A) Immune-mediated thrombocytopenia (ITP) B) Myasthenia gravis C) Cushing's disease D) Tetanus

D) Tetanus Explanation - Tetanus, also known as "lock-jaw", causes muscle rigidity. Animals with tetanus are light-sensitive and have a "saw horse" stance. Tetanus is caused by the bacteria Clostridium tetani and can be present in the soil, gaining access to the body through an open or penetrating wound. Horses should be vaccinated against tetanus

Which of the following errors in taking and processing a radiograph would result in a completely clear film? A) The collimator was left all the way open B) The film was exposed to light after x-ray exposure due to a leak in the darkroom C) The cassette was not closed completely and the film was exposed to light prior to x-ray exposure D) The exposed film was put in fixer before developer

D) The exposed film was put in fixer before developer Explanation - In normal radiograph processing, silver halide crystals in the film that are sensitized by exposure during the x-ray are converted to black metallic silver in the developer. The remaining crystals are removed by the fixer. If a film is accidently run through the fixer first, all of the silver halide crystals will be removed, leaving a clear image. The other choices will all increase exposure of the film, making it darker

A pet-sitter is holding a 4-year old male intact Bichon while the veterinarian examines a rash on the dog's abdomen; you are assisting with a skin scraping. The dog reaches back and bites the pet-sitter on the arm. She had insisted on holding the dog for examination despite the posted sign saying "for your safety please do not ask to restrain your animal". This dog has been known to be aggressive in the past. If the pet-sitter seeks compensation for being bit, who is most likely to be found liable for her injury? A) The pet-sitter is liable and therefore no damages can be claimed B) You, the technician C) The dog's owner D) The veterinarian E) This would be considered an accident and fault of both parties, therefore no one is liable.

D) The veterinarian Explanation - Even though the owner insisted on holding the dog for the exam, the veterinarian could still be liable for the owner's injury. While most clients would not pursue damages for an incident such as this, courts have ruled that veterinarians are liable if a pet hurts its owner while the two are located on the premises of a veterinary hospital. The best solution to prevent this is to have a technician or assistant trained to restrain animals hold the patients for examination. It is a good idea for a veterinary practice to post a sign in each exam room stating that owners please not ask to restrain pets during exam and this rule should be enforced. In animals that are known to be aggressive, a muzzle should be placed prior to the exam for the safety of all involved.

You are asked to take a lateral oblique radiograph of a horse's fetlock with a portable x-ray unit. Where will the x-ray unit be and where should the film cassette be? A) The x-ray beam should come from the more dorsal side of the limb, and the film will be on the more lateral aspect of the limb. B) The x-ray beam should come from the more plantar side of the limb, and the film will be on the more medial aspect of the limb. C) The x-ray beam should come from the more dorsal side of the limb, and the film will be on the more lateral aspect of the limb. D) The x-ray beam should come from the more dorsal side of the limb, and the film will be on the more medial aspect of the limb.

D) The x-ray beam should come from the more dorsal side of the limb, and the film will be on the more medial aspect of the limb. Explanation - A lateral oblique is also referred to as a DLPMO or dorsolateral-palmaro/plantaromedial oblique. With this technique, the beam comes from 45 degrees between dorsal and lateral sides of the limb, and the film is placed 45 degrees between the medial aspect and the palmar (or plantar) aspect

You are running some blood tests, and the veterinarian asked that the pet be fasted. Why is this necessary? A) To prevent icterus B) To prevent clotting C) To prevent hemolysis D) To reduce lipemia

D) To reduce lipemia Explanation - Some test results require fasting for accuracy. Fasting an animal helps to reduce the lipemia (fats) in the blood that can interfere with testing. This is especially important when checking values such as triglycerides and cholesterol

When a tortoise is kept in an indoor enclosure, what is the most important light in that enclosure? A) Heat Lamp B) UVA C) Incandescent lightbulb D) UVB E) Basking bulb

D) UVB Explanation - A UVB light is needed to properly process and absorb calcium from the diet. Sunlight is the natural way to process calcium. Other lighting is also needed for proper digestion. A basking bulb and heat lamp help regulate temperature, and UVA light helps regulate activity

A cystogram is a radiographic study performed to identify the position, shape, and lesions involving what structure? A) Skin and subcutis B) Kidney C) Esophagus D) Urinary bladder E) Liver

D) Urinary bladder Explanation - A cystogram is a radiograph of the urinary bladder region taken after injection of positive-contrast medium directly into the bladder via a urinary catheter

Which condition occurs least commonly in the horse? A) Laminitis B) Intestinal incarceration C) Enterolithiasis D) Urolithiasis

D) Urolithiasis Explanation - As compared to small animals, urolithiasis is uncommon in horses. If they do develop stones in their bladder, it is usually secondary to formation of calcium oxalate crystals. Enterolithiasis and intestinal incarcerations are common reasons for colic in horses and usually require surgical intervention. Laminitis is a very serious and often life-threatening condition in horses. The pathophysiology of laminitis is poorly understood but involves inflammation of the lamina of the feet. It can involve all feet or even just one foot. There are multiple conditions that can lead to laminitis, such as endotoxemia, grain overload, and pneumonia.

You are attempting to give an intra-jugular injection to a horse and insert the needle to the hub. Blood is forcefully pulsating and ejecting from the needle. What is the most likely explanation? A) This is normal when injecting medication into the jugular of a horse B) The horse has hypertension C) The vein is spasming, and this is normal; it is okay to give the medication D) You are in the carotid artery

D) You are in the carotid artery Explanation - This is typical of insertion into an artery. If this occurs, the needle should be removed immediately and pressure placed on the site for several minutes to assure hemostasis

Which of the following solutions used for fecal flotation floats protozoal organisms with the least distortion? A) Sodium nitrate B) Formalin C) Saturated sodium chloride D) Zinc sulfate

D) Zinc sulfate Explanation - Zinc sulfate solutions are generally used in diagnostic laboratories because they cause the least distortion of organisms. Other solutions, particularly sodium nitrate, are commonly used in veterinary practices because of cost and/or convenience. It is particularly important with sodium nitrate solutions to avoid letting them sit too long, as they cause distortion and form crystals, particularly after sitting longer than 20 minutes. Saturated sodium chloride is rarely used because it distorts organisms, creates crystals, and corrodes laboratory equipment. Formalin solutions alone are not used for flotation.

When taking a lateral thoracic radiograph of a dog, where should the field be centered? A) 2nd rib and the manubrium of the sternum B) 9th rib and the xiphoid process of the sternum C) 7th rib and the spine of the scapula D) 3rd rib and the cranial border of the scapula E) 5th rib and caudal border of the scapula

E) 5th rib and caudal border of the scapula Explanation - A lateral thoracic radiograph should be taken by drawing the forelimbs forward to prevent them from overlapping the cranial thorax. The film should be centered at the 5th rib and caudal border of the scapula; the film should also be collimated to include the manubrium (as the cranial landmark) and half way between the xiphoid and the last rib (as the caudal landmark). The head and neck should be in a natural position, neither extended nor flexed, and the sternum and dorsal spinous processes should be in a plane parallel to the table, indicating that the animal is not rotated. All lung fields should be included, and the film should be taken on maximum inspiration

Which disease can cause a severe bradycardia that could be life-threatening? A) Hypothyroidism B) Portosystemic shunt C) Hyperthyroidism D) Cushing's disease E) Addison's disease

E) Addison's disease Explanation - Hypoadrenocorticism (Addison's disease) can cause a high potassium level (hyperkalemia) and can be life-threatening. Addison's occurs when the adrenal glands do not produce enough cortisol, resulting in a crisis. A low heart rate is often associated with high potassium. Sodium chloride is the fluid treatment of choice to help reduce the potassium

In order to prevent distortion on a dental radiograph when parallel technique cannot be used (impossible to position the film parallel to the long axis of the tooth), which of the following will prevent image distortion (elongation or foreshortening)? A) Volume averaging technique B) Perpendicular technique C) Horizontal technique D) Vertical technique E) Bisecting angle technique

E) Bisecting angle technique Explanation - In most radiographs, to avoid distortion (elongation or foreshortening), the x-ray beam should be perpendicular to the long axis of the bone and the x-ray cassette. If the bones are not perpendicular to the beam, they will appear foreshortened. If the cassette is not perpendicular to the beam, the bones will appear elongated. In dental radiography, it is frequently impossible for the long axis of the tooth and the film to be parallel to each other. In this case, the bisecting angle technique may be used, where the beam is angled half-way between the angle of the long axis of the tooth and the angle of the film in order to obtain a true image that is neither elongated nor foreshortened. By angling half-way between those two angles, this technique effectively balances out the two types of distortion to obtain a true image

When dogs are mating, the male dog "ties" with the female so that the penis cannot be removed from the vagina for a period of time. This is due to the swelling of which structure? A) Os penis B) Epididymis C) Prostate D) Scrotum E) Bulbis glandis

E) Bulbis glandis Explanation - The bulbis glandis is erectile tissue on the penis that swells during mating and locks or ties the penis in the vagina

Which of the following is another term for the type of parasite known as a tapeworm? A) Ascarid B) Protozoan C) Nematode D) Trematode E) Cestode

E) Cestode Explanation - Tapeworms or cestodes are long, segmented, flat worms. Examples include the canine tapeworms, Taenia pisiformis and Dipylidium caninum. Trematodes are flukes, nematodes are roundworms, ascarids are a type of nematode (roundworm), and protozoans are single-celled organisms that may be parasitic

Which of the following hormones is produced by the adrenal glands? A) Thyroid stimulating hormone (TSH) B) Antidiuretic hormone (ADH) C) Adrenocorticotropic hormone (ACTH) D) Growth hormone (GH) E) Cortisol

E) Cortisol Explanation - Cortisol is a steroid hormone, or glucocorticoid, produced by the adrenal glands. The other hormones listed here are all produced in the pituitary gland

Giardiasis may be the cause of which clinical sign? A) Panting B) Seizures C) Urticaria D) Coughing E) Diarrhea

E) Diarrhea Explanation - Giardia is a protozoan parasite that often causes a watery diarrhea. It may sometimes also cause vomiting. Seizures, urticaria (hives), coughing, and panting are not typical symptoms of giardiasis

When drawing blood for a fructosamine level in a canine, how many hours do you have to wait after insulin is given to draw the blood? A) 12 hours B) 8 hours C) 6 hours D) 1 hour E) Doesn't matter

E) Doesn't matter Explanation - A fructosamine measurement is a blood test which indicates the average glucose concentration over the past 1-2 weeks. Since it measures an average it is not time sensitive. It is used to assist in monitoring the effectiveness of insulin therapy in diabetic pets, and is useful in differentiating stress hyperglycemia from diabetes. A glucose curve is the other common way to monitor diabetic regulation. With a glucose curve, a series of blood glucose values are taken (typically every 2 hours) to help determine how effective the insulin is reducing the glucose level. It helps to determine when the glucose starts to decrease and then increase again (the low point is called the nadir), thus determining how long the insulin works in the patient. By doing this curve it helps the doctor determine if the amount of insulin needs to be increased or decreased and the frequency it needs to be given

Which breed is most at risk for developing Degenerative Myelopathy? A) Corgi B) Standard Poodle C) Golden Retriever D) Dachshund E) German Shepherd

E) German Shepherd Explanation - This is a progressive spinal cord disease that causes that myelin sheath around the nerves to degenerate. German Shepherds are over-represented for this condition. Dachshunds most commonly get intervertebral disk disease

Which species has wolf teeth? A) Dogs B) Pigs C) Cats D) Cows E) Horses

E) Horses Explanation - Wolf teeth are small teeth that are the first premolars in the horse. It is thought they may be deciduous teeth. They are located immediately rostral to the first cheek teeth in horses

What term would describe inflammation of the kidneys? A) Gastritis B) Blepharitis C) Hepatitis D) Cystitis E) Nephritis

E) Nephritis Explanation - Inflammatory conditions typically end in "-itis". Nephritis is inflammation of the kidneys. Hepatitis is inflammation of the liver. Cystitis is inflammation of the bladder. Blepharitis is inflammation of the eyelids/follicles. Gastritis is inflammation of the stomach

Which of the following tests amplifies DNA? A) Histopathology B) ELISA (Enzyme-linked immunosorbent assay) C) Flow cytometry D) Cytology E) PCR (Polymerase chain reaction)

E) PCR (Polymerase chain reaction) Explanation - PCR is a method of amplifying a segment of nucleic acid (DNA or RNA) segments. ELISA is a test that detects the presence of antibodies or antigens. Flow cytometry is a technique used to sort cells. Cytology and histopathology are qualitative tests to look at cellular morphology under the microscope

A Gram stain causes Gram-negative bacteria to turn which color? A) Blue B) Orange C) Purple D) Green E) Pink

E) Pink Explanation - Gram staining is the method of differentiating Gram-positive and Gram-negative bacteria based on cell wall properties. Gram staining stains Gram-positive bacteria blue/purple and Gram-negative bacteria pink/red

A 10-year old male intact Rottweiler is currently undergoing surgery for removal of a prostatic mass. Which term best describes the procedure? A) Orchidectomy B) Cholecystectomy C) Prostatitis D) Onychectomy E) Prostatectomy

E) Prostatectomy Explanation - There are a few basic rules that will allow you to comprehend a majority of the medical terms. Any term that ends with "-ectomy" infers to removal of that organ. Onychectomy is another word declawing or removal of the distal phalanx. Orchidectomy is the removal of a testicle. A cholecystectomy is removal of the gall bladder. The ending "-itis" means inflammation. In this case, prostatitis would be inflammation of the prostate

Clipping an abdomen for a ventral midline laparotomy is one of the more common procedures done as a surgical nurse working with dogs and cats. Which of the following statements is most accurate? A) The hair should be clipped from the xiphoid to the base of the pubis in all males B) Only enough hair should be clipped to perform the procedure C) The abdomen should be clipped from the manubrium to the pubis with 2 cm margins cranial to the manubrium and 2 cm caudal to the pubis D) The abdomen should be clipped widely with at least 2 cm margins cranial to the xiphoid and 2 cm caudal to the pubis

E) The abdomen should be clipped widely with at least 2 cm margins cranial to the xiphoid and 2 cm caudal to the pubis Explanation - The abdomen should be clipped widely with at least 2 cm margins cranial to the xiphoid and 2 cm caudal to the pubis. The manubrium is the most cranial bone of the sternum and would imply clipping the chest, which is not necessary for a laparotomy. Occasionally surgical complications necessitate the lengthening of an incision; in effect, it is important to clip widely to allow the surgeon the flexibility to extend the incision if necessary

You perform an ear cytology showing large numbers of Malassezia. Which of the following types of organisms is this? A) Gram-positive bacteria B) Gram-negative bacteria C) Acid-fast bacteria D) Ear mite E) Yeast

E) Yeast Explanation - Malassezia pachydermitis is a yeast/fungus very common in skin and ear infections. It resembles a bowling pin under the microscope. The ear mite is Otodectes

What is the "top" shell of a tortoise called? A) Patagium B) Cloaca C) Plastron D) Scute E) Choana F) Carapace

F) Carapace Explanation - The carapace is the top shell and the plastron is the bottom shell. Patagium is the skin or membrane that extends between the body and a limb or wing (such as seen in a flying squirrel or wing of a bat). A scute is a bony external plate or scale. A choana is either of the pair of posterior openings between the nasal cavity and nasopharynx. The cloaca is the posterior opening that serves to pass feces, urine, and reproductive excretions (connects the rectum, vagina, and urethra into a single channel), such as in reptiles and birds

"Scabies" in dogs generally refers to skin disease caused by which type of organism? A) Bacterium B) Fly larva C) Flea D) Protozoa E) Louse F) Mite

F) Mite Explanation - Scabies refers to infestation with Sarcoptes scabiei var. canis, which is a skin mite that can cause severe pruritus and skin lesions. It is usually seen in dogs and rarely in cats. It may also cause less severe lesions in humans

A 620-kg horse weighs how many pounds? *A* 1,364 pounds *B* 1,625 pounds *C* 281 pounds *D* 1,240 pounds

*A* 1,364 pounds Explanation - There are 2.2 pounds per kg. So, to convert the kg into pounds, multiply the kg by 2.2. 620 kg X 2.2 pounds/kg = 1364 pounds

How tall would a horse that measures 66 inches be, in hands? *A* 16 - 2 hands *B* 14 - 3 hands *C* 16 hands *D* 15 hands

*A* 16 - 2 hands Explanation - A hand is equivelent to 4 inches. Thus 66 divided by 4 = 16 hands + 2 inches or 16 - 2 hands

You have just completed a manual differential and obtained the values below. The total white blood cell count is 21,800. Segmented Neutrophils 88% Lymphocytes 3% Monocytes 8% Eosinophils 1% What is the absolute value of the Segmented Neutrophils? *A* 19,184 *B* 1,918,400 *C* 247.72 *D* 88

*A* 19,184 Explanation - The percentage of each white blood cell type counted is multiplied by the total white blood cell count to determine the absolute value of each of the cell types present. 21,800 x 0.88= 19,184 Lymphocyte 654 Monocyte 1744 Eosinophil 218 The total of all absolute values should equal the total white blood cell count

What is the ideal age for tail docking in puppies? *A* 3 to 5 days *B* 2 to 3 weeks *C* 4 to 6 months or at the time of sterilization *D* At birth

*A* 3 to 5 days Explanation - Tail docking in puppies should occur within 3 to 5 days after birth

A 38-kg Great Dane is recovering from Gastric Dilatation Volvulus surgery. The doctor would like to send him home on Tramadol. The doctor requested that a prescription be filled out for 5 days at 3 mg/kg orally three times per day. How many pills should you send home? Tramadol is available in 50 mg tablets. Round to the nearest whole number. *A* 30 tablets *B* 40 tablets *C* 25 tablets *D* 50 tablets

*A* 30 tablets Explanation - The correct answer is 30 tablets: 38 kg x 3 mg/kg = 114 mg 114 mg / 50 mg = 2.28 (Rounding to the nearest whole number would be 2 tablets) If getting 2 tablets per dose, then 6 tablets per day are needed. Five days' treatment would be 30 tablets

Anesthetic monitoring of pet patients is one of the most important veterinary technician duties. How often should vitals be checked and recorded? *A* 5 minutes *B* 10 minutes *C* 20 minutes *D* 1 minute

*A* 5 minutes Explanation - Most places require that readings be taken at 5 minute intervals. The requirements may vary by state or facility

A reticulocyte count should be included on a complete blood cell count in which of the following instances? *A* A 13-year old cat with a packed cell volume of 19% *B* A 9-year old horse with a total white blood cell count of 2,000/ul *C* An 8-year old cat with a platelet count of 30,000/ul *D* A 3-year old dog with a total white blood cell count of 32,000/ul

*A* A 13-year old cat with a packed cell volume of 19% Explanation - A reticulocyte count is performed to assess whether a patient is generating a bone marrow response to anemia. Reticulocytes are immature red blood cells that are released from the bone marrow when the marrow is stimulated to increase red blood cell production. Therefore, anemia (or a low packed cell volume) is the primary reason to perform a reticulocyte count

What is the term that describes nerve fibers that carry an impulse TOWARD the brain? *A* Afferent *B* Efferent *C* Peripheral *D* Dendritic

*A* Afferent Explanation - Afferent fibers carry impulses toward the brain and/or spinal cord. Efferent fibers carry them away from the brain and/or spinal cord. Peripheral nerves are outside of the brain and spinal cord. Dendritic is an adjective that describes a highly branched structure and refers to the projections from a nerve cell.

If a dog has atrophy of its muscles, what appearance will the muscles take on? *A* Appear sunken with loss of tone *B* Be irregular and lumpy on palpation *C* Appear larger than normal *D* Have calcification and be hard to the touch

*A* Appear sunken with loss of tone Explanation - Atrophy means to decrease in size. Hypertrophy is to enlarge in size

Which of the following should be a sterile procedure? *A* Arthrocentesis *B* Tooth extraction *C* Anal gland abscess culture *D* Fine needle aspirate of a mass *E* Lymph node aspirate

*A* Arthrocentesis Explanation - Arthrocentesis, or joint tap, should be performed with sterile technique. A joint tap is the act of inserting a needle into the joint space to withdraw synovial fluid for culture or analysis. The oral cavity is not sterile, and tooth extraction is not sterile. Lymph node and mass aspirates should be clean procedures but are not sterile. The same applies for culturing an abscess or site of infection

You need to perform a CBC and take thoracic radiographs on a somewhat fractious cat. You sedate the cat with an intramuscular injection of medetomidine. While on the X-ray table, you become concerned that the cat is not doing well, and the veterinarian asks you to reverse the effects of medetomidine. What should you give the cat? *A* Atipamezole *B* Yohimbine *C* Naloxone *D* Flumazenil *E* Atropine

*A* Atipamezole Explanation - Atipamezole (trade name: Antisedan), an alpha-2 antagonist, is the reversal agent for medetomidine (trade name: Dormitor). Medetomidine is an alpha-2 agonist. Xylazine would be another example of an alpha-2 agonist. Yohimbine is its reversal agent. Other alpha-2 agonists are clonidine, detomidine, dexmeditomidine and romifidine. Flumazenil is a reversal agent for benzodiazepines. Naloxone is used to reverse opioids. Atropine is a muscarinic antagonist of acetylcholine and may be dangerous to use after administration of alpha-2 agonists. This is because alpha-2 agonists cause marked vasoconstriction and high afterload on the heart. Giving atropine and increasing the heart rate can place further stress on the heart. You do not want to set in motion peripheral vasoconstriction and compensatory bradycardia brought on by the alpha-2, thereby increasing the heart rate against the high afterload

When performing abdominal surgery, which of the following instruments is frequently used retract the abdominal wall? *A* Balfour retractor *B* Finochietto retractor *C* Myerding retractor *D* Malleable retractor

*A* Balfour retractor Explanation - Balfour retractors are self-retaining retractors specifically designed to hold the abdomen open. Finochietto retractors are designed for thoracotomies to spread the ribs apart. A malleable retractor is a versatile retractor that can be bent in multiple directions to achieve the desired retraction. A Myerding retractor is typically used in orthopedics to hold muscle bellies out of the surgical field

A 2-year old bulldog presents with respiratory distress, specifically on inhalation, and stertor. What, of the following, would be the most common cause of respiratory distress in this patient? *A* Brachycephalic syndrome *B* Tracheal mass *C* Laryngeal paralysis *D* Tracheal foreign body *E* Asthma

*A* Brachycephalic syndrome Explanation - As a young bulldog, this patient would be suspected of having an elongated soft palate, stenotic nares, and (as a result of turbulent airflow), everted laryngeal saccules, or a combination of the three. Laryngeal paralysis is typically seen in large breed, older dogs. Asthma is nonexistent in dogs. A tracheal mass would be uncommon in such a young dog. While a tracheal foreign body is possible, brachycephalic syndrome is a better fit considering the breed and presence of stertor.

A 3-year old female Doberman presents for a routine spay. The owner states that she has not recently been in heat and has been healthy her entire life. The veterinarian is concerned that Doberman Pinchers are predisposed to having von Willebrand's Disease (vWD). Which of the following diagnostic tests would give the best indication of whether the dog has an increased risk of bleeding due to this disorder? *A* Buccal mucosal bleeding test *B* Abdominal ultrasound *C* Total protein level *D* Chest radiographs

*A* Buccal mucosal bleeding test Explanation - Doberman Pinschers are predisposed to having von Willebrand's Disease (vWD). Specifically, Dobermans have increased incidence of Type 1 vWD. In this type, there is a reduced presence of the functional von Willebrand factor. This factor is crucial in initial clot formation; it attracts platelets and allows them to bind to exposed subendothelium after injury. A buccal mucosal bleeding test should result in a clot in less than 4 minutes in normal dogs. An abnormal result warrants further investigation to confirm the presence of vWD in order to take the appropriate measures prior to surgery.

Which of the following is a local anesthetic of choice for use in veterinary dentistry because it can provide 3-5 hours of pain relief? *A* Bupivacaine *B* Lidocaine *C* Preservative-free morphine *D* Proparacaine

*A* Bupivacaine Explanation - 0.5% bupivacaine is a local anesthetic of choice because it provides a longer duration of effect that the other agents listed, frequently 3-5 hours. Some studies report that the duration may be even longer

Release of what molecule from the sarcoplasmic reticulum ultimately helps to achieve muscle contraction? *A* Calcium *B* Sodium *C* Adenosine triphosphate *D* Chloride

*A* Calcium Explanation - The correct answer is calcium, which is required for muscle contraction.

Platelets of which species tend to clump? *A* Cat *B* Dog *C* Horse *D* Parrot *E* Cow

*A* Cat Feline platelets (thrombocytes) clump easily. When counting platelets in cat blood, either manually or with an automated cell counter, this must be taken into account.

Which species commonly has laryngospasm when you are trying to place an endotracheal tube during anesthetic induction? *A* Cat *B* Cow *C* Dog *D* Horse

*A* Cat Explanation - Cats very commonly have laryngospasm, which can make it tricky to get the endotracheal tube placed. A drop of topical anesthetic can help to facilitate easier placement

Which part of the brain coordinates motor activity? Disease of this part of the brain can cause a hypermetric gait. *A* Cerebellum *B* Brainstem *C* Frontal lobe *D* Cerebral cortex

*A* Cerebellum Explanation - The cerebellum has a major role in motor control. It helps to coordinate voluntary movement and is located just above the brainstem. Hypermetria is an exaggerated gait in which the movements extend beyond what is intended.

Pyrethrin (which is used to kill fleas) is derived or extracted from which of the following plants? *A* Chrysanthemum *B* Hibiscus *C* Bougainvillea *D* Stargazer lily

*A* Chrysanthemum Explanation - Pyrethrin comes from the Chrysanthemum plant and acts on the nervous system to cause flea death

Organelles reside in which of the following compartments of the cell? *A* Cytoplasm *B* Nucleus *C* Ribosome *D* Cell membrane

*A* Cytoplasm Explanation - Organelles reside in the cytoplasm. Some examples include ribosomes, endoplasmic reticulum, lysosomes, mitochondria, and the Golgi complex. The nucleus contains the cell's DNA for replication. The cell membrane (cell wall) is what separates the cell from the external environment. The cell wall is semi-permeable and will allow certain compounds and electrolytes in and out of the cell.

A dirty wound has edges and tissue that need to be removed. What is the term for removing questionable tissue? *A* Debridement *B* Grafting *C* Skin scraping *D* Incising

*A* Debridement Explanation - Debriding is removing tissue or material from a wound to aid in healing. A skin scraping is done by taking a dull blade and mineral oil and scraping the skin in an attempt to find mites such as Demodex or Sarcoptes. A graft is a piece of tissue taken from one part of the body to use in another region of the body. Incising is the simple term for cutting or making an incision

Why is atropine or glycopyrrolate (anti-cholinergics) often given as a premedication to dogs and cats before inducing anesthesia with ketamine or tiletamine? (Dissociative anesthetics) *A* Decrease salivation *B* Increase airway secretions *C* Slows the heart rate *D* Helps keep eyes closed

*A* Decrease salivation Good Work, you picked the right answer! Anti-cholinergics like Atropine or Glycopyrrolate are given as pre-meds to DECREASE salivation, DECREASE airway secretions and INCREASE heart rate. They do NOT affect whether eyes are open or closed, but they do DILATE the pupils. Remember that many anesthetic drugs (like opiates, barbiturates, gas anesthetics) promote bradycardia (slowed heart rate) and some dissociative anesthetics (like ketamine, tiletamine) cause excessive salivation.

Where is the hypothalamus is located? *A* Diencephalon *B* Mid-brain *C* Brain stem *D* Mediastinum

*A* Diencephalon Explanation - The diencephalon is composed of the thalamus, hypothalamus, and pituitary gland. The brain stem is the posterior part of the brain adjoining the brain and spinal cord. Most of the cranial nerves originate in the brain stem. The brain stem includes the medulla oblongata, pons, and the midbrain. The thymus is located in the mediastinum. The mesencephalon is the mid-brain (part of the brain stem).

Which breed most commonly has von Willebrand's disease? *A* Doberman Pinscher *B* Labrador Retriever *C* Shar pei *D* English Bulldog

*A* Doberman Pinscher Explanation - Von Willebrand's disease is an inherited bleeding disorder most often seen in the Doberman. Other breeds can also have the disease, just not as commonly.

You are entering an exam room to obtain a history and perform a physical exam on a 2 year old German Shepherd. Which of the following body language signals would indicate that the dog is nervous or anxious? *A* Dog is standing but crouched down, ears pinned back, tail down, and the head lowered. *B* Standing at attention, making eye contact, hackles raised, and growling. *C* Dog is laying on his back, tail tucked to his abdomen, and ears flattened. *D* Dog is standing with ears erect, tail straight down, and looking around the room. *E* Dog is standing with ears erect and forward, tail straight down, and looking forward.

*A* Dog is standing but crouched down, ears pinned back, tail down, and the head lowered. Explanation - A dog that is crouched down, with the ears pinned back, head lowered, and the tail down is displaying nervous body language. A dog with erect ears, tail down, and looking around the room is calm. A dog holding his ears erect and forward with the tail down is alert. A dog laying on his back with his tail tucked and ears flat is submissive. A dog standing at attention with hackles raised and growling is showing aggression and may be protective of the owner

Which of the following correctly lists the species in order from shortest to longest gestation length? *A* Dog, Pig, Sheep, Horse, Llama *B* Ferret, Dog, Sheep, Pig, Horse *C* Cat, Ferret, Sheep, Cow, Horse *D* Pig, Sheep, Horse, Cow, Llama

*A* Dog, Pig, Sheep, Horse, Llama Explanation - This list of gestations should be committed to memory: Llama 1 year (350 days), Horse 11 months (330 days), Cow 9 months (280 days), Sheep/Goat 5 months (150 days), Pig 4 months (114 days), Dog/Cat 2 months (63 days), Ferret 1.5 months (42 days)

A pet that is exhibiting the symptom of ptyalism has which clinical sign? *A* Drooling *B* A penis that is stuck protruding from the prepuce *C* A jaw locked shut *D* A head that is hung downward

*A* Drooling Explanation - Ptyalism is drooling or hypersalivation. The term for a penis that is stuck protruding from the prepuce is paraphimosis

A patient presents with red eyes and the owner reports he has been rubbing his face on the carpet. He is currently taking the sulfa-based drug Primor for a skin infection. What side effect of this medication could cause these symptoms? *A* Dry Eye *B* Uveitis *C* Dendritic ulcer formation *D* Glaucoma *E* Epiphora

*A* Dry Eye Explanation - Tears are made up of 3 components, lipid, mucous, and liquid. The liquid and mucous portion of the tear film may decrease from sulfa drugs, thus resulting in dry eye (kerratoconjunctivitis sicca/KCS). Sometimes this problem can be irreversible. This patient should receive a Schirmer Tear Test to test for KCS. The eyes should also be stained, because corneal ulceration can occur secondary to the low tear production. Glaucoma is increased intraocular pressure. Uveitis is inflammation in the uveal tract inside the eye. Epiphora is excessive tearing, which is the opposite of KCS. Dendritic ulcers are most typically seen in cats with herpesvirus infection

Which one of the following choices is the chief advantage of using ethylenediamine tetra-acetic acid (EDTA) for complete blood count (CBC) examinations? *A* EDTA doesn't interfere with blood cell morphology *B* EDTA stains cells and allows for easier examination *C* EDTA allows thrombocytes to aggregate, aiding cell counts *D* EDTA is one of the rare anticoagulants that works on lipemic blood *E* EDTA allows differentiation between white blood cell types

*A* EDTA doesn't interfere with blood cell morphology Ethylenediamine tetra-acetic acid (EDTA) is the anticoagulant of choice for a complete blood cell count (CBC) is used because it doesn't stain the cells and doesn't interfere with blood cell morphology. EDTA generally prevents thrombocyte (platelet) aggregation, though there are occasional reports of EDTA-induced platelet clumping , especially in horses.

What is cerumen? *A* Ear wax *B* Oil produced by sebaceous glands *C* A mucoid substance secreted by the stomach *D* The oily part of the tear film

*A* Ear wax Explanation - Cerumen is the proper term for ear wax. It is the yellowish waxy hydrophobic protective substance that is secreted in the ear canal. Sebum is an oily substance that is produced by the sebaceous glands.

Which of these therapies would prevent weight-bearing of a limb? *A* Ehmer sling *B* Schroeder-Thomas splint *C* Mason-Meta splint *D* Fiberglass cast

*A* Ehmer sling Explanation - Two common examples of slings which prevent weight-bearing are: 1) Ehmer sling- used for coxofemoral luxations (dislocated hips) to hold hip in place 2) Velpeau Sling- used for shoulder luxation or after shoulder surgery Splints provide rigid support and do not prevent weight bearing on the limb. They do not go around the entire limb. Casts provide support and go all the way around the limb

Ruminants frequently belch or burp to release gas from the forestomach. What is the term for this process known as? *A* Eructation *B* Apnea *C* Fermentation *D* Casting

*A* Eructation Explanation - Eructation is the process of belching or burping.

What does the abbreviation "q" stand for on a prescription? *A* Every *B* Orally *C* Give *D* Discontinue

*A* Every Explanation - "q" means every. It may say something such as: Give 1 tablet q 12 hours for 14 days. This would mean give 1 tablet every 12 hours for 14 days. The abbreviation for orally is "P.O." (per os).

What term describes an enlarged thyroid gland? *A* Goiter *B* Thyroiditis *C* Thyroidosis *D* Hypothyroidism

*A* Goiter Explanation - Goiter is the abnormal gross enlargement of the thyroid gland. Thyroiditis is inflammation of the thyroid tissue. Thyroidosis is not a medical term. Hypothyroidism is inadequate production of thyroid hormone.

Which is a "ball and socket" type of joint? *A* Hip *B* Atlantoaxial *C* Elbow *D* Stifle

*A* Hip Explanation - The hip joint (acetabulofemoral joint) is the ball of the head of the femur sitting in the socket (acetabulum) of the pelvis. The atlantoaxial joint is a pivot joint connecting the first and second cervical vertebrae. The stifle and elbow are hinge joints.

What is a male ferret called? *A* Hob *B* Felicia *C* Jake *D* Boomer *E* Buck

*A* Hob Male ferrets are called "Hobs" and females are called "Jills" and a group of ferrets is called a "business". In 1971, a female ferret named "Felicia" did in fact help clean pipes at Fermi National physics labs by pulling a string attached to a cotton swab behind her.

Which of the following species have hypsodont teeth? *A* Horses *B* Monkeys *C* Dogs *D* Pigs

*A* Horses Explanation - Hypsodont, or high-crowned teeth, continue to erupt throughout life. All of the permanent teeth of horses and cheek teeth of ruminants are this type, with the roots embedded in the alveolus of the jaw bone. Dogs, cats, monkeys, and pigs have brachydont, or low-crowned teeth in which the root is embedded in the jaw bone

Urine from which of the following species is normally cloudy or milky in appearance? *A* Horses and rabbits *B* Cats and cattle *C* Sheep and goats *D* Dogs and ferrets

*A* Horses and rabbits Explanation - Urine from horses and rabbits contain high concentrations of calcium carbonate crystals that can make the urine appear milky. Additionally, horses normally have mucous from glands in the renal pelvis that contribute to the urine, which can make it appear cloudy

An anthelmintic is used for treating which of the following? *A* Intestinal parasites *B* Mange *C* Ticks *D* Fleas

*A* Intestinal parasites Explanation - Anthelmintics are drugs used for expulsion of intestinal worms. Ivermectin, praziquantel, fenbendazole, pyrantel, etc. are a few examples of anthelmintics

You have a small canine patient in cardiac arrest that needs emergency drugs. The animal does not have venous access, and placing a venous catheter would be too difficult and take too long. What is the best way to get the drugs to this animal quickly? *A* Intraosseous catheter *B* Subcutaneous injection *C* Intracardiac injection *D* Intramuscular injection

*A* Intraosseous catheter Explanation - An intraosseous catheter or spinal needle can easily be placed in the trochanteric fossa of the femur for vascular access. The marrow has a similar rate of absorption as venous catheters allowing for quick delivery of drugs and fluids. Intracardiac injection presents high potential hazards such as inducing fibrillation and pneumothorax. Subcutaneous injections are very slowly absorbed and would not be a good option for this patient. Intramuscular injection would not be effective because of the length of time the drugs would require to reach circulation

Which toxicity is not a common concern in cattle? *A* Iron toxicity *B* Salt toxicity *C* Lead toxicity *D Aspergillus toxicosis

*A* Iron toxicity Explanation - Iron toxicity is most common in newborn pigs. Aspergillus toxicosis is caused by moldy feed. Salt toxicity is caused by consumption of excessive salt and lack of adequate amounts of water. Lead toxicity is associated with seeding and harvesting activities when used oil and battery disposal from machinery is handled improperly

An 8-year old female spayed Dalmatian presents to your clinic for dribbling urine when sleeping. Bloodwork and urinalysis are performed, and phenylpropanolamine is prescribed. How will this drug help treat the dog's problem? *A* It increases the tone of the urethral sphincter *B* It increases re-absorption of water by the kidneys, resulting in decreased urine volume *C* It kills Gram-negative bacteria *D* It alkalinizes the urine, decreasing urinary crystal formation *E* It provides hormonal replacement to reverse the condition *F* It acidifies the urine, decreasing urinary crystal formation

*A* It increases the tone of the urethral sphincter Explanation - Phenylpropanolamine is an alpha-adrenergic agonist and causes tightening of the urethral sphincter. It is used to treat "sphincter mechanism incontinence". This is a non-hormonal treatment frequently used to treat incontinence. Older dogs, particularly female spayed dogs, commonly develop urinary incontinence due to weakness of the urethral sphincter. In some cases, this may be "hormone-responsive" meaning that hormone replacement (estrogen-based in females) may reverse the condition. Urine acidifying or alkalinizing agents could be useful in treating animals prone to formation of urinary calculi. Diethylstilbestrol (DES) is the most common estrogen replacement drug used in dogs to treat incontinence

An arthrogram involves contrast administration into what region? *A* Joint *B* Esophagus *C* Epidural space *D* Subdural space *E* Bladder *F* Subarachnoid space

*A* Joint Explanation - An arthrogram involves contrast administration into a joint. A myelogram involves contrast administration into the subarachnoid space. Contrast is injected into the bladder for a cystogram and ingested into the esophagus for an esophagram

An 8-year old cat suffering from intestinal lymphoma has been prescribed TPN. Which of the following is necessary for the administration of TPN? *A* Jugular catheter *B* A-line *C* Subcutaneous catheter *D* Cephalic catheter *E* Esophagostomy tube

*A* Jugular catheter Explanation - TPN stands for total parenteral nutrition. TPN solutions contain 10% (or greater) dextrose and must be given in a central catheter to avoid phlebitis. TPN solutions cannot be administered through peripheral catheters. Medications should never be injected into A-lines

Struvite crystals are one of the most common observed in urine. What are the chemical constituents of struvite? *A* Magnesium, ammonium, and phosphate *B* Ammonium, silica, and carbon *C* Calcium and phosphate *D* Magnesium, bilirubin, and phosphate

*A* Magnesium, ammonium, and phosphate Explanation - Struvite crystals are made up of magnesium, ammonium, and phosphate

Which medication may help in a case of acute glaucoma? *A* Mannitol *B* Chloramphenicol *C* Atropine *D* Ketamine

*A* Mannitol Explanation - Mannitol is an osmotic diuretic that will help to decrease intraocular pressure. Atropine raises intraocular pressure and would be contraindicated. Chloramphenicol is an antibiotic, and ketamine is an NMDA receptor antagonist and dissociative anesthetic. Neither of these are indicated in glaucoma

Which organism causes pinkeye in cattle? *A* Moraxella bovis *B* Dermatophilus congolensis *C* Arcanobacterium pyogenes *D* Bovine herpesvirus

*A* Moraxella bovis Explanation - Moraxella bovis is the bacterium that causes bovine keratoconjunctivitis (pinkeye) in cattle. It is transmitted from one bovid to another via flies, fomites, or direct contact. There are many different types of bovine herpesvirus that can result in different types of presentations. BHV-1 also known as infectious bovine rhinotracheitis virus can occur concurrently with moraxella bovis, complicating the disease. Arcanobacterium pyogenes can cause wound infections and mastitis. Dermatophilus congolensis contributes to "rain scald"

What is the proper terminology to describe a laceration under the right forelimb paw? *A* Palmar aspect *B* Plantar aspect *C* Anterior aspect *D* Posterior aspect

*A* Palmar aspect Explanation - Palmar is the bottom of the paw of the forelimb. Plantar is the bottom of the paw of the hind limb. Anterior is toward the head and posterior is away from the head. In veterinary medicine, anterior and posterior are usually used only to describe distal extremities or areas of the head. Cranial is toward the head; caudal is toward the tail. Ventral is toward the abdomen; dorsal is toward the backbone. Lateral is away from midline and medial is toward midline. Proximal is close to the spine or body while distal is away; these are typically used when describing limbs (e.g. fracture of the proximal femur vs. fracture of the distal femur).

A client has been giving her pet a "nutraceutical" she found that is supposed to help with her dog's arthritis. What are nutraceuticals? *A* Phytochemicals *B* Minerals *C* Herbs *D* Vitamins

*A* Phytochemicals Explanation - Nutraceuticals are chemicals or foodstuffs derived from plants in the form of a supplement. These are not regulated by the government

A blood smear is performed and a large variation is seen in erythrocyte shape. What is this called? *A* Poikilocytosis *B* Polychromasia *C* Anisokaryosis *D* Anisocytosis

*A* Poikilocytosis Explanation - Anisocytosis is a variation in cell size (not shape). Anisokaryosis is a variation in the amount of cytoplasm present in a cell and no necessarily shape or size. Polychromasia describes a variation red blood cell color. Poikilocytosis describes a variation in cell shape. Some shapes occur with enough frequency that they have their own name, such as acanthocytes and echinocytes

Which of the following species require Vitamin C in their diets because they cannot synthesize it? *A* Primate *B* Chinchilla *C* Hamster *D* Cat

*A* Primate Explanation - Primates, including humans, are unable to synthesize adequate amounts of Vitamin C and must have it included in the daily diet. Guinea Pigs too have the same requirement for additional daily Vitamin C.

What is the name for the muscle group that includes the rectus femoris, vastus lateralis, vastus medialis, and vastus intermedius? *A* Quadriceps *B* Gluteals *C* Hamstrings *D* Triceps

*A* Quadriceps Explanation - "Quad" means four, and the Quadriceps femoris is a large group of muscles that cover most of the middle thigh. This muscle group extends the knee. The rectus femoris originates on the ilium and also flexes the hip. The other three muscles originate from the femur. All four parts attach to the patella through the quadriceps tendon. The hamstrings are the biceps femoris, semitendinosus, and semimembranosus muscles.

In what order does an electrical impulse travel through the heart? *A* SA node, AV node, Bundle of His, Purkinje fibers *B* AV node, SA node, Bundle of His, Purkinje fibers *C* SA node, Bundle of His, AV node, Purkinje fibers *D* Bundle of His, SA node, AV node, Purkinje fibers *E* SA node, Purkinje fibers, AV node, Bundle of His

*A* SA node, AV node, Bundle of His, Purkinje fibers Explanation - The sinoatrial node (SA node) is the pacemaker of the heart located in the wall of the right atrium. It generates an electrical impulse that is carried to the atrioventricular node (AV node) which is located between the atria and ventricles. The impulse is then relayed down the conducting tissues, or the Bundle of His, and branches to the paths that supply the right and left ventricles (the right and left bundle branches). Purkinje fibers are reached last and are conductive fibers within the walls of the ventricles that relay the impulses to the cells that cause contraction of the ventricles. This process of impulse travel is known as depolarization.

The pulse oximeter measures what variable? *A* Saturation of hemoglobin with oxygen *B* Saturation of plasma with oxygen *C* artial pressure of oxygen in arterial blood *D* Partial pressure of carbon dioxide in arterial blood *E* Saturation of hemoglobin with carbon dioxide

*A* Saturation of hemoglobin with oxygen Explanation - Pulse oximetry detects the amount of oxygen present on hemoglobin and is expressed as a percentage (i.e. 97% saturation). This monitor measures saturation by using the different wave lengths of saturated and desaturated hemoglobin. This is different than the amount of pressure that oxygen exerts on a vessel (i.e. the partial pressure of oxygen).

A foal has petechial hemorrhages on its pinna, injected sclera, and brick red mucous membranes. These are signs of which of the following? *A* Sepsis *B* Failure of passive transfer *C* Cryptosporidiosis *D* Milk allergy

*A* Sepsis Explanation - Depression, cold limbs, aural petechiations, injected tongue and mucous membranes, hypoglycemia, increased respiration, decreased suckling of the mare, hypothermia, and recumbency are all symptoms of sepsis in the foal. Failure of passive transfer predisposes the foal to sepsis

Activated charcoal would be used for which of the following? *A* Snail bait ingestion *B* Fluoroscopy *C* Thermal skin burns *D* Foreign body ingestion

*A* Snail bait ingestion Explanation - Activated charcoal is a black liquid administered orally to prevent absorption of toxins through the gastrointestinal tract. It is often used after induction of emesis or after gastric lavage is performed

What does the term "metastasis" used in veterinary medicine most often describe? *A* Spread of cancer from the primary site to another site *B* A malignant tumor that is locally aggressive *C* Degeneration of a tumor without treatment *D* A non-cancerous lesion

*A* Spread of cancer from the primary site to another site Explanation - A malignant tumor is synonymous with cancer. Malignant tumors may metastasize, or spread to other sites. Very common areas to which tumors metastasize are the lymph nodes and lungs. This is why lymph node aspirations and chest radiographs are often done as part of cancer staging

How do oral flea control products such as Program or Sentinel work? *A* Stopping egg hatching *B* Paralyzing the nervous system of the flea *C* Killing adult fleas directly *D* Sterilizing the male flea to prevent reproduction

*A* Stopping egg hatching Explanation - These products interfere with chitin synthesis. The flea's "egg tooth", which is made of chitin, is no longer able to help the flea hatch from the egg. Another possible correct choice, but not provided, would be that females produce sterile eggs. Female fleas may produce 150-300 eggs per week (or up to 50 eggs a day). These products do not kill adult fleas

The interns at the hospital where you work are required to "SOAP" their patients daily. What does this stand for? *A* Subjective, Objective, Assessment, Plan *B* Subjective, Outline, Action, Plan *C* Symptoms, Outline, Assessment, Plan *D* Symptoms, Outline, Assessment, Prognosis

*A* Subjective, Objective, Assessment, Plan Explanation - SOAP stands for Subjective, Objective, Assessment, Plan. Subjective includes the history; Objective includes the physical exam and other information gathered; Assessment includes the problem list and rule outs; and Plan includes the next steps in the evaluation/plan

What are the "Joints" in the skull are referred to as? *A* Sutures *B* Planes *C* Hinge *D* Ball and socket

*A* Sutures Explanation - Sutures are fibrous bands of tissue that connect the bones of the skull. In neonates, the sutures are open and allow for growth of the brain and skull

What is collected when performing an arthrocentesis? *A* Synovial fluid *B* Urine *C* Bone marrow *D* Blood

*A* Synovial fluid Explanation - Arthrocentesis is inserting a needle into the joint space to collect synovial fluid (joint fluid). This procedure is performed by a veterinarian

The "hock joint" of a horse is more appropriately identified as what structure? *A* Tarsocrural joint *B* Metatarsophalangeal joint *C* Tarsometatarsal joint *D* Distal interphalangeal joint

*A* Tarsocrural joint Explanation - The hock joint, more specifically called the tarsocrurual joint in the horse, is a high-motion joint composed of numerous bones.

Why are polyurethane catheters more appropriate for long-term (> 7 days) use in horses? *A* They are the least thrombogenic *B* They are the largest *C* They are rigid *D* They do not require maintenance

*A* They are the least thrombogenic Explanation - Polyurethane catheters (such as those made by Mila) are the least reactive and thrombogenic and, if placed correctly, can be left in place for more than a week. Polyurethane is a flexible material. All IV catheters require maintenance and monitoring

Which bone is not associated with the forelimb? *A* Tibia *B* Radius *C* Ulna *D* Humerus *E* Metacarpal

*A* Tibia Explanation - The tibia is a bone of the hindlimb also known as the "shinbone". The tibia is located distal to the femur and proximal to the tarsus

Pre-anesthetic drugs are used for all of the following reasons except which of the following? *A* To decrease the blood pressure *B* Smoother anesthetic induction and recovery period *C* Less induction drugs are needed *D* Provide analgesia *E* Reduce anxiety

*A* To decrease the blood pressure Explanation - Decreasing blood pressure is not a goal of a pre-anesthetic drug. Pre-meds help to reduce anxiety in the patient which allows for a smoother induction with fewer drugs and a smoother post-operative recovery. Many pre-anesthetic agents also provide analgesic effects

Which small animal disease has been implicated as a specific danger to pregnant women? *A* Toxoplasmosis *B* Coccidiosis *C* Camylobacter *D* Clostridium

*A* Toxoplasmosis Explanation - The Toxoplasmosis tachyzoites can migrate transplacentally and harm the fetus. Transmission can occur by eating undercooked meat or by inadvertently ingesting oocytes from cat feces. Therefore, it is advised that pregnant women not clean litter boxes, or if they must, wear personal protective equipment while doing so.

What is the most common blood type in cats in the United States? *A* Type A *B* Type O *C* Type AB *D* Type B

*A* Type A Explanation - Cats most commonly have type A blood. Some cats have type B blood, many of which are exotic-type species. Type B cats have strong anti-A alloantibodies, so type A blood given to a type B cat results in life-threatening acute hemolytic transfusion reactions. A cat should always have a blood type test before a transfusion

Which type of urinary stone is lucent and does not typically show up on a radiograph? *A* Urate *B* Struvite *C* Magnesium ammonium phosphate *D* Calcium oxalate

*A* Urate Explanation - Urate and Cystine are the two types of stones that do not usually show up on radiographs and require ultrasound or contrast studies to diagnose. Calcium oxalate and struvite stones are radiodense and usually visible on a radiograph. Magnesium ammonium phosphate is another name for a struvite stone

For which of the following disorders is an intravenous pyelogram most useful in detecting? *A* Ureteral stones *B* Lymphangiectasia *C* Bile duct obstruction *D* Gall bladder stones *E* Urethral stones

*A* Ureteral stones Explanation - An intravenous pyelogram (IVP) is an x-ray examination of the kidneys, ureters and urinary bladder using iodinated contrast material injected intravenously and excreted via the kidneys. It is used to detect problems in the urinary tract (particularly the upper urinary tract) including kidney and ureteral stones or tumors involving the kidney, ureters, or urinary bladder. An IVP is not as useful for urethral stones because they are further down in the urinary tract; in this case, retrograde cystography would be more helpful

What is the most accurate method to estimate a horse's weight out on a farm call? *A* Use a weight measuring tape *B* Pick up a foot and use that weight to approximate the weight *C* Approximate the weight based on body condition score *D* Approximate the weight by height

*A* Use a weight measuring tape Explanation - Specialized measuring tapes are commonly used to approximate the weight of a horse. There are markings that show weights and the tape is wrapped around the barrel of the horse just caudal to the withers

The broad ligament attaches which structure to the body? *A* Uterus *B* Bladder *C* Spleen *D* Intestines

*A* Uterus Explanation - The broad ligament is attaches the uterus to the pelvis

Which of the following drugs is NOT approved for the use in cattle? *A* Xylazine *B* Albendazole *C* Diazepam *D* Butorphanol

*A* Xylazine Explanation - Ruminants are highly sensitive to xylazine when compared with horses, dogs, and cats. They generally require only one-tenth of the dosage required for horses to exhibit the same effect. It is not approved for the use in ruminants but is sometimes used off-label. It is important to know how sensitive ruminants are to this drug. Albendazole is a broad spectrum de-wormer commonly used in cattle. Diazepam or Butorphanol can be used as a sedative or tranquilizer in cattle

Which of the following drugs should be used with extreme caution in ruminants? *A* Xylazine *B* Atropine *C* Acepromazine *D* Lidocaine

*A* Xylazine Explanation - Xylazine is an alpha-2 adrenergic agonist and is classified as a sedative/analgesic with muscle relaxant properties. It is not approved for any species to be consumed for food purposes. Ruminants are very sensitive to xylazine and it is not used much in these animals. In fact, ruminants generally require only 1/10 of the dosage required for horses to exhibit the same effect. Due to the unique GI system of the ruminant, much care must be taken when administering drugs so as not to interfere with proper digestion, and xylazine inreases risk of ileus

A 285-gram guinea pig weighs: *A* 2.8 pounds *B* 0.285 kg *C* 1.2 pounds *D* 2.85 kg

*B* 0.285 kg Explanation - 1 kilogram equals 1,000 g 285 g/ 1,000 g/kg = 0.285 kg If you want to convert into pounds, multiply kg X 2.2 pounds/kg. 0.285 kg x 2.2 pounds/kg = 0.627 pounds

A dog having a rough recovery from anesthesia has been ordered to receive 0.02 mg/kg of acepromazine. The dog's body weight is 40 lbs, and the acepromazine is diluted to a 1 mg/ml solution. How much acepromazine should this patient be administered? *A* 0.036 ml *B* 0.36 ml *C* 0.8 ml *D* 1.2 ml

*B* 0.36 ml Explanation - First, convert the body weight to kilograms. 40 lbs / (2.2 lbs/kg) = 18.2 kg Now determine the amount of milligrams needed. 0.02 mg/kg x (18.2 kg) = 0.36 mg (note the kilograms cancel out) Finally determine the amount of milliliters needed. 0.36 mg / (1 mg/ml) = 0.36 ml (note the milligrams cancel out)

A patient's fluid rate is currently set at 150 ml/hr. The doctor asks you to decrease the fluid rate by 25%. What is the new rate? *A* 37.5 ml *B* 112.5 ml *C* 90 ml *D* 87.5 ml

*B* 112.5 ml Explanation - First, we need to determine how much 25% of 150ml is. 150 ml x 0.25 = 37.5 ml Now, subtract the answer above from the original fluid rate to determine the new rate. 150 ml - 37.5 ml = 112.5 ml

How many thoracic vertebrae do horses have? *A* 12 *B* 18 *C* 13 *D* 9

*B* 18 Explanation - Horses have 18 thoracic vertebrae. Dogs, cats , cows, and sheep have 13 thoracic vertebrae. Pigs have 14-15 thoracic vertebrae. Humans have 12 thoracic vertebrae

Approximately what percentage of body weight should a horse consume in forage (i.e. hay) per day? *A* 12% *B* 2% *C* 8% *D* 5%

*B* 2% Explanation - The average horse should consume approximately 2% of its body weight in forage. In a 1000 lb horse, this would equal 20 lbs of hay/day.

What is the normal respiratory rate of a cat? *A* 40-76 breaths per minute *B* 24-42 breaths per minute *C* 12-18 breaths per minute *D* 6-14 breaths per minute

*B* 24-42 breaths per minute Explanation - Cats have a resting respiratory rate of approximately 24-42 breaths per minute. Dogs have a resting respiratory rate of approximately 10-30 breaths per minute. Horses have a respiratory rate of approximately 8-16 breaths per minute.

Frequently, the doctor will ask that a prescription for Cephalexin be filled. How many 500 mg capsules should be sent home for a dog receiving 22 mg/kg PO TID for 10 days? The dog weighs 50 lbs. *A* 45 *B* 30 *C* 15 *D* 60

*B* 30 Explanation - To get the answer, the body weight must first be converted to kilograms. 50 lbs / 2.2 kg/lb = 22.7 kg (There are 2.2 lbs per kilogram) Then, 22.7 kg x (22mg/kg) = 499.4 mg This is rounded up to 500 mg. The patient takes 3 capsules per day for 10 days. 3 x 10 = 30 The patient should be sent home with 30 capsules

Which of the following is not an anticipated finding when evaluating a urine sample after performing a cytocentesis? *A* Calcium oxalate crystals *B* 5-10 white blood cells per high-powered field *C* Erythrocytes *D* Struvite crystals

*B* 5-10 white blood cells per high-powered field Explanation - There should be less than 0-5 white blood cells per high-powered field in a urine sample. Erythrocytes are not uncommon when performing a cystocentesis. This can occur secondary to trauma to the bladder wall when performing urine collection. Keep in mind that an excessive presence of blood can be abnormal. Struvite and calcium oxalate crystaluria is not necessarily abnormal

What are red blood cells (RBCs) with multiple irregularly spaced projections called? *A* Metarubricytes *B* Acanthocytes *C* Rouleaux *D* Cornified *E* Spherocytes

*B* Acanthocytes When some red blood cells (RBCs) have a membrane abnormality that causes numerous, irregularly spaced projections, these cells are called acanthocytes.

Which of these is an effective drug to induce emesis in the dog? *A* Azathioprine *B* Apomorphine *C* Magnesium hydroxide *D* Xylazine

*B* Apomorphine Explanation - The correct answer is apomorphine. Apomorphine is an opioid dopaminergic agonist that acts on the chemoreceptor trigger zone to induce vomiting in dogs. It can be administered IM, SC, IV, or in the conjunctival sac. Xylazine is a fairly effective emetic in the cat but is not used in the dog for this purpose. Azathioprine is an immunosuppressive drug not used to induce vomiting. Magnesium hydroxide or Milk of Magnesia is a cathartic but is not used for vomiting

Which of the following foods is considered toxic to parrots? *A* Chicken *B* Avocado *C* Blueberries *D* Cheese *E* Habanero peppers

*B* Avocado Explanation - A toxin found in the Guatemalan type of avocado causes myocardial necrosis in parrots. Blueberries and peppers (even "hot" peppers) are not toxic to psittacines. Chicken and cheese are high in fats and are not appropriate to feed parrots, but there is no toxicity associated with them

What is the term that describes an animal with elevated blood urea nitrogen and creatinine levels measured in the blood? *A* Nephrotic *B* Azotemic *C* Hyperemic *D* Uremic

*B* Azotemic Explanation - Azotemia is the term meaning an animal has elevated blood urea nitrogen and creatinine. Uremia indicates a clinical syndrome seen in animals with severe kidney disease where they develop nausea, inappetence and potentially other signs such as oral ulcers. Hyperemia refers to red-colored mucous membranes. Nephrotic syndrome refers to a series of changes found in animals with ongoing or severe proteinuria

Which of the following appropriately describes a surgical preparation of an incision site? *A* Scrub from cranial to caudal starting at the hairline *B* Begin where the incision will be made and scrub in outward circles, not returning to the central area *C* Alternate chlorhexidine and iodine scrubs for a total of 6 scrubs *D* Scrub and rinse the area a minimum of 6 times

*B* Begin where the incision will be made and scrub in outward circles, not returning to the central area Explanation - Either chlorhexidine or betadine can be used, but not both together. Scrubbing is started near the center of the incision site in a circular scrubbing motion, moving from the center to the periphery without returning from the periphery to the center. Frequently, the site is scrubbed alternately with alcohol or saline although the benefit of this practice is controversial

Which test would provide the best measure of liver function? *A* Serum BUN level *B* Bile acids *C* Ultrasound *D* Serum phosphorus level

*B* Bile acids Explanation - When a meal is consumed, the gall bladder contracts and releases bile into the upper small intestine as needed for digestion. The bile acids break down lipids during this process of digestion. The bile acids are then absorbed by the intestine and into the portal bloodstream and returned back to the liver. If the liver is functioning well, the bile acids are removed from the bloodstream and returned to the gall bladder until they are needed again. Comparing the two blood levels (pre- and post-prandial bile acids) allows the veterinarian to see how well the liver is functioning. Bile acids are removed from portal blood by the hepatocytes (liver cells). If the liver cells are not working as they should, the bile acids remain in circulation and enter the systemic circulation where the elevated levels are measured by the bile acids test. Ultrasound is good for looking at architecture of the liver, but does not evaluate function of the hepatocytes. BUN and phosphorus are most often looked at when evaluating kidney disease

Uric acid is the major end product of nitrogen metabolism in which of the following animals? *A* Horses *B* Birds *C* Cats *D* Goats *E* Ferrets

*B* Birds Explanation - Birds metabolize most of their nitrogen to uric acid, whereas mammals metabolize most of their nitrogen to urea.

Which of the following is NOT true regarding platelets? *A* Platelet clumping can cause falsely low platelet counts *B* Birds and reptiles have nucleated platelets *C* They are derived from megakaryocytes *D* A purple top tube with EDTA should be used when collecting blood for a platelet count

*B* Birds and reptiles have nucleated platelets Explanation - Birds and reptiles have nucleated red blood cells, not nucleated platelets. A purple top is used for platelets counts. Clumping of platelets can cause a low reading, so the periphery of the slide should be checked for clumping. In cases where clumping is present, a platelet estimate may be more valuable. The megakaryocyte is a bone marrow cell responsible for the production of blood thrombocytes (platelets).

What is a comedone? *A* Sebaceous wart *B* Blackhead *C* Fungus *D* Hyperkeratinized skin

*B* Blackhead Explanation - A comedone is another term for blackhead. Comedones are fairly common in dogs with seborrhea (excess production of sebum), endocrinopathies, and demodex

What tissue is the MOST sensitive to radiation therapy? *A* Cartilage *B* Bone marrow *C* Soft Tissue *D* Muscles

*B* Bone marrow Explanation - Bone marrow consists of progenitor and stem cells, the most radiosensitive cells. Cells going through division are generally the most sensitive

For which of the following diseases should an animal be isolated? *A* Demodectic mange *B* Bordetella bronchiseptica *C* Ehrlichia canis *D* Diabetes

*B* Bordetella bronchiseptica Explanation - Bordetella is one cause of kennel cough and is contagious to other dogs. The other diseases listed are not contagious. Diabetes is not a contagious condition as is a result of an endocrinopathy. Demodectic manage is secondary to a non-contagious mite. Ehrlichia canis is a tick-borne disease and is not directly transmissible

Which of the following diseases should ferrets be vaccinated against? *A* Feline panleukopenia virus *B* Canine distemper virus *C* Infectious canine hepatitis virus *D* Canine parvovirus

*B* Canine distemper virus Explanation - Ferrets should be vaccinated against canine distemper virus. Ferrets are not susceptible to the other diseases listed

What are the components of a typical X-ray cassette in order from front (where X-rays enter) to back? *A* Cassette front, intensifying screen, padding, film, padding, intensifying screen, cassette back *B* Cassette front, padding, intensifying screen, film, intensifying screen, padding, cassette back *C* Cassette front, padding, intensifying screen, film, padding, cassette back *D* Cassette front, padding, film intensifying screen, film, padding, cassette back

*B* Cassette front, padding, intensifying screen, film, intensifying screen, padding, cassette back Explanation - The intensifying screen must be next to the film and not shielded by padding in order to expose the film. Most cassettes have intensifying screens and padding on each side of the film

Which part of the brain is associated with regulation and coordination of movement, posture, and balance? *A* Hypothalamus *B* Cerebellum *C* Brain stem *D* Cerebral cortex

*B* Cerebellum Explanation - The cerebellum is involved in the coordination of voluntary motor, balance and equilibrium. It is located just above the brain stem

A rabbit is brought in for skin problems. Upon exam, you notice large flakes of dead skin over the dorsum. Some of these flakes appear to move. What might you be seeing? *A* Notoedres *B* Cheyletiella *C* Demodex *D* Sarcoptes *E* Psoroptes *F* Otodectes

*B* Cheyletiella Explanation - Cheyletiella is also known as "walking dandruff". It is a 8 legged mite that can live on the skin of rabbits, dogs, cats and humans. They do not bury in the skin but live in the keratin layer. They have a 21 day life cycle and cannot live more than 10 days off the host.

A dog has ruptured a chordae tendinae. What condition can this lead to? *A* Chronic osteoarthritis and lameness *B* Congestive heart failure *C* Incontinence *D* Paralyzed diaphragm muscle

*B* Congestive heart failure Explanation - The chordae tendinae are cord-like tendons that connect the papillary muscles to the tricuspid valve and the mitral valve in the heart. If this structure ruptures, the valves can no longer function properly, and this can lead to congestive heart failure

When discussing the equine hoof, which structure produces cells that become the hoof wall? *A* Frog *B* Coronary band *C* Ergot *D* Sole

*B* Coronary band Explanation - The hoof wall grows from the coronary band. The coronary corium produces cells that become the horn of the hoof wall. It is soft skin tissue such as the cuticles on our fingers. If the coronary band is ever damaged, the hoof may grow improperly and could cause lameness. The ergot is a callus-type structure on the underside of the fetlock. The sole is the bottom of the hoof. The frog is the structure that is shaped like a heart and extends forward across two-thirds of the sole. It grows from front to back where it then merges with the heel periople.

What is most often seen in a cat with heartworm disease? *A* Unkempt haircoat *B* Coughing *C* Petechial hemorrhages *D* Seizuring *E* Syncope

*B* Coughing Explanation - Most cats with heartworm are not clinical for the disease. If they do show symptoms, they are typically respiratory in nature (coughing, wheezing). Chronic vomiting can also be a sign. All coughing cats should be tested for heartworm

Which of the following is an ectoparasite of animals? *A* Ancylostoma caninum *B* Ctenocephalides felis *C* Filaroides osleri *D* Feline infectious peritonitis *E* Paragonimus kellicotti

*B* Ctenocephalides felis Explanation - By definition, an ectoparasite is a eukaryotic organism that lives on the outside of the host. Examples include lice, flies, fleas, and mites. Ctenocephalides is a flea. Ancylostoma, Filaroides, and Paragonimus are all endoparasites. Feline infectious peritonitis is not a parasite, it has a viral cause

Pre-renal azotemia is most commonly caused by which of the following? *A* Acute renal failure *B* Dehydration *C* Urethral obstruction *D* Pyelonephritis *D* Chronic renal failure

*B* Dehydration Explanation - Dehydration is the most common cause of pre-renal azotemia. Azotemia is the term for elevated renal values (BUN and creatinine). When a patient is dehydrated, renal values may be elevated. When re-hydration occurs, renal values should return to normal. Renal failure (acute or chronic) causes primary renal azotemia. Post-renal azotemia is most often caused by obstruction either in the bladder or urethra, which prevents urine from being voided.

Which of the following would not be an underlying cause for a urine specific gravity of less than 1.020? *A* Renal failure *B* Dehydration *C* Diabetes mellitus *D* Hypoadrenocorticism

*B* Dehydration Explanation - Dehydration usually results in an elevated urine concentration (USG > 1.050). Overhydration would result in an increased urine output in an effort to return to a normal hydration state. This would result in more dilute urine. Renal failure, hypoadrenocorticism (Addison's), and diabetes mellitus can all be causes of dilute urine and all cause polyuria and polydipsia

Fructosamine levels are used to evaluate the control of which of the following diseases? *A* Hypothyroidism *B* Diabetes mellitus *C* Cushing's disease *D* Addison's disease *E* Hyperthyroidism

*B* Diabetes mellitus Explanation - The fructosamine level is a measure of the amount of glucose bound to plasma proteins. It goes up when glucose levels are persistently elevated in the blood. It is less subject to the fluctuations seen in blood glucose because plasma proteins have much longer half-lives. Fructosamine levels correspond to the average serum glucose over the previous 1-2 week period. A related test is glycosylated hemoglobin level, which corresponds to average serum glucose over several months

Which of these are used in the major cross-match for blood products to be transfused to a dog? *A* Donor red blood cells, donor plasma *B* Donor red blood cells, recipient plasma *C* Recipient red blood cells, recipient plasma *D* Recipient red blood cells, donor plasma

*B* Donor red blood cells, recipient plasma Explanation - The correct answer is donor red blood cells, recipient plasma. In a major crossmatch, you are looking to see if the factors in the plasma of the recipient are going to react to the donated red blood cells

A fistulogram involves taking a radiograph after a contrast agent is administered into what location? *A* Intravenous *B* Draining tract *C* Bladder *D* Tarsus *E* Carpus

*B* Draining tract Explanation - A draining tract, also known as a fistula, can be evaluated by injecting a contrast agent into the tract and taking a radiograph to track where the tract goes.

What instrument is used to perform a castration in horses? *A* Kelly carmalt *B* Emasculator *C* Rongeurs *D* Freedom forceps

*B* Emasculator Explanation - The correct answer is an emasculator. This device crushes the spermatic cord as well as cuts it. The crushed end stays with the patient to prevent hemorrhage. Care must be taken when applying the device to ensure that the crushed end stays with the patient, otherwise major bleeding can occur

Where do sperm mature? *A* Oviduct *B* Epididymis *C* Vas deferens *D* Prostate

*B* Epididymis Explanation - The epididymis is the tube attached to the backside of the testes. It is found within the scrotum and is a coiled segment of the spermatic ducts in which spermatozoa mature. It connects the testicle with the vas deferens. Sperm matures within this tube. Oviducts, also called fallopian tubes, are found in the female reproductive tract. The egg passes from the ovary into the uterus via the oviduct. The prostate stores and secretes a portion of the seminal fluid

In which species will you never perform a reticulocyte count? *A* Canine *B* Equine *C* Bovine *D* Feline

*B* Equine Explanation - Horses do not release reticulocytes (immature erythrocytes) into circulation

Where is a meibomian gland located? *A* Testicles *B* Eyelid *C* Ear canal *D* Foot pads

*B* Eyelid Explanation - Meibomian glands line the eyelids. They secrete sebum and help to prevent evaporation of the tear film.

In neonatal calves and foals, what is the most common reason that these animals succumb to disease and death? *A* Gastrointestinal parasitism *B* Failure of passive transfer *C* Iron deficiency *D* Infection acquired by the mother from the placenta

*B* Failure of passive transfer Explanation - Newborns require essential maternal antibody absorption from the colostrum to help protect against infectious diseases. If they do not get these precious immunoglobulins, they can suffer from increased morbidity and mortality from infections. There are now field test kits that can detect blood levels of IgG in calves and foals to help determine if failure of passive transfer has occurred

Canine Distemper Virus can be transmitted to which species? *A* Pig *B* Ferret *C* Horse *D* Cat

*B* Ferret Explanation - Raccoons, ferrets, skunks and some other species can contract Canine Distemper Virus

When looking at the bottom of a horse's hoof, what is the name of the triangular shaped structure behind the sole of the foot? *A* Secondary sole *B* Frog *C* Laminae *D* Bars *E* White line

*B* Frog Explanation - The name of the triangular structure is the frog (cuneus ungulae); this structure fills in the gap between the bulbs of the heels. The frog is a soft structure and is a normal part of the hoof anatomy

Which of the following could be used for hemostasis at the site of a liver punch biopsy? *A* Bone wax *B* Gelfoam *C* Silver nitrate *D* Styptic powder

*B* Gelfoam Explanation - Gelfoam is a sterile compressed sponge that is intended to aid in hemostasis; it allows for absorption of blood and formation of a clot at the bleeding site. It has a very useful application in liver biopsies. Bone wax controls bleeding from bone and functions as a mechanical plug that is pressed into the bleeding bone surface. Silver nitrate applicators are used to stop small superficial bleeders such as a snipped skin tag or a bleeding toenail. These are not sterile. Styptic powder is used to stop bleeding toenails and would never be used on tissues

Rats are known to produce a substance called "red tears" or chromodacyorrhea. Where are red tears produced? *A* Conjunctival gland *B* Harderian gland *C* Retro-orbital gland *D* Nasal tissue gland

*B* Harderian gland Explanation - The Harderian gland produces the substance known as red tears. The red color is from a porphyrin pigment in the tears. Rats under stress may show this red tear production. Due to the connection between the tear ducts and the nasal cavity, the red stain may show up as a nasal discharge. Many owners will think it is blood

Goats exhibiting aggressive behavior may do so most commonly in which manner? *A* Kicking with the hind legs *B* Head butting *C* Urinating and defecating *D* Biting

*B* Head butting Explanation - Head butting in goats can be play behavior, but is primarily a form of aggression with other goats and possibly humans. Always watch behind you when you are in with a herd of goats, as they will often come up behind you!

An owner called and said that her dog ate a pack of sugarless gum containing xylitol. You tell her to bring her dog in right away. Which of the following could this cause? *A* Hyperglycemia *B* Hypoglycemia *C* Hypophosphatemia *D* Hypercalcemia

*B* Hypoglycemia Explanation - Xylitol is a sugar alcohol found in many sugarless candies, sweets, and gum. Xylitol toxicity can be life-threatening. Dogs that ingest doses of >0.1 g/kg of xylitol are at risk for developing hypoglycemia, while dogs that ingest >0.5 g/kg may develop acute liver failure. Supportive care and dextrose for treatment of hypoglycemia are indicated

Which electrolyte abnormality is known to cause ventroflexion of the neck and extreme weakness? *A* Hypermagnesemia *B* Hypokalemia *C* Hyponatremia *D* Hypercalcemia

*B* Hypokalemia Explanation - Hypokalemia (low potassium) can result from various metabolic and endocrine disorders. It causes muscle weakness and can cause cervical ventroflexion. Hypokalemic myopathy has been linked with chronic kidney disease and excessive potassium loss (especially in cats). Hypokalemia can also result from osmotic diuresis, correction of acidosis, or insulin-mediated cell uptake that is seen in diabetic ketoacidosis therapy. Some medications can also contribute to hypokalemia, such as furosemide, which causes excess potassium loss in the urine. Other causes include inadequate potassium intake in the diet, gastrointestinal loss, and hereditary causes; also it is often seen in cases of hepatic lipidosis

What does the word anhidrosis mean? *A* Lack of drinking *B* Inability to sweat *C* Without tears *D* Hidden disease

*B* Inability to sweat Explanation - The cause is unknown, but some horses lack the ability to sweat. This condition in known as anhidrosis. Keep horses with anhidrosis as cool as possible during hot or humid weather. Use of mist, sponging, fans, shade or stall rest can be helpful during exposure to heat

What is the name of the tick that is the primary vector for transmitting the bacteria which causes Lyme Disease? *A* Rhipicephalus sanguineus *B* Ixodes scapularis *C* Dermacentor andersoni *D* Amblyomma americanum *E* Dermacentor variabilis

*B* Ixodes scapularis Explanation - Lyme Disease is caused by Borrelia burgdorferi (a spirochete bacteria) and is transmitted by Ixodes (Deer Tick). It may cause fatigue, lameness, decreased appetite, and depression. The pet may not show symptoms until months after infection. Rhipicephalus (Brown Dog Tick) transmits Ehrlichia canis and prefers warmer climates. Dermacentor variabilis (American Dog Tick) and Dermacentor andersoni (Rocky Mountain Wood Tick) are known for transmitting Rickettsia rickettsii, which causes Rocky Mountain Spotted Fever. Amblyomma (Lone Star Tick) transmits Ehrlichia chaffeensis, which causes Human Monocytic Ehrlichiosis. It has also been implicated in other diseases

Erythropoetin (EPO) is a glycoprotein hormone produced mainly by the: *A* Spleen *B* Kidney *C* Bone marrow *D* Pituitary

*B* Kidney Explanation - EPO is produced mainly by peritubular fibroblasts of the renal cortex. It is a cytokine for red blood cell precursors in the bone marrow. This is partly why animals with chronic renal failure become anemic. The kidneys are no longer able to make EPO efficiently

Where is the most recommended area to administer a subcutaneous injection in a horse? *A* Over the withers *B* Lateral aspect of the neck *C* Caudal aspect of the forearm *D* Dorsal aspect of the flank

*B* Lateral aspect of the neck Explanation - The lateral aspect of the neck is most commonly used due to the ease of drainage if a problem occurs secondary to the injection. Horses are known to commonly have reactions to injections. If an abscess forms, the area on neck is easiest to drain

A shock organ is the organ or tissue that exhibits the most marked response to an allergic reaction. What is the shock organ in the cat? *A* Gastrointestinal tract *B* Lungs *C* Heart *D* Liver

*B* Lungs Explanation - In the cat, the shock organ is the lungs. In the dog, it is the liver. Both species often also have gastrointestinal involvement (vomiting, diarrhea).

Which medication is typically given through a filter? *A* Cefazolin *B* Mannitol *C* Dexamethasone *D* Metoclopramide

*B* Mannitol Explanation - Mannitol forms crystals in solution. An in-line 5 micron IV filter or filter needle is recommended. None of the other choices require a filter for administration. Blood products are frequently filtered prior to IV administration

Which of the following cells release histamine? *A* Platelets *B* Mast cells *C* Sebaceous cells *D* Neutrophils *E* Lymphocytes

*B* Mast cells Explanation - A mast cell contains many granules that contain histamine and heparin. They play a role in the inflammatory response, especially allergic reactions. Diphenhydramine (Benadryl) is often given pre-op for mast cell tumor removal to minimize histamine release as the mass is being manipulated. Histamine release can cause vasodilatation and, if severe enough, can lead to hypotension

Which one of the following lab abnormalities is frequently seen with acetaminophen (Tylenol) toxicity in cats? *A* Thrombocytopenia (low platelets) *B* Methemoglobinuria (brown urine) *C* Increased RBCs (red blood cells) *D* Toxic neutrophils *E* Alkalosis

*B* Methemoglobinuria (brown urine) See methemoglobinuria (brown urine), methemoglobinemia (brown blood). Oxidative damage to RBC causes Heinz body anemia . More frequent in cats than dogs. Cats are deficient in glucuronyl transferase, so they have a limited ability to metabolize acetaminophen and aspirin.

Valium is often given with ketamine to provide which effect? *A* Increased muscle tone *B* Muscle relaxation *C* Increased cardiac contractility *D* Increased heart rate

*B* Muscle relaxation Explanation - Ketamine can cause muscle rigidity. Valium (diazepam) is often given with ketamine to provide muscle relaxation.

Which of the following is the scabies mite in cats? *A* Sarcoptes gati *B* Notoedres cati *C* Cuterebra *D* Cheyletiella blakei

*B* Notoedres cati Explanation - Feline scabies is caused by the mite Notoedres cati. It often causes scaly or scabby facial lesions in cats and is intensely pruritic. It is a highly contagious mite and is closely related to the canine scabies mite Sarcoptes scabiei. It is diagnosed via a skin scraping. Demodex gati is the demodex cat mite. Sarcoptes gati is not a real mite. Cuterebra is a fly larva that may infest the skin of domestic animals. Cheyletiella is a surface mite that causes itchy and flaking skin in dogs and cats, also known as "walking dandruff".

In a cow that is having difficulty calving and seems to have her legs splaying out from underneath her, you are most concerned about damage to which nerve? *A* Peroneal *B* Obturator *C* Femoral *D* Tibial

*B* Obturator Explanation - Obturator nerve paralysis (ONP) is most common. This nerve is responsible for adducting the hind limbs and runs from the spinal cord into the birth canal before it innervates the inner thighs. During birthing, this nerve can become crushed and lead to this problem

Regarding sterilization in reptiles, which is true? *A* Reptiles should not be sterilized due to sensitivity to anesthetics *B* Ovaries should always be removed during sterilization *C* Sterilization of female pet reptiles is recommended so that they do not develop estrogen toxicity *D* Only the oviducts should be removed and ovaries should be left intact

*B* Ovaries should always be removed during sterilization Explanation - When sterilizing a reptile, the ovaries should always be removed. Removal of only the oviducts puts the patient at risk for egg binding. With just the oviducts removed, the ovaries are still active and yolks can be released into the coelomic cavity. In ferrets, spaying is recommended due to the risk of bone marrow toxicity from the high levels of circulating estrogen during prolonged heat cycles

Which of the following parasites can be associated with colic and respiratory disease in the foal? *A* Cryptosporidium *B* Parascaris equorum *C* Echinococcus equinus *D* Parelaphostrongylus tenuis

*B* Parascaris equorum Explanation - Parascaris equorum, or the equine ascarid (roundworm), typically only infests foals and can cause ill thrift, coughing, pneumonia, and colic. If a foal has a heavy burden of roundworms and is subsequently given an anthelmintic, impaction colic can result from a mass of dead worms obstructing the intestinal tract. Foals develop immunity to this organism as they age and are typically free of infestation as adults

You are taking a lateral radiograph of the left stifle of a dog to assess for a suspected cruciate ligament injury. How should the patient and leg be oriented relative to body and film? *A* Patient should be in right lateral recumbency with the leg against the cassette *B* Patient should be in left lateral recumbency with the leg against the cassette *C* Patient should be in right lateral recumbency with the leg away from the cassette *D* Patient should be in ventral recumbency with the leg to the side of the body against the cassette *E* Patient should be in dorsal recumbency with the leg to the side of the body against the cassette

*B* Patient should be in left lateral recumbency with the leg against the cassette Explanation - Having the leg against the film instead of away from the film will provide a crisper and less distorted image. Therefore, positioning the dog in left lateral recumbency with the leg against the cassette is the best position

Which breed does not typically shed its haircoat? *A* Golden retriever *B* Poodle *C* Pug *D* Labrador

*B* Poodle Explanation - Poodles do not typically shed hair. They may lose hair during chemotherapy treatments or with certain endocrine diseases

You are on a farm call with a veterinarian to investigate the cause of agalactia in the farmer's cows. These cows have: *A* Blood in their milk *B* Poor milk production *C* Low calcium *D* Infertility

*B* Poor milk production Explanation - Agalactia (agalactorrhea) is either an absence of milk production or abnormally low milk production following parturition. There are many common causes including infection (especially in sheep, goats, and pigs), poor nutrition or water deprivation, hormonal imbalances, anatomical causes(inverted teats or absence of mammary tissue, scarring of tissue), or failure of the neonate to suckle properly so that the milk let-down reflex is not stimulated

Which is true regarding blood flow in the heart? *A* Pressure in the left atrium is greater than the right atrium, so blood flows right to left *B* Pressure in the right atrium is greater than the left atrium, so blood flows right to left *C* Pressure is equal across both atria to prevent backup of blood in the heart *D* Pressure in the left atrium is greater than the right atrium, so blood flows left to right

*B* Pressure in the right atrium is greater than the left atrium, so blood flows right to left Explanation - Blood flow to the heart is explained below. Blood in the heart flows right to left, and the pressure in the right atrium exceeds the pressure in the left atrium. Right side: Blood (oxygen-poor) enters the heart through the vena cava into the right atrium. It then flows into the right ventricle through the tricuspid valve. It then goes to the pulmonary artery and to the lungs to become oxygenated. Left side: The pulmonary vein empties blood (oxygen-rich) from the lungs to the left atrium, where it proceeds to the left ventricle through the mitral valve. Blood leaves the heart into the aorta through the aortic valve to reach the rest of the body.

Giardia is what type of parasite? *A* Cestode *B* Protozoan *C* Trematode *D* Ascarid *E* Coccidial

*B* Protozoan Explanation - Protozoa are unicellular organisms. Giardia exists in two forms: the motile trophozoite and the cyst form. Trematodes are flukes. Cestodes are tapeworms. Coccidia are a different type of protozoan parasite consisting of several species of Isospora, Cryptosporidium, Toxoplasma, and others

Which of the following is considered a reportable swine disease? *A* Sarcoptic mange *B* Pseudorabies *C* Pasturella multocida *D* Erysipelas

*B* Pseudorabies Explanation - Pseudorabies is a virus caused by porcine herpesvirus 1. Wild mammals, cattle, sheep, dogs, and raccoons are also susceptible, and the disease can be fatal to these species. It is shed in oral and nasal secretions of swine and spread through oral/nasal contact or by fomites. Symptoms in pigs include abortion, nasal discharge/sneezing, seizures, depression, circling, and increased salivation. In cattle, it causes intense itching, also known as "mad itch", then seizures and death. The other diseases listed are more common in pigs and are not considered as reportable

Which of the following species has a double row of incisors? *A* Hamster *B* Rabbit *C* Rat *D* Chinchilla *E* Guinea Pig *F* Ferret

*B* Rabbit Explanation - Rabbits have a second row of incisors distal to the proximal incisors. When malocclusion occurs, it is important to trim both sets of teeth to maintain proper dentition

Which of the following is the "minute volume" of a patient under anesthesia? *A* Tidal volume divided by lung capacity *B* Respiratory rate multiplied by tidal volume *C* Respiratory rate multiplied by lung capacity *D* Oxygen consumption multiplied by tidal volume

*B* Respiratory rate multiplied by tidal volume Explanation - Minute volume is the volume of air that is inspired by a patient in one minute. It is calculated by multiplying tidal volume by the number of breaths per minute. The tidal volume is the volume of air that is inspired on each breath. The respiratory rate is the number of breaths per minute. Tidal volume is usually considerably smaller than the total lung capacity

Which organism causes equine protozoal myeloencephalitis (EPM) in the horse? *A* Haemonchus contortus *B* Sarcocystis neurona *C* Micronema deletrix *D* Gasterophilus intestinalis

*B* Sarcocystis neurona Explanation - EPM is caused by the aberrant migration of Sarcocystis neurona through the central nervous system of the horse. This causes variable clinical signs such as ataxia and muscle atrophy in the horse

Which structure is the white part of the eye? *A* Optic nerve *B* Sclera *C* Lens *D* Cornea *E* Vitreous

*B* Sclera Explanation - The sclera is the white fibrous tissue that extends from the cornea (the clear surface of the eye) to the optic nerve in the back of the eye.

What is the most likely period of time for a scrotal hernia to occur in a stallion? *A* Shortly after a race *B* Shortly after breeding *C* Shortly after a trailer ride *D* Shortly after heavy jumping

*B* Shortly after breeding Explanation - Newborn foals and breeding stallions are most commonly affected with scrotal hernias. In foals, the hernias may spontaneously resolve as long as they are addressed every time they occur. However, there is a risk that the herniated contents can become incarcerated, resulting in a surgical emergency. In breeding stallions, the contents usually become incarcerated and emergency surgery will be needed

Patients that have received prednisone chronically need to be tapered off slowly to avoid which of the following? *A* Signs of diabetes insipidus *B* Signs of hypoadrenocorticism *C* Signs of exocrine pancreatic insufficiency *D* Signs of hyperinsulinism *E* Signs of hyperadrenocorticism

*B* Signs of hypoadrenocorticism Explanation - Chronic prednisone administration can lead to atrophy of the adrenal gland due to the absence of signaling to produce cortisol. If prednisone is suddenly stopped, the adrenal gland will not be capable of producing enough cortisol, and the animal may show signs of hypoadrenocorticism (Addison's disease).

An animal that has been diagnosed with underlying cardiac insufficiency may need to be restricted on intake of: *A* Magnesium *B* Sodium *C* Thiamine *D* Potassium

*B* Sodium Explanation - Water follows salt, so an animal consuming more sodium is likely to retain more water. This may be detrimental to an animal with underlying heart disease

It is a very busy day at the clinic. Multiple emergencies walk in all at the same time. Choose the order in which the following cases should be seen. *A* Dyspnea, status epilepticus, stranguria, cervical pain, fracture, abscess *B* Status epilepticus, dyspnea, fracture, stranguria, cervical pain, abscess *C* Dyspnea, fracture, status epilepticus, cervical pain, abscess, stranguria *D* Stranguria, status epilepticus, dyspnea, fracture, abscess, cervical pain

*B* Status epilepticus, dyspnea, fracture, stranguria, cervical pain, abscess Explanation - In a perfect situation both the seizuring pet and the pet with difficulty breathing would be treated immediately. Sometimes this may not be possible and triage is necessary. Status epilepticus is a state of seizuring that is continuous. This is life-threatening, and the seizures must be stopped immediately or permanent brain and organ damage can occur. Dyspnea or difficulty breathing is always a top priority emergency. The dyspneic animal should be assessed and placed on oxygen while the seizuring dog is being treated. Fractures, while painful, are not immediately life-threatening as long as there is no bleeding vessel that needs to be controlled, which is sometimes the case with an open fracture. Straining to urinate is not usually life-threatening unless it is a "blocked cat", or urethral obstruction. If this were a blocked cat, it should be seen before the fracture. Neck pain and abscesses are not life-threatening and should be seen last if the patients are stable

Which of these drugs should not be used in Greyhounds? *A* Ivermectin *B* Thiopental *C* Morphine *D* Propofol

*B* Thiopental Explanation - The correct answer is thiopental. Thiopental is an ultra-short acting barbiturate. Recovery depends on redistribution to tissues, including fat. Because sighthounds have very little fat, they have prolonged recoveries and greater complications with these drugs

What is the best method for handling tissues during abdominal surgery when they must be placed outside of the peritoneal cavity? *A* Tissues and organs outside the abdomen should be wrapped in a dry gauze or sponge *B* Tissues and organs outside the abdomen should be covered with saline-moistened sponges *C* Tissues and organs outside the abdomen should be covered in an antibiotic-impregnated solution and sponges *D* Tissues and organs should never be exteriorized from the abdomen

*B* Tissues and organs outside the abdomen should be covered with saline-moistened sponges Explanation - Tissues and organs should always be handled with exceptional care. To avoid desiccation, all tissues/organs should be intermittently moistened with sterile saline or wrapped in saline-impregnated gauze or lap sponges. Antibiotic-impregnated solutions are not necessary, can be irritating, and may select for resistant organisms. Allowing the tissues/organs to dry will cause damage to their surfaces and result in subsequent inflammation

Which of the following is a chelonian? *A* Goat *B* Tortoise *C* Bird *D* Rabbit

*B* Tortoise Explanation - Turtles and tortoises are from the order Chelonia, and are often referred to as chelonians.

Where would you find a Cuterebra parasite? *A* In the anterior chamber of the eye *B* Under the skin *C* Protruding from the anus *D* In the intestines *E* In the nasal cavity

*B* Under the skin Explanation - Cuterebra are flies whose larvae infest the skin of rodents, squirrels, rabbits, dogs, and cats. Typically a lump is seen with a small hole under the skin from with the larva breathes

What is a pulse deficit? *A* When no pulses are palpable in a patient *B* When some heart beats do not result in a palpable pulse *C* When an extra pulse occurs sporadically *D* When a pulse is synchronous with a heartbeat

*B* When some heart beats do not result in a palpable pulse Explanation - A pulse deficit is the condition in which a peripheral pulse rate is less than the ventricular contraction rate because some heart beats do not create a palpable pulse. This can be indicative of a lack of peripheral perfusion

Which bone articulates with the patella? *A* Humerus *B*Femur *C* Tibia *D*Fibula

*B*Femur Explanation - The distal femur articulates with the patella at the level of the trochlear groove. The patella attaches to the tibial tuberosity but does not articulate with this bone. The fibula is the bone directly adjacent to the tibia. The humerus is a bone of the forelimb.

You are monitoring an adult cat under anesthesia. Since induction, the heart has been slowing down. What is the approximate lowest acceptable heart rate for this cat? *A* 180 *B* 160 *C* 120 *D* 140

*C* 120 Explanation - If the heart rate drops below 100-120 beats per minute in an anesthetized cat, the heart rate needs to be increased and the anesthetic depth should be evaluated. If appropriate, atropine may be administered in this situation, and anesthetic gas turned down if possible

You make a 1:5 dilution of a serum sample and measure the urea nitrogen. The analyzer reads that the concentration of urea nitrogen is 30 mg/dl. What is the urea nitrogen level of the patient's serum? *A* 6 mg/dl *B* 35 mg/dl *C* 150 mg/dl *D* 120 mg/dl *E* 180 mg/dl

*C* 150 mg/dl Explanation - When a dilution is made prior to analyzing a sample, you must multiply that dilution factor with the reported value to obtain the true level in the sample. In this case, the sample was diluted by a factor of 5. Therefore, 30 x 5 = 150

You want to give a pre-surgical dose of antibiotics to your patient. You have a 1 gram vial of cefoxitin that is reconstituted with 5 mls of sterile water. You want to give a 20 mg/kg dose to a 25 kg dog. How many mls do you give? *A* 7.5 *B* 10 *C* 2.5 *D* 1.25 *E* 5

*C* 2.5 Explanation - The resulting concentration of the cefoxitin solution is 1gm/5ml or 200mg/ml. The desired dose is 20mg/kg x 25 kg = 500 mg 200 mg/ml x ____ ml= 500 mg, Therefore the answer is 2.5 ml

The attending veterinarian would like you to administer a 22 mg/kg dosage of Cefazolin to a 67 pound dog that has been hospitalized. You find that the vial that has been reconstituted is empty and you need to make a new bottle. How much sterile water will you add to a 1 gram vial if you want the concentration to be 200 mg/ml? *A* 1 gram of powder can not be reconstituted to 200 mg/ml *B* 200 mls *C* 5 ml *D* 50 mls

*C* 5 ml Explanation - First convert grams to milligrams 1 gram = 1000 mg. 1000 mg divided by 200 mg/ml = 5 mls

A 60 lb Bulldog is to receive intravenous fluids at a maintenance rate. Assuming the maintenance rate can be calculated as 60 ml/kg/day, what would the fluid rate be per hour? Round answer to the closest milliliter/hour. *A* 108 *B* 150 *C* 68 *D* 30 *E* 163

*C* 68 Explanation - The answer is 68. 60 lb/2.2 kg/lb = 27.27 kgs in weight. 27.27 x 60 ml/kg = 1636 ml/day. 1636 ml/day / 24hrs/day = 68 ml/hour

When taking a radiograph of an equine limb, you take a radiograph where the portable X-ray unit source is 90 cm away from the cassette. You then move closer so that the X-ray source is now 30 cm away from the cassette. If no changes were made to the settings of the machine, what will the beam intensity be at the second radiograph relative to the first one? *A* 3 times greater at 30 cm compared to 90 cm *B* 6 times greater at 30 cm compared to 90 cm *C* 9 times greater intensity at 30 cm compared to 90 cm *D* 1/6 intensity at 30 cm compared to 90 cm *E* 1/3 intensity at 30 cm compared to 90 cm

*C* 9 times greater intensity at 30 cm compared to 90 cm Explanation - The inverse square law states the intensity of the x-ray beam is inversely proportional to the square of the distance from the X-ray source. Therefore, the intensity is greater as the cassette is moved closer to the source. To calculate the answer, the intensity of beam 1 is (x/90)^2 and the intensity of beam 2 is (x/30)^2 where x is the number of photons produced by the x-ray machine. The math here works out as follows: (x/30)^2 / (x/90)^2 = [(x/30)/(x/90)]^2 = (90/30)^2 = (3)^2 = 9

You are asked to start a dog on maintenance fluids. The dog weighs 80 pounds. What fluid rate do you start on this dog? *A* 45 mL/hr *B* 25 mL/hr *C* 90 mL/hr *D* 150 mL/hr

*C* 90 mL/hr Explanation - There are several ways to calculate maintenance fluids. Some are more accurate than others, with the more accurate ones requiring more complex math. Fortunately, in most patients, being off by a few milliliters does not adversely affect the patient. The most basic calculation to approxiamte maintenance rate fluids in a dog is: 60mL/kg/day 80 pounds/2.2= 36 kg 60mL X 36 kg = 2160 mL per day 2160 mL/24 hours in a day = 90 mL per hour

What is the neurotransmitter for skeletal muscle? *A* GABA *B* Acetylcholinesterase *C* Acetylcholine *D* Dopamine *E* Norepinephrine

*C* Acetylcholine Explanation - Acetylcholine (ACh) is the transmitter for skeletal muscle and is located at the terminal ends of each motor neuron. Acetylcholinesterase destroys ACh and thus turns off muscle activity. GABA and Dopamine are CNS neurotransmitters. Norepinephrine is an inhibitory neurotransmitter in smooth muscle.

The upper respiratory tract includes all of the following except for which of these? *A* Nasal passages *B* Trachea *C* Alveoli *D* Pharynx

*C* Alveoli Explanation - Alveoli are the sites of gas exchange between air and blood and are in the lower-most respiratory tract (lung). The other three choices are all found in the upper respiratory tract

A complete blood count (CBC) from a sick cow shows 3% reticulocytes (normal=0%). Which one of the following disease processes is most likely? *A* Lead poisoning *B* Low hemoglobin *C* Anemia *D* Nitrate toxicity *E* Toxic left shift

*C* Anemia Reticulocytes are immature red blood cells (RBCs) that are typically seen when the bone marrow tries to respond to anemia by releasing more immature RBCs (the reticulocytes). This is also called a "regenerative" anemia. Reticulocytes are found in very small percentages in normal blood of dogs and cat. Healthy ruminants do not show them, but they appear if the ruminant is responding to an anemia. Horses don't show any reticulocytes on a complete blood count (CBC) in health or anemia. Click here to see canine reticulocytes in a dog responding to an anemia. The reticulocytes are larger than regular blood cells. Click here to acute blood loss anemia in a dog-there are no reticulocytes because the body has not had time to respond yet.

You are performing an FeLV snap test on a kitten that was found by your neighbor. What type of test is this? *A* Antibody test *B* Immunodiffusion test *C* Antigen test *D* Immunohistochemistry test

*C* Antigen test Explanation - The FeLV (feline leukemia virus) test is an antigen test, meaning it is testing for the agent itself. The FIV test is an antibody test, meaning it is testing for antibodies made by the host against the agent (immune response). Antigen tests are often referred to as an ELISA test (enzyme-linked immunosorbent assay

Which of the following is the most mature neutrophil precursor? *A* Promyelocyte *B* Metamyelocyte *C* Band *D* Myeloblast *E* Prorubricyte

*C* Band Explanation - The neutrophil lineage, from most immature to mature, proceeds as follows: Myeloblast, promyelocyte, neutrophilic myelocyte, neutrophilic metamyelocyte, neutrophilic band, neutrophil. A prorubricyte is an erythrocyte precursor

Estrogen toxicity most commonly causes which side effect? *A* Hypoglycemia *B* Hypertension *C* Bone marrow suppression *D* Seizures

*C* Bone marrow suppression Explanation - Estrogen containing medications (such as Diethylstilbesterol also known as DES) or natural causes of increased estrogen (like ferrets having prolonged heat cycles) may lead to bone marrow suppression. Although bone marrow suppression is rare with low doses of DES, it is good to monitor complete blood counts in patients taking this medication to monitor for bone marrow suppression

Parathyroid hormone mostly regulates which blood value? *A* Glucose *B* Potassium *C* Calcium *D* Blood Urea Nitrogen

*C* Calcium Explanation - Parathyroid hormone is the endocrine regulator of calcium and phosphorus concentration. This hormone is secreted from the parathyroid glands and acts on target cells mostly in bones, kidneys, and small intestine. It acts by mobilizing calcium from bone, helps absorption of calcium in the intestine, and decreases calcium loss in urine. By doing these things, it helps to prevent hypocalcemia (low calcium). Parathyroid hormone also acts on the kidney to stimulate loss of phosphate ions in urine

A rupture or tear in the thoracic duct would result in which of the following? *A* Hemoabdomen *B* Pericardial effusion *C* Chylothorax *D* Pyothorax

*C* Chylothorax Explanation - Chyle (a fluid high in chylomicrons and lymph) effuses from the thoracic duct into the pleural space if the thoracic duct tears or ruptures (usually trauma-related). The effusion typically appears as a milky white fluid and is odorless. This is only one cause of chylothorax. Other causes include lymphangiectasia, lung lobe torsion, heartworm disease, cardiomyopathy, neoplasia, idiopathic, and others. If the triglyceride level of the effusion is greater than that of peripheral blood, this can help to determine the effusion is chylous

What is the single most important thing to do when removing a stomach tube from a patient? *A* Flush the tube with 5-10 ml of water *B* Advance the tube 3cm infuse 10ml of air and then remove *C* Clamp or pinch off stomach tube prior to removing *D* Twist the tube as you are pulling it out

*C* Clamp or pinch off stomach tube prior to removing Explanation - Clamping or pinching off a stomach tube just prior to removing the tube is essential to prevent spillage of the contents within the tube as it is being removed. Any inadvertent spillage could result in contents entering the airway, subsequently causing aspiration pneumonia. None of the other answer choices effectively prevent this potential complication

You are assisting with an ACTH stimulation test to test for hypoadrenocorticism (Addison's disease). ACTH stimulates the release of which of the following? *A* Amylase *B* Insulin *C* Cortisol *D* Bile

*C* Cortisol Explanation - The ACTH (cosyntropin) stimulation test measures the ability of the adrenals to respond to the hormone ACTH. This hormone is made in the pituitary and stimulates the adrenals to produce cortisol. Cushing's disease is an overproduction of cortisol due to either a pituitary or adrenal mass. Addison's disease is the lack of production of cortisol from the adrenal glands

What is the optimal method of sampling if a urine culture is to be performed in a dog? *A* Free catch *B* Urinary catheterization *C* Cystocentesis *D* Bladder expression

*C* Cystocentesis Explanation - The only way to definitively obtain a sterile urine sample is by performing a cytocentesis. All other methods carry the risk of environmental contamination with bacteria

Which of the following description of the properties of an inhalant anesthetic is true? *A* Drugs with high solubility (blood/gas solubility coefficients) are the most potent *B* Drugs with high minimum alveolar concentrations (MAC) are the most potent *C* Drugs with low minimum alveolar concentrations (MAC) are the most potent *D* Drugs with low solubility (blood/gas solubility coefficients) are the most potent

*C* Drugs with low minimum alveolar concentrations (MAC) are the most potent Explanation - MAC is a measure of anesthetic potency or, the concentration of drug required to prevent response to a stimulus in 50% of patients. Listed from lowest to highest MAC, inhalant anesthetics include methoxyflurane, halothane, isoflurane, enflurane, sevoflurane, and desflurane. This means that methoxyflurane is the most potent, or, it requires the lowest percentage of gas to cause its effects. Desflurane is the least potent (requires the highest percentage of gas to cause its effects). Potency is not necessarily related to the speed of induction and recovery with an inhalant anesthetic; this property is determined by solubility

A client calls to schedule an appointment with the doctor because her dog has been shaking his head and now his ear flap feels swollen, warm to the touch, and is squishy. What should you book the appointment as? *A* Pinnal balloon *B* Ear hemangiosarcoma *C* Ear hematoma *D* Ear canal ablation

*C* Ear hematoma Explanation - An ear hematoma occurs when the vessels inside the pinna rupture (from shaking the head, trauma, or vasculitis). The pinna (ear flap) fills with blood like a balloon. The term for this condition is an ear hematoma. The most common treatment includes placing a drain in the pinna that stays in place for a couple of weeks. Underlying ear disease should be treated as well

A dog has dislocated his hip joint and the doctor has reduced the dislocation. What can be used to hold the hip in place? *A* External fixator *B* Strict cage rest *C* Ehmer sling *D* Robert Jones bandage

*C* Ehmer sling Explanation - The Ehmer sling is a figure-eight type of bandage that keeps the affected hindlimb in the flexed position. When the hip is dislocated, the joint capsule becomes torn. The goal of the Ehmer sling is to hold the hip joint in place after the joint has been reduced back to normal positioning to allow the joint capsule to heal. This sling provides abduction and internal rotation of the femur and flexion of the knee.

Which of the following should be done with a dog after a myelogram to decrease the risk of procedure-related complications? *A* Administer a bolus of crystalloid IV fluids *B* Stabilize the neck *C* Elevate the head *D* Place a chest tube

*C* Elevate the head Explanation - After a myelogram, dogs have a risk of having a seizure due to the contrast administration. The risk can be decreased by keeping the head elevated after the procedure.

Which of the following vaccines is not available for horses? *A* Equine Influenza Virus Vaccine *B* Eastern Equine Encephalitis Vaccine *C* Equine Protozoal Myeloencephalitis Vaccine *D* Streptococcus equi (strangles) Vaccine *E* Equine Herpes Virus Type 1 Vaccine

*C* Equine Protozoal Myeloencephalitis Vaccine Explanation - While equine protozoal myeloencephalitis is a common neurologic disease in horses caused by Sarcocystis neurona, there currently is no effective vaccine to prevent this disease. The other vaccines listed are all available

What are Omega 3 and Omega 6? *A* Amino acids *B* Fat soluble vitamins *C* Fatty acids *D* Antifungals

*C* Fatty acids Explanation - Omega 3 and Omega 6 are types of essential fatty acids (meaning they are needed in the diet because the body does not produce them). These acids are needed in immune and inflammatory responses, certain hormone production, blood clotting, and cell growth. They are used in veterinary medicine most often for skin disease and as an arthritis supplement

An example of a clean-contaminated surgery would be which of the following? *A* Nephrectomy *B* Ovariohysterectomy *C* Gastric foreign body removal *D* Splenectomy

*C* Gastric foreign body removal Explanation - A clean-contaminated surgery is surgery in an area that cannot be made sterile. This includes elective opening of respiratory, gastrointestinal, biliary or genitourinary tracts with minimal spillage

After which procedure is it most important that you immediately wash your hands as soon as the procedure is complete? *A* Fluoroscopy *B* Skin scraping on a dog with Demodex *C* Holding a dog that may have Microsporum canis *D* Restraining a cat that has Feline Immunodeficiency Virus (FIV)

*C* Holding a dog that may have Microsporum canis Explanation - Microsporum canis is the most common cause of ringworm and is a zoonotic fungal disease. Fluoroscopy is a term that describes the acquisition of real-time x-rays. Demodex is a non-contagious mite. FIV is transmitted by close contact between cats. Of course, it is safe practice to always wash your hands before and after handling patients.

Which of the following species is unable to vomit? *A* Cats *B* Dogs *C* Horses *D* Cows *E* Birds

*C* Horses Explanation - The correct answer is horses. Horses are unable to regurgitate or vomit; they are less likely to develop aspiration pneumonia secondary to vomiting or regurgitation while under anesthesia. Regardless, horses are kept off food for approximately 12 hours prior to surgery to allow the stomach to empty. Taking this precaution may help the horse achieve better ventilation while under anesthesia

Bacterial sepsis frequently causes which finding on a blood panel? *A* Hypercalcemia *B* Eosinophilia *C* Hypoglycemia *D* Anemia

*C* Hypoglycemia Explanation - Hypoglycemia is an important finding in patients with sepsis. Decreased glycogen stores, impaired gluconeogenesis, and increased utilization of peripheral glucose may all contribute to this finding

A dog presents with weakness, depression, polyuria, polydipsia, and dilute urine. X-rays reveal a mediastinal mass. Where is the mediastinum located? *A* Caudal to the kidneys *B* In the cervical spine *C* In the chest *D* In the central abdomen *E* In the lumbar spine

*C* In the chest Explanation - The mediastinum is the central compartment of the chest. It is located between the right and left pleura and extends from the sternum to the vertebral column. It contains all the thoracic organs except the lungs

Which of the following delays wound healing? *A* Bed of granulation tissue *B* Increased blood supply *C* Infection *D* Jagged edges

*C* Infection Explanation - Bacteria and inflammation from infection cause direct cellular damage. This can cause separation of tissues, which leads to infection and delays healing time. Granulation tissue is a normal part of healing unless it becomes exuberant. Other factors which delay healing include corticosteroid administration or immunosuppression, poor blood supply, and hypoproteinemia

Which of the following is true of tungsten, making it an ideal material for the filament and target of an x-ray machine? *A* It has a low atomic number, making it more efficient at producing x-rays *B* It is a good conductor of heat *C* It has a very high melting point *D* It has a low number of electrons

*C* It has a very high melting point Explanation - Tungsten has a very high melting point (greater than 3,000 C). It has a high atomic number (74), making it a relatively more efficient target for producing electrons. It also has many electrons, which makes it more efficient at producing an electron cloud at the cathode (filament) via a process called thermionic emission. Despite being relatively efficient at producing x-rays, the process itself is still very inefficient, with more than 99% of the energy generated being converted to heat rather than x-rays. Therefore, the tungsten target is often embedded in copper (a good conductor of heat). Tungsten by itself is a poor heat conductor

You are performing an ear cytology and see some purple bowling pin-appearing organisms. What are these most likely? *A* Gram-negative rods *B* Pollen *C* Malassezia *D* Pseudomonas

*C* Malassezia Explanation - The purple bowling pin structures seen on a stained ear cytology slide are classic yeast (Malassezia pachydermatis). Depending on their stage, they appear large and round or look like a bowling pin. Pseudomonas is gram-negative rod and is a common culprit in resistant ear infections

A client rushes in her puppy which was accidentally dropped and received a trauma to the head. Which medication is an osmotic diuretic and may be used to help decrease intracranial pressure in a situation like this? *A* Dexdomitor *B* Diazepam *C* Mannitol *D* Dobutamine

*C* Mannitol Explanation - Mannitol is an osmotic diuretic that is often given to help decrease intracranial pressure in cases of trauma or seizures. It may also help to decrease intraocular pressures in acute glaucoma cases by dehydrating the vitreous humor. It also has some renal vasodilation properties and is sometimes given in cases of oliguric renal failure to help stimulate urine production. The other medications listed are not osmotic diuretics and would not be used in this situation. Dexdomitor (dexmedetomidine) is an alpha-2 adrenoreceptor agonist and is often used for sedation. Diazepam is often used to stop seizure activity and causes muscle relaxation. Vasopressors, such as dobutamine, cause vasoconstriction and work to increase blood pressure

Which type of cell is rich in histamine and heparin? *A* Neutrophil *B* Erythrocyte *C* Mast cell *D* Platelet

*C* Mast cell Explanation - Mast cells contain histamine and heparin. They play a role in inflammation and are thus involved in wound healing. They are mostly known for their involvement in allergies and anaphylaxis.

What does MRSA stand for? *A* Medical Rating of Staff Accreditation *B* Modified Recumbent Standing Apposition *C* Methicillin-Resistant Staphylococcus aureus *D* Moderate Radiopaque Screen Application

*C* Methicillin-Resistant Staphylococcus aureus Explanation - MRSA is a resistant bacterium that is becoming increasingly difficult to treat. Most infections are nosocomial and occur in open wounds, surgical or otherwise. This is why meticulous care and attention to cleanliness of the hospital, hand washing, and sterilization of medical equipment are so important

Which of the following drugs increases gastrointestinal motility? *A* Famotidine *B* Diphenhydramine *C* Metoclopramide *D* Barium

*C* Metoclopramide Explanation - Prokinetic drugs, such as metoclopramide, increase gastrointestinal motility. Metoclopramide (Reglan) also acts centrally as an antiemetic by blocking dopamine and serotonin receptors in the chemo-receptor-trigger-zone (CRTZ). Famotidine is an H2 blocker. Diphenhydramine helps prevent mast cell degranulation. Barium is used to perform contrast studies of the gastrointestinal tract.

Which antibiotic often used for diarrhea has potential to cause a neurologic side effect? *A* Amoxicillin *B* Tylosin *C* Metronidazole *D* Bismuth subsalicylate

*C* Metronidazole Explanation - Metronidazole is used frequently for treating diarrhea. It can cause neurologic symptoms in some sensitive patients or at higher dosages. The treatment for metronidazole toxicity is supportive care, including Diazepam as a main treatment. Owners with pets on higher dosages of this medication should be advised of this potential side effect. The most common use for higher doses is treatment of Giardia. Fenbendazole is often a safer option for this parasite

Choose the proper corresponding acronym for the following in order: Right eye, left ear, both ears, both eyes, left eye. *A* AD, OS, OU, AD, AS *B* OS, AS, AU, OU, OD *C* OD, AS, AU, OU, OS *D* OU, AU, AS, OD, OS

*C* OD, AS, AU, OU, OS Explanation - Think O for ocular and A for auricular. OD-Right eye OS-Left eye OU-Both eyes AD-Right ear AS-Left ear AU-Both ears

Orf is the cause of contagious ecthyma primarily in which species? *A* Porcine *B* Canine *C* Ovine *D* Bovine

*C* Ovine Explanation - Ovine (sheep) and goats are the species which may contract Orf, also known as "soremouth". It is zoonotic and caused by a parapox virus. It is transmitted via direct contact with infected animals or by contact with fomites, so handlers should wear gloves when dealing with infected animals. It is a benign and self-limiting disease in most people but may be serious in immunocompromised people. Symptoms in sheep and goats include papules or pustules on the lips and muzzle that may then crust and bleed.

Yersinia pestis is the causative agent for which zoonotic disease? *A* Cat scratch fever *B* Parrot fever *C* Plague *D* Q fever

*C* Plague Explanation - Yersinia pestis is the causative agent of plague. Plague is usually transmitted by the bites of infected fleas. Yersinia pestis is a gram-negative bacterium and has a bipolar safety-pin appearance. Definitive diagnosis is based on culture. However, before collecting any samples, the state vet or CDC should be contacted. Plague is highly zoonotic, and bubonic, septicemic, pneumonic, and meningeal forms can occur. Early treatment is critical for survival. Antibiotic treatment early in the course of disease can greatly improve prognosis. Cat scratch fever is caused by the bacterium Bartonella henselae. Parrot fever is caused by the bacterium Chlamydophila psittaci. Q fever is caused by the bacterium Coxiella burnetii

Which of the following should never be given in a bolus to a patient? *A* Lidocaine *B* Hetastarch *C* Potassium chloride *D* 0.9% Sodium chloride *E* Mannitol

*C* Potassium chloride Explanation - Rapid infusion of potassium can cause cardiac arrest. There is never a good reason to bolus potassium chloride; however sometimes it is inadvertently bolused when administering fluids which had potassium chloride added to them. Administration of greater than 0.5 meq/kg of potassium chloride can be deadly. All other choices as long as appropriately dosed may be bolused.

Chronic exposure to which hormone leads to a pyometra? *A* Testosterone *B* Estrogen *C* Progesterone *D* Luteinizing hormone

*C* Progesterone Explanation - Pyometra is often related to increased progesterone levels and their effect on the lining of the uterus. Prolonged exposure to estrogen can cause bone marrow suppression.

Casts in the urine generally indicate damage in what part of the urinary system? *A* Glomerulus *B* Ureters *C* Renal tubules *D* Urethra *E* Bladder

*C* Renal tubules Explanation - Casts form in the distal and collecting tubules of the kidney because urine is the most concentrated and acidic in these areas.

Which of the following tests is best for evaluating a dog with possible exocrine pancreatic insufficiency? *A* Serum amylase *B* Serum pancreatic lipase immunoreactivity (PLI) *C* Serum trypsin-like immunoreactivity (TLI) *D* Serum lipase

*C* Serum trypsin-like immunoreactivity (TLI) Explanation - TLI is a sensitive and specific test for exocrine pancreatic insufficiency in dogs. The other tests listed are used more commonly to diagnose pancreatitis

In the heart, depolarization occurs when ________ moves into the cells. *A* Chloride *B* Potassium *C* Sodium *D* Calcium

*C* Sodium Explanation - Rapid depolarization occurs when sodium channels in the sarcolemma open and allow an influx of sodium ions into the cardiac muscle cells. A plateau phase happens when calcium enters the cytosol of the muscle cell. Repolarization occurs as potassium channels open and potassium diffuses out of the cell.

What is the biggest risk when restraining a rabbit? *A* Broken ear *B* The rabbit can easily escape *C* Spinal cord trauma *D* Inguinal hernia

*C* Spinal cord trauma Explanation - Rabbits have very strong hindlimbs that must be restrained properly. This will prevent them from kicking. If not, they will kick and risk severe spinal cord trauma that could result in a broken back or permanent paralysis. A rabbit should never be picked up or restrained by its ears

A cat presents in respiratory distress and displays very rapid and shallow breathing. Which of the following is an accurate description of this breathing pattern? *A* Agonal *B* Tachycardic *C* Tachypneic *D* Obstructive

*C* Tachypneic Explanation - Tachypnea is rapid breathing. Tachycardia is a rapid heart rate. Agonal breathing is characterized by shallow, slow, irregular inspirations followed by irregular pauses. Obstructive breathing is characterized by a slow inspiratory phase followed by a rapid expiratory phase and can sometimes have respiratory noise (i.e. stertor) associated with breathing

What maintains the membrane potential in excitable cells? *A* Calcium channel pump *B* Neurons *C* The sodium-potassium pump *D* Chloride anions

*C* The sodium-potassium pump Explanation - The principal ions involved in an action potential are sodium and potassium cations. Sodium enters the cell, and potassium leaves the cell and restores equilibrium. The sodium-potassium pump maintains the normal ratio of ion concentrations across the membrane. Calcium cations and chloride anions are involved in a few types of action potentials, such as the cardiac action potential.

The autonomic nervous system refers to what? *A* The cranial nerves *B* The limbic nervous system *C* The sympathetic and parasympathetic nervous systems *D* The central nervous system (CNS) *E* The peripheral nervous system (PNS)

*C* The sympathetic and parasympathetic nervous systems Explanation - The sympathetic and parasympathetic nervous systems direct many of the body's homeostatic functions and is made up of nerves that innervate internal organs and blood vessels. The central nervous system consists of the brain and spinal cord. The peripheral nervous system is composed of cranial and spinal nerves. There is actually some overlap between these two arbitrary divisions. The limbic nervous system is actually within the brain and is thought to be associated with emotional responses in humans and other animals

Why should pigs be fed from a bin or trough and not on the ground? *A* It causes them to swallow excessive amounts of air *B* They cannot swallow well unless their food is elevated from the floor *C* To prevent development of trichobezoars *D* It causes gastroesophageal reflux

*C* To prevent development of trichobezoars Explanation - A trichobezoar is a ball of swallowed hair that collects in the stomach and can fail to pass through the intestines. Pigs are constantly shedding hair, so it accumulates on the ground; when eating food off that ground, pigs will ingest enough over time to put them at risk for formation of a trichobezoar

What is the name for the triangular region of the bladder where the ureters and urethra are connected to the bladder? *A* Fundus *B* Medulla *C* Trigone *D* Body *E* Cortex

*C* Trigone Explanation - The trigone is a smooth triangular part of the internal urinary bladder formed by the two ureteral openings and the internal urethral opening. The fundus is an anatomical term referring to the portion of an organ opposite from its opening (i.e. fundus of the stomach). Medulla generally refers to the middle of an organ (i.e. renal medulla). Cortex generally refers to the outer layer of an organ (i.e. adrenal cortex).

Which of these forms of chocolate contains the highest concentration of theobromine? *A* Semi-sweet chocolate *B* Milk chocolate *C* Unsweetened baking chocolate *D* White chocolate

*C* Unsweetened baking chocolate Explanation - The correct answer is unsweetened baking chocolate. This contains about 7 times more theobromine than milk chocolate. White chocolate has very little methylxanthines. Semi-sweet chocolate rates in between. Theobromine per oz. of milk chocolate= 44-60 mg/oz Baker's chocolate has 390-450 mg/oz, Semi-sweet chocolate has about half as much as Baker's or dark chocolate

What structure connects the kidneys to the bladder? *A* Trigone *B* Loop of Henle *C* Ureter *D* Urethra

*C* Ureter Explanation - The kidneys connect to the bladder via the ureters. The bladder excretes urine to the outside via the urethra. The trigone is the smooth triagular region inside the bladder that is formed by the area where the ureters and urethra attach to the bladder. The Loop of Henle is part of the nephron in the kidney.

Which of the following is NOT a good recommendation to make to an owner after having had a declaw performed on their cat? *A* Restrict his activity as much as possible *B* Monitor the bandages for slippage *C* Use clay based litter in the litter box *D* Discourage licking or chewing of the toes

*C* Use clay based litter in the litter box Explanation - Typical "gravel-type" clay litter will stick to the incisions and may perpetuate complications with healing and subsequent osteomyelitis. Therefore, it is important to confirm that owners are using approved litter such as torn paper towels or newspaper to help prevent small particles from embedding into the healing incisions. The other answer choices are good recommendations to make

What should be done when administering nitroglycerin paste to a patient? *A* Have the patient on oxygen *B* Apply it directly to the mucous membranes *C* Wear gloves *D* Have the patient hooked up to IV fluids to prevent hypotension

*C* Wear gloves Explanation - Nitroglycerine is a potent vasodilator used in cases of heart failure. It is critical to always wear gloves when handling this medication as it is absorbed through the skin. Mostly, it is applied to the skin inside the pinna of the ear. The ear should be marked with tape so that others know not to touch the area after the medication has been applied

Which of the following is true regarding the properties of x-rays compared to visible light? *A* X-rays have shorter frequency and shorter wavelength *B* X-rays and visible light have the same wavelength *C* X-rays have higher frequency and shorter wavelength *D* X-rays have higher frequency and higher wavelength

*C* X-rays have higher frequency and shorter wavelength Explanation - The energy of a photon is directly proportional frequency and inversely proportional to wavelength. X-rays are higher energy rays than photons in the visible light spectrum, which gives them greater penetrating ability

A client calls and would like her dog's phenobarbital refilled. He is currently getting 1/4 grain every 12 hours. How many milligrams is in 1/4 grain? *A* 58.0 mg *B* 64.8 mg *C* 32.4 mg *D* 16.2 mg

*D* 16.2 mg Explanation - 1 grain is equal to 64.8 mg. Therefore, 1/4 of 64.8 mg is equal to 16.2 mg. Phenobarbital is commonly dispensed in "grains".

While caring for a week old orphan puppy, it is important to monitor body temperature. Rectal temperature should be between: *A* 92-94F *B* 101-103F *C* 99-101F *D* 94-99F

*D* 94-99F Explanation - If the temperature falls below 94F, the heart rate will drop and intestinal motion ceases. Death will occur if not corrected

You are assisting with chemotherapy treatments. Which of the following is true regarding blood collection from chemotherapy patients? *A* The patient should not have blood drawn within 72 hours of their last chemotherapy treatment *B* All chemotherapy patients should be sedated prior to blood draws *C* Do not use the jugular vein if possible *D* All blood samples should be drawn from the jugular if possible

*D* All blood samples should be drawn from the jugular if possible Explanation - Blood should be drawn from the jugular veins when possible. Peripheral veins are spared for intravenous chemotherapy administration or for catheter placement during hospitalization stays. The other answer choices are false.

Diphenhydramine is typically used in treating which of the following conditions? *A* Urinary tract infections *B* Urinary incontinence *C* Keratoconjunctivitis sicca *D* Allergic reactions *E* Auto-immune conditions

*D* Allergic reactions Explanation - Diphenhydramine is also known as Benadryl. It is a an anti-histamine most commonly used for treating allergies or allergic reactions. A urinary tract infection would require an antibiotic. Keratoconjunctivitis sicca is an immune mediated condition and is usually treated with cyclosporine. Auto-immune conditions also call for immunosuppressive agents. Urinary incontinence may be treated with phenylpropanolamine.

MRI-compatible equipment might be made of which of the following metals? *A* Nickel *B* Iron *C* Cobalt *D* Aluminum

*D* Aluminum Explanation - For safety purposes, whenever there is any question, no metal should be brought in or near an MRI suite. However, magnets in MRI machines attract ferrous metals which are iron, nickel, and cobalt. There are many non-ferromagnetic metals including aluminum, titanium, brass, and others. Many MRI-compatible devices are made with these. Alloys such as stainless steel may or may not contain ferromagnetic metals

What causes cutaneous larval migration in humans? *A* Echinococcus *B* Toxocara *C* Dipylidium *D* Ancylostoma

*D* Ancylostoma Explanation - The correct answer is Ancylostoma. These are hookworms; when they come in contact with unprotected skin, the infective larvae penetrate the epidermis but generally cannot penetrate the basement membrane. They therefore migrate aimlessly, and the disease is usually self-limiting in humans

Which drug inhibits platelet aggregation? *A* Enalapril *B* Metronidazole *C* Amoxicillin *D* Aspirin

*D* Aspirin Explanation - Aspirin has antithrombotic effects (it is a COX-1 inhibitor); it blocks the platelet enzyme cyclooxygenase (aspirin is a Cox-1 inhibitor) resulting in inhibition of platelet aggregation

To assess liver function in a patient with a possible portosystemic liver shunt, which of the following tests is most likely to be helpful? *A* Insulin:glucose ratio *B* Dexamethasone suppression test *C* Water deprivation test *D* Bile acids test

*D* Bile acids test Explanation - A bile acids test is the only choice here that is a test of liver function. Bile acids are one of the most helpful tests in evaluating liver function, particularly in cases of portosystemic shunts. In this test, a fasted blood sample is drawn and a post-feeding blood sample is drawn to compare bile acid levels. Normally, bile acids increase after a meal due to release from the gall bladder, but they do not increase too much because they are reabsorbed by the liver through the portal circulation. In animals with liver dysfunction, including portosystemic shunts, bile acids are not reabsorbed and accumulate in the systemic circulation after a meal. Insulin-to-glucose ratio is a test that may be used to diagnose functional insulin-secreting tumors. Dexamethasone suppression tests are typically used to diagnose Cushing's disease. A water deprivation test may be used to diagnose diabetes insipidus

Blood lactate is a marker of what? *A* Activity of the liver *B* Amount of tissue carbon dioxide production *C* Activity of muscle *D* Blood perfusion and oxygen delivery to tissues

*D* Blood perfusion and oxygen delivery to tissues Explanation - Blood lactate is a marker of perfusion and oxygen delivery to the tissues. When there is adequate oxygen present in the patient, aerobic metabolism (oxidative phosphorylation) occurs, producing energy. When there is lack of oxygen delivery to the body or a specific tissue/organ, anaerobic metabolism occurs and produces lactate as a by-product. Instances in which lactate may be elevated include sepsis, GDV in dogs, and strangulating intestinal lesions in horses

Increased input from the vagus nerve can result in which of the following responses? *A* Tachycardia *B* Seizures *C* Vomiting *D* Bradycardia

*D* Bradycardia Explanation - Excessive vagal tone can cause a slower heart rate (bradycardia). Vomiting can sometimes stimulate the vagus nerve but vagal nerve stimulation should not cause vomiting. The vagus nerve carries parasympathetic fibers

Myoglobinuria (pigment in the urine) from muscle damage causes the urine to appear what color? *A* Green *B* Bright yellow *C* Purple *D* Brown

*D* Brown Explanation - Myoglobinuria causes the urine to appear red or brown in color but is negative for red blood cells. This can occur from muscle damage, such as in instances of trauma, electrocution, or exertional myopathy ("tying-up" in horses).

Which test measures somatic cells to identify mastitis in cows? *A* Bacterial Somatic Count Test *B* Somatic Udder Test *C* Connecticut Mastitis Test *D* California Mastitis Test

*D* California Mastitis Test Explanation - The California Mastitis Test (also called the CMT) counts somatic cells, mostly neutrophils, in cow's milk to identify mastitis (bacterial udder infection).

Which of the following is the correct term for dental decay, causing demineralization of the hard tissues of the tooth? *A* Enamel hypoplasia *B* Odontoclastic resorptive lesions *C* Stomatitis *D* Caries

*D* Caries Explanation - Dental caries is the condition where trapped food is fermented by bacteria, forming acids that demineralize the hard tissue of the tooth. Odontoclastic resorptive lesions are a process where the cementum of the tooth is resorbed and replaced with bone-like material. Enamel hypoplasia, is the absence of enamel formation on the tooth crown during tooth development. Stomatitis is inflammation of the oral mucosal surfaces

Sodium-phosphate-containing enemas are specifically contraindicated in which group? *A* Geriatric animals *B* Dogs weighing more than 20 kg *C* Animals with megacolon *D* Cats *E* Pot-belly pigs

*D* Cats Enemas containing sodium phosphate (like Fleet enemas, a human product) are contraindicated in cats and small dogs because they can cause severe hyperphosphatemia and subsequent hypocalcemia. Avoid administration of phosphate-containing enemas to: Cats Small dogs (under 10 kg) Patients with severe obstipation or compromise of the colonic wall Patients with compromised renal function Patients with hypernatremia, hyperphosphatemia, or hypocalcemia. Sodium biphosphate or sodium phosphate enemas are hyperosmotic cathartics that draw fluid into the intestine by osmosis.

What is the term given to the end of the spinal cord? *A* Corte finale *B* Spinous terminata *C* Cordus altus *D* Cauda equina

*D* Cauda equina Explanation - The cauda equina is at the lower end of the spine and is made up of nerve roots. The base of this region includes around 3-5 lumbar, 5 sacral, and a coccygeal nerve. The reason it is called the cauda "equina" is that the appearance of this area grossly resembles a horse's tail.

What is a round or spherical shaped bacteria called? *A* Bacillus *B* Nematode *C* Pleomorphic *D* Coccus *E* Spirochete

*D* Coccus Explanation - Cocci are spherical bacteria. Bacilli are rods. Spirochetes are spiral shaped. A nematode is a parasite, not a bacterium. Pleomorphic describes variably shaped organisms

Through which structures does light pass as it moves from the outside to the inside of the eye? *A* Lens, retina, cornea, vitreous *B* Retina, lens, pupil, cornea *C* Cornea, iris, retina, lens *D* Cornea, pupil, lens, retina

*D* Cornea, pupil, lens, retina Explanation - The cornea is the clear coating that surrounds the eyeball itself, and light passes through the cornea first. The iris is the colored part of the eye which encompasses the pupil. The iris controls the amount of light coming in to the eye by dilating or constricting the pupil. After passing through the pupil, light reaches the lens, which focuses light onto the retina. The vitreous body is located behind the lens and in front of the retina.

A diet high in concentrates or grains may lead to which of the following in cattle? *A* Hyperlipidemia *B* Hypocalcemia *C* Increased rumen pH *D* Decreased rumen pH

*D* Decreased rumen pH Explanation - Rumenal acidosis (decreased pH) may occur when feeding a high concentrate diet. This is due to microbes fermenting the starches in the grains too quickly such that the pH drops dramatically (less than a pH of 5.5). This increased acidity in the rumen can cause severe damage to the rumenal papillae

Which of the following diseases would be most likely to cause the development of cataracts in dogs? *A* Hypoadrenocorticism *B* Hypothyroidism *C* Toxocara canis *D* Diabetes mellitus

*D* Diabetes mellitus Explanation - Diabetes mellitus very commonly causes the development of cataracts in dogs. Cats do not typically get cataracts from diabetes. Toxocara canis (roundworm intestinal parasite) may cause ocular larval migrans in humans and thus is a zoonotic disease. It does not cause cataracts. Hypoadrenocorticism (Addison's disease) does not cause cataracts. Hypothyroidism could lead to elevated blood lipids and an increased chance of lipid corneal deposition but would not lead to cataract formation

What is the name for the blood pressure measurement when the ventricles of the heart are relaxed? *A* Mean corpuscular volume *B* Mean arterial pressure *C* Systolic pressure *D* Diastolic pressure *E* Central venous pressure

*D* Diastolic pressure Explanation - Diastolic pressure (DP) is the minimum arterial pressure during relaxation and dilation of the ventricles, which represents the pressure when the heart is resting. The systolic pressure (SP) is the pressure when the heart is contracting. Mean arterial pressure (MAP) is essentially an average blood pressure that represents the perfusion pressure seen by the organs in the body. It is calculated by using the systolic and diastolic pressures as seen by the formula: MAP=DP + 1/3 (SP-DP) Central venous pressure (CVP) is the pressure of blood in the thoracic vena cava and is a good estimation of right atrial pressure. CVP reflects the amount of blood returning to the heart and the ability of the heart to pump blood. Mean corpuscular volume is a laboratory term used to describe average volume of a red blood cell that helps to classify an anemia as microcytic or macrocytic.

Which of the following correctly lists the species in order from shortest to longest gestation length? *A* Ferret, Dog, Sheep, Pig, Horse *B* Pig, Sheep, Horse, Cow, Llama *C* Cat, Ferret, Sheep, Cow, Horse *D* Dog, Pig, Sheep, Horse, Llama

*D* Dog, Pig, Sheep, Horse, Llama Explanation - This list of gestations should be committed to memory: Llama 1 year (350 days), Horse 11 months (330 days), Cow 9 months (280 days), Sheep/Goat 5 months (150 days), Pig 4 months (114 days), Dog/Cat 2 months (63 days), Ferret 1.5 months (42 days)

The term pseudocyesis means which of the following? *A* Infertility *B* Resembling a cystic structure *C* Ovarian cysts *D* False pregnancy

*D* False pregnancy Explanation - Pseudocyesis is a false pregnancy. Dogs can become pseudopregnant after estrus when the female dog is not bred, or if it is bred by an infertile male. Pseudopregnant dogs can develop mammary glands, lactate, and have nesting behaviors. Spaying is the best way to prevent future episodes

A patient who has been diagnosed with pancreatitis should avoid foods that are high in which of the following? *A* Carbohydrates *B* Water *C* Vitamins *D* Fat *E* Protein

*D* Fat Explanation - Pancreatitis means inflammation of the pancreas, the glandular organ that secretes enzymes needed to digest food. When something causes these enzymes to be activated prematurely, they can actually begin to digest the pancreas itself, resulting in pain and inflammation. There is evidence that feeding a low protein, high fat diet can induce pancreatitis

Which of the following drugs is commonly delivered via a transdermal patch? *A* Budesonide *B* Butorphanol *C* Diazepam *D* Fentanyl

*D* Fentanyl Explanation - Fentanyl is often administered via a transdermal patch. It provides analgesia for approximately 4 days, and absorption rates may vary. For post-operative uses, it is best to apply the patch at least 12 hours before the procedure if possible.

During which of the following periods is a human embryo or fetus most sensitive to adverse effects from radiation? *A* Second trimester *B* Third trimester *C* The is no difference in the risk based on the stage of pregnancy *D* First trimester

*D* First trimester Explanation - The first 3 months of pregnancy are the most important in terms of radiation safety because the embryo/fetus is most sensitive at this time. This heightened sensitivity is due to the organogenesis that occurs during the first trimester

What test can be performed to confirm diabetes if a stressed cat has hyperglycemia? *A* Insulin level *B* Blood glucose curve *C* Urine culture *D* Fructosamine

*D* Fructosamine Explanation - Excitement, stress, and fear can increase blood glucose levels. To determine if a patient has diabetes, a fructosamine level can be of benefit. The fructosamine represents an average glucose reading for the previous 2 weeks. Blood glucose curves can be used to monitor diabetic regulation in pets that are on insulin

All of the following can be expected to be found on a complete blood count EXCEPT for which of the following? *A* Hemoglobin *B* Hematocrit *C* Neutrophils *D* Glucose

*D* Glucose Explanation - A glucose level is not part of a complete blood count. Glucose would be reported on a chemistry panel.

Which anesthetic gas would be most likely to induce a cardiac arrhythmia? *A* Isoflurane *B* Sevoflurane *C* Oxygen *D* Halothane

*D* Halothane Explanation - While any of the inhaled anesthetics may cause cardiac arrhythmias, they would be more likely to occur with the use of Halothane gas. Because of this, most veterinary practices have moved toward the use of Isoflurane or Sevoflurane for anesthetic maintenance during procedures

Which of these animals does not have a gall bladder? *A* Cow *B* Bird *C* Cat *D* Horse *E* Rabbit

*D* Horse Explanation - Horses do not have a gall bladder. The other species listed all do

Amphotericin B is often used in cases of resistant fungal infections, such as occurs with diseases like Coccidioides immitis. This medication is known for the potential to cause severe damage to which organ? *A* Liver *B* Intestine *C* Bladder *D* Kidneys *E* Skin

*D* Kidneys Explanation - Renal parameters must be followed closely in animals that are receiving amphotericin B injections

Which of the following medications blocks the chemoreceptor trigger zone to help reduce nausea and vomiting in small animals? *A* Sucralfate *B* Famotidine *C* Metronidazole *D* Metoclopramide

*D* Metoclopramide Explanation - The chemoreceptor trigger zone (CTZ or CRTZ) is part of the medulla that communicates with the vomiting center to initiate vomiting. Metoclopramide (Reglan) is an anti-emetic primarily used to treat nausea and vomiting in small animals. This medication blocks the chemoreceptor trigger zone. Sucralfate is a gastric "band-aid" that helps to protect and heal irritated mucosa. Famotidine (Pepcid AC) is a gastric acid reducer. Metronidazole is an antibiotic often prescribed for diarrhea or colitis

Nerve axons which are covered with a white sheath are known as being which of the following? *A* Sympathetic *B* Dead *C* Slow transmitters *D* Myelinated

*D* Myelinated Explanation - The myelin sheath is lipid-rich (made of cholesterol and phospholipids) which gives it its white color. The purpose of the myelin sheath is to increase the speed at which impulses can travel.

At your small animal clinic, the anesthesia cost is included for elective surgeries. An example of this type of surgery is which of the following? *A* Aural hematoma repair *B* Splenectomy *C* Cystotomy *D* Neuter

*D* Neuter Explanation - Spays, neuters, and routine dentals are examples of elective surgeries. They are surgeries that are scheduled in advance and are not a medical emergency. The other procedures listed are medically necessary and not considered elective

Viruses with this characteristic are significantly more resistant to environmental degradation; in other words, they survive longer in the environment. *A* Enveloped viruses *B* RNA viruses *C* DNA viruses *D* Non-enveloped viruses

*D* Non-enveloped viruses Explanation - The correct answer is non-enveloped viruses. Non-enveloped viruses are typically very resistant to environmental degradation and therefore survive longer and are harder to disinfect than enveloped viruses. For example, parvoviruses, which are non-enveloped viruses, are difficult to disinfect and survive longer compared to influenza viruses which are enveloped. Whether a virus is a DNA or RNA virus has no effect on its environmental durability

What type of estrous cycle does a cow have? *A* Induced ovulation when bred *B* Polyestrous in the fall *C* Estrus every 6 months *D* Non-seasonal polyestrous

*D* Non-seasonal polyestrous Explanation - Cows have non-seasonal polyestrous cycles. This means that they have estrus cycles year round. Average estrus is 21 days and lasts for 18 to 24 hours, but heat stress can shorten this window. Estrus is the time of "standing heat" when the cow will stand to be mounted by the bull. Sheep are polyestrous in the fall. Cats are induced ovulators.

A cat with type A blood is in need of a transfusion, what type of blood is it able to receive? *A* Only AB blood *B* Both AB and A blood *C* Only B blood *D* Only A blood

*D* Only A blood Explanation - A cat with type A blood can only receive type A blood. Type B cats can only receive type B blood, while AB cats are able to receive type A or type B blood

On a bone marrow smear, which of the following is the most immature red blood cell precursor? *A* Metarubricyte *B* Reticulocyte *C* Metamyelocyte *D* Rubriblast

*D* Rubriblast Explanation - The erythorocyte lineage, from most immature to mature, proceeds as follows: Rubriblast, prorubricyte, basophilic rubricyte, polychromatophilic rubricyte, metarubricyte, reticulocyte. The metamyelocyte is part of the lineage of the segmented leukocytes

If a veterinarian says an animal has pruritus, which of the following clinical signs is the animal likely displaying? *A* Diarrhea and vomiting *B* Lethargy *C* Polyuria and polydipsia *D* Scratching *E* Coughing

*D* Scratching Explanation - Pruritus is the term indicating that an animal is itchy. Scratching is the most common sign of pruritus. Others might include rubbing or head shaking, depending on the location of the pruritus

Electrical activity that serves as the pacemaker in the heart originates from what specific area? *A* Atrioventricular node in the left atrium *B* Sinoatrial node in the left atrium *C* Atrioventricular node in the right atrium *D* Sinoatrial node in the right atrium *E* Bundle of His in the left ventricle *F* Bundle of His in the right ventricle

*D* Sinoatrial node in the right atrium Explanation - In health, the pacemaker of the heart is the sinoatrial node within the right atrium. This impulse is transmitted to the atrioventricular node located at the top of the interventricular septum, followed by the bundle of His (within the walls of the interventricular septum) and through the Purkinje system

For which procedure is a celiotomy performed? *A* Hemilaminectomy *B* Pericardial window *C* Lung lobectomy *D* Spay

*D* Spay Explanation - A celiotomy is another name for laparotomy and involves making an incision into the abdomen. An ovariohysterectomy requires an incision into the abdomen, or a celiotomy. A pericardial window is a procedure to remove the pericardial sac from the heart and does not require an abdominal incision. A lung lobectomy is removal of a lung lobe and requires a thoracotomy (opening of the chest), not a celiotomy. A hemilaminectomy is a surgery used to relieve spinal cord compression

What is a sarcolemma? *A* Connective tissue putting muscle fibers into groups *B* A contractile cell in the muscle *C* A calcium storage unit in the muscle *D* The cell membrane which encloses a muscle fiber

*D* The cell membrane which encloses a muscle fiber Explanation - A sarcolemma is a thin membrane enclosing a striated muscle fiber, or simply the cell wall of a muscle cell. A sarcomere is the contractile unit of a myofibril, or muscle cell. Sarcomeres are repeating units and are located between Z lines of the muscle cells. Perimysium is connective tissue that groups individual muscle fibers into bunches or groups

Which of the following is true about the appearance of an image relative to the orientation of the slide when viewing through a compound microscope? *A* The image is right-side up and oriented normally left to right *B* The image is right-side up and reversed *C* The image is upside-down and oriented normally left to right *D* The image is upside-down and reversed

*D* The image is upside-down and reversed Explanation - Images are seen upside-down and reversed when viewed on a compound microscope. The left side of the image is seen on the right side of the visual field. Accordingly, movement of the stage is reversed such that as the stage is moved to the left, the image appears to move to the right

A dog is being referred to the neurologist for a cervical disk compression that is causing forelimb paresis. Where will the surgical incision likely be made? *A* The dorsal neck region *B* Inside the mouth *C* Between the shoulder blades *D* The ventral side of the neck

*D* The ventral side of the neck Explanation - A cervical disk surgery typically requires an incision made on the underside of the neck. This surgery is referred to as a "ventral slot".

Which of the following vitamins are fat-soluble? *A* Vitamin C, Vitamin K *B* Vitamin B12, Vitamin K *C* Vitamin C, folate *D* Vitamins A, D, E, and K

*D* Vitamins A, D, E, and K Explanation - The fat-soluble vitamins are A, D, E, and K. Vitamin C (ascorbic acid) and the B-complex vitamins are water-soluble

A castrated male goat is known as a: *A* Gelding *B* Barrow *C* Buck *D* Wether

*D* Wether Explanation - Wether is the term for a neutered male goat. Buck is an intact male goat. Barrow is a castrated male pig. Gelding is a castrated male horse

Which drug is a commonly used sedative for horses? *A* Propofol *B* Diazepam *C* Ketamine *D* Xylazine

*D* Xylazine Explanation - Xylazine is a very common sedative for horses. Detomidine is also commonly used for sedation in horses. Both of these drugs are alpha 2-adrenergic agonists used for their sedative and analgesic effects

Which drug can reverse the effects of xylazine (alpha2-adrenergic agonist with analgesic and sedative effects)? *A* Diazepam (Valium®) *B* Pralidoxime *C* Atropine *D* Yohimbine

*D* Yohimbine Yohimbine is the reversal agent for xylazine. Remember "X goes with Y and Y goes with Z". Xylazine is reversed by Yohimbine. Yohimbine is also an antidote for amitraZ, the drug used to treat generalized demodicosis. Remember 2 things about xylazine: 1. Cattle are EXTREMELY SENSITIVE. Cow xylazine dose is about 20 times LESS than dogs or horses 2. Xylazine is used as an EMETIC in CATS, causes vomiting. Pralidoxime is used to treat organophosphate toxicity, along with a sedative for seizures, diazepam (Valium ®) or pentobarbital) and Atropine.

You want to give a 22 pound cat a 5 mg/kg dose of a drug. The drug comes as a 2.5% solution. How many milliliters should you give the cat? *A* 0.2 *B* 20 *C* 0.4 *D* 4.4 *E* 2 *F* 8.8

*E* 2 Explanation - First, you must recognize that the cat's weight is given in pounds and should immediately be converted to kilograms. There are 2.2 lbs/kg so this is a 10 kg cat. Second, you must be able to convert a percent solution to mg/ml. A 2.5% solution is 25 mg/ml. This is hard for some students to remember. A memory tip is that a 100% solution would be 1 gram/ml. Therefore a 50% solution would be 500mg/ml, and a 5% solution would be 50 mg/ml. The math to solve this question is: 22 lbs x (1 kg/2.2 lbs) x 5mg/kg x (1ml/25mg) = 2 ml

How many chambers does the stomach of a llama have? *A* 6 *B* 1 *C* 2 *D* 4 *E* 3

*E* 3 Explanation - The llama has a stomach that functions similar to that of a true ruminant, but it only has 3 compartments (rumen, omasum, and abomasum). True ruminants such as cows, goats, sheep, and deer have 4 compartments including the rumen, omasum, abomasum, and the reticulum.

Your clinic has decided to start harvesting blood from volunteer donors to administer to patients. How long can whole blood units be stored in the refrigerator if properly collected? *A* 45 days *B* 21 days *C* 1 year *D* 3 months *E* 35 days

*E* 35 days Explanation - Whole blood units may be stored for 35 days if the unit is collected aseptically, stored appropriately and mixed daily

Which vaccination is most commonly given the intranasal route? *A* Periodontal *B* Distemper *C* Rabies *D* Parvo *E* Bordetella

*E* Bordetella Explanation - Bordetella bronchiseptica is a common cause of kennel cough. An intranasal vaccine against Bordetella is widely used. There is also an injectable vaccine available for Bordetella

Which of the following mites that can be found on dogs is not contagious? *A* Sarcoptes *B* Otodectes *C* Cheyletiella *D* Psoroptes *E* Demodex

*E* Demodex Explanation - The correct answer is Demodex. Demodex is a normal inhabitant of the skin and causes disease only when there is either a genetic predisposition or systemic disease that allows Demodex to develop into disease. Therefore it is not contagious in dogs. Demodex gatoi in cats is thought to be contagious amongst cats

Which of the following is not typically associated with pruritus? *A* Malassezia *B* Cheyletiella *C* Otodectes *D* Sarcoptes *E* Demodex *F* Microsporum canis

*E* Demodex Explanation - Demodex mites are typically not itchy. It is important to note however that with severe infections of demodex, secondary bacterial infections are common, and those can be itchy. Sarcoptes (scabies mites), Cheyletiella mites, Otodectes (ear mites), Malassezia (yeast), and Microsporum canis (ringworm) all cause pruritus

A vet at your clinic is preparing to perform a punch biopsy of a dermal mass in a cat under local anesthesia. He asks you to add epinephrine to his lidocaine before administering the local anesthetic. What is the rationale for adding epinephrine? *A* Epinephrine neutralizes the acidic pH of lidocaine, decreasing the pain on injection *B* Epinephrine acts as a pharmacologic antagonist to the systemic effects of lidocaine, to which cats can be particularly sensitive *C* Epinephrine, when given by this route, has a sedative effect on cats *D* Epinephrine allows the lidocaine to be more rapidly cleared *E* Epinephrine prolongs the duration of lidocaine's effects

*E* Epinephrine prolongs the duration of lidocaine's effects Explanation - Epinephrine is used with lidocaine to cause local vasoconstriction, preventing rapid systemic absorption. This is typically done to prolong the duration of local anesthetic effects of lidocaine by maintaining it at the site. It can decrease systemic uptake and toxicity, but it is not a lidocaine antagonist. Epinephrine does not neutralize the pH of lidocaine; sodium bicarbonate is sometimes added for this purpose

Periodontal disease can refer to processes affecting which of the following? *A* Dentin *B* Hard palate *C* Tongue *D* Enamel *E* Gingiva

*E* Gingiva Explanation - Periodontal disease is one of the most common conditions seen. The periodontum includes the gingiva, the alveolar bone, the periodontal ligament and cementum of the tooth. Gingivitis is usually the first sign of periodontal disease.

Which of the following fluid types is a colloid? *A* Hypertonic saline *B* Sodium chloride 0.9% *C* Lactated Ringers *D* 5% dextrose in saline *E* Hetastarch

*E* Hetastarch Explanation - A colloid contains larger molecules than a crystalloid. This size difference allows the fluid to stay in the intravascular space for longer and thus helps to increase oncotic pressure. Examples of colloids are Hetastarch, blood products (whole blood, plasma, etc.), and Dextrans

The adrenal glands are closest to what other structure? *A* Liver *B* Brain *C* Bladder *D* Pancreas *E* Kidneys

*E* Kidneys Explanation - The adrenal glands are located right above the kidneys. The bladder sits in the caudal abdomen ventral to the colon. The liver is just caudal to the diaphragm and is the most cranial organ in the abdomen. The right limb of the pancreas runs parallel with the duodenum while the left limb runs along the body of the stomach

You are helping the neurologist with a Tensilon test in a dog. Tensilon is the trade name for edrophonium chloride. By administering this drug, you are testing for which disease? *A* Hydrocephalus *B* Old dog vestibular disease *C* Diskospondylitis *D* Meningitis *E* Myasthenia gravis

*E* Myasthenia gravis Explanation - Myasthenia gravis can sometimes be diagnosed with a Tensilon test. With this disease, there are not many acetylcholine receptor sites on the muscles, and acetylcholine is broken down before it can fully cause muscle stimulus. This results in muscle weakness. By blocking the action of acetylcholinesterase, Tensilon prolongs stimulation to the muscles, thus temporarily improving strength. Results are usually seen within a minute of administering the drug IV. A blood test that can also help to aid in diagnosis of Myasthenia gravis is the acetylcholine receptor antibody test

A Dachshund presents for back problems and the dog is ambulatory but very unsteady in both hindlimbs. The doctor suspects an intervertebral disk protrusion that is affecting the spinal cord. Regarding the gait, this dog has: *A* Paralysis *B* Plegia *C* Vestibular syndrome *D* Hypermetria *E* Paresis

*E* Paresis Explanation - Paresis is partial loss or impaired movement; this is when motor is still present. A dog with paresis of both hindlimbs would have "paraparesis". Paralysis and plegia are terms used to describe lack of any voluntary motor. Hypermetria is an exaggerated gait caused by cerebellar disease. Pets with vestibular syndrome often have an abnormal gait and will veer toward one side or the other and have difficulty standing, but this is not a lack of motor

A 3-year old indoor/outdoor MN feline named Dudley presents to your clinic because the owner saw "some white rice-looking things" around his anus. They were small and flat and seemed to be moving. Some of them were dried up. Which of the following drugs is effective against this organism? *A* Selamectin (Revolution) *B* Fenbendazole (Panacur) *C* Fipronil (Frontline) *D* Pyrantel (Strongid) *E* Praziquantel (Droncit)

*E* Praziquantel (Droncit) Explanation - This cat likely has a tapeworm infection. Tapeworm segments are typically flat and white and small, resembling a grain of rice. The most common tapeworms in cats are Taenia taeniaeformis and Dipylidium caninum. The only medication that will treat both types is praziquantel. Praziquantel is in the products Drontal Plus and Profender, approved for the use in cats. Drontal Plus also contains pyrantel. Profender also contains emodepside. Both are also effective against roundworm and hookworm. Pyrantel is not effective against tapeworms and treats hookworm and roundworm infection. Fenbendazole (or Panacur), treats Taenia but not Dipylidium, and also treats hookworm, roundworm, and whipworm infection. Revolution treats and prevents hookworm, roundworm, heartworm, fleas, and ear mites in cats. Frontline treats and prevents fleas and ticks. A flea control should be recommended, since Dipylidium is transmitted by ingestion of an infected flea. Taenia is transmitted through eating an infected prey

Tularemia would most likely be carried by which animal? *A* Goats *B* Guinea pigs *C* Cows *D* Deer *E* Rabbits

*E* Rabbits Explanation - Tularemia is caused by the bacteria Francisella tularensis and is known as rabbit fever. It is zoonotic, and the bacteria can gain entrance through a cut or scratch when handling an infected animal. It leads to a skin ulcer and then swollen glands, fever, headache, and rash. It can also be transmitted to humans via tick bites or deerflies

Horses have unique structures that are paired diverticuli of the Eustachian tube located near the pharyngeal region of the horse. What is the common name of these structures? *A* The nasopharyngeal pouches *B* The dorsal pharyngeal recesses *C* The hyoid apparatus *D* The oropharyngeal openings *E* The guttural pouches

*E* The guttural pouches Explanation - The guttural pouches are blind pouches located in the pharynx of the horse. The function of these pouches is unknown but numerous vital cranial nerves and blood vessels course through these structures.

What does it mean if an animal is a paratenic host of a parasite? *A* The animal is another vertebrate that serves as a source of infection for humans or domestic animals *B* The animal is where the parasite develops from larval to nymph form *C* The parasite's adult or sexual stage lives on the animal *D* The animal is where the parasite develops from nymph to adult form *E* The parasite does not undergo development on the animal

*E* The parasite does not undergo development on the animal Explanation - A paratenic host is also known as a transport host. This indicates a type of intermediate host on which a parasite does not undergo development into the next stage. The definitive host harbors the adult, sexual stage of a parasite. The intermediate host harbors larval, asexual, or immature forms of a parasite. A reservoir host is another vertebrate host for a parasite that serves as a source of infection for people or domestic animals

Which of the following is another term for the type of parasite known as a fluke? *A* Cestode *B* Nematode *C* Ascarid *D* Protozoan *E* Trematode

*E* Trematode Explanation - Trematodes, or flukes, are leaf-shaped flatworms with unsegmented bodies. Adults are hermaphrodites. They primarily are found in the intestinal tract, liver, and lungs. Examples include the liver fluke of cattle (Fasciola hepatica) and the lungworm of cats (Paragonimus kellicotti). Nematodes are roundworms, ascarids are a type of nematode (roundworm), cestodes are tapeworms, and protozoans are single-celled organisms that may be parasitic

Four patients present to your emergency clinic at the same time: a) 2-year old male domestic short hair with a urethral obstruction b) 5-year old female Labrador with a tracheal foreign body c) 10-year old male standard poodle with a laceration on its forelimb d) 4-year old male hamster with elongated teeth In what order should these patients be triaged? *A* a, b, c, d *B* a, c, b, d *C* c, d, a, b *D* a, d, b, c *E* b, a, c, d

*E* b, a, c, d Explanation - A patient with an airway obstruction is in immediate danger of respiratory arrest and therefore takes priority over all other patients. A cat with urethral obstruction would be the next priority due to the possibility of a bladder rupture. A laceration needs care within a few hours. Elongated or overgrown teeth in a hamster, while needing treatment, would have the lowest priority in this group of emergency patients

Which of the following components or settings influence x-ray beam quality? *A* mAs and collimation *B* mAs and filtration *C* kVp and mAs *D* Collimation and filtration *E* kVp and collimation *F* kVp and filtration

*F* kVp and filtration Explanation - The "quality" of an x-ray beam refers to the energy distribution of the photons that are emitted. mAs and collimation have no effect on the energy of the emitted photons. mAs specifies the quantity of photons emitted; collimation constrains the area where the photons are emitted. The kVp is related to the peak energy of the photons emitted. Applied filtration preferentially absorbs lower-energy photons to affect beam quality.

A dog presents in congestive heart failure. You are asked to administer 2 mg/kg of furosemide IV. The concentration of furosemide is 50 mg/mL. The dog weighs 25 pounds. How many milliliters should you administer to this patient? A) 0.44 mL B) 0.12 mL C) 1.2 mL D) 2.0 mL

A) 0.44 mL Explanation - The dog is 25 pounds. 25 pounds/2.2 pounds per kg= 11.3 kg Now multiply the dog's weight in kg by the dose you need (2mg/kg): 11kg X 2mg = 22 mg The dog needs 22 mg. Now divide this dose by the concentration of furosemide (50 mg/mL): 22mg/ 50 = 0.44 mL

What is the average cloacal temperature of a chicken? A) 107F B) 98.8F C) 104F D) 102F

A) 107F Explanation - The deep body temperature of a chicken is around 107 degrees Fahrenheit

Which is the dental formula for a cat? A) 2 (I 3/3 C 1/1 P 3/2 M 1/1)= 30 B) 2 (I 3/3 C 1/1 P 4/4 M 2/3)= 42 C) 2 (I 3/2 C 2/2 P 3/3 M 0/1)= 32 D) 2 (I 2/2 C 1/1 P 3/3 M 2/1)= 30

A) 2 (I 3/3 C 1/1 P 3/2 M 1/1)= 30 Explanation - Cats have 3 incisors, 1 canine, 3 premolars, and 1 molar on one side of the upper jaw and 3 incisors, 1 canine, 2 premolars, and 1 molar on one side of the lower jaw so the dental formula for the cat is: 2 (I 3/3 C 1/1 P 3/2 M 1/1)= 30 permanent teeth The last choice is the dental formula for the dog (42 teeth)

What minimum quantity of feces is required to run a reliable fecal test for ova and parasites? A) 2 grams B) 5 grams C) 10 grams D) 0.5 grams

A) 2 grams Explanation - Two grams of feces, about the size of a garbanzo bean, is the smallest quantity of feces that would be acceptable to run a reliable fecal flotation with zinc centrifugation. Five grams is ideal

A 12-pound cat has presented with diabetic ketoacidosis and is currently in shock. The doctor orders that a 40 ml/kg bolus of 0.9% saline be administered. How many milliliters will this patient receive? A) 220 ml B) 9 ml C) 480 ml D) 36 ml

A) 220 ml Explanation - The body weight needs to be converted to kilograms. 12 lb / (2.2 lb/kg) = 5.5 kg 5.5 kg x (40 ml/kg) = 220 ml

The heart rate for an adult horse should generally be in which of the following ranges? A) 25-50 beats per minute B) 60-80 beats per minute C) 15-25 beats per minute D) 45-65 beats per minute

A) 25-50 beats per minute Explanation - Horses on average have a heart rate of around 35 beats per minute (with a range of 25 to 50 beats per minute

How many feline blood types exist? A) 3 B) 4 C) 2 D) 1 E) 13

A) 3 Explanation - There are three feline blood types: A, B, and AB. Group A is the most common, as 98% of cats are type A. Type B is more rare and seen most often in some exotic breeds. Only approximately 1.7% of cats have type B blood. AB is rare in all breeds. Always crossmatch cats unless you know their blood types. Dogs have 13 blood types

A 120-pound Great Dane has presented with Gastric Dilatation Volvulus. The clinician has asked you to quickly prepare the drug protocol. As a premedication, the dog is to receive 0.08 mg/kg of Hydromorphone. How many milligrams will you administer? A) 4.4mg B) 9.6 mg C) 2.2 mg D)6.4 mg

A) 4.4mg Explanation - To calculate milligrams administered the dog's body weight should be converted to kilograms. As a general rule, the majority of medications are dosed in a milligram-per-kilogram dosing. Therefore all body weights need to be converted to kilograms so that you are working in the same units. 120 lb/2.2 = 54.5 kg (There are 2.2 lbs per kilogram) 54.5 kg x (0.08 mg/kg) = 4.4 mg (Notice the "kg" will cancel out when you do the math) If your answer was 9.6 mg, you did not convert the body weight into kilograms.

What is the average toxic dose of theobromine in dogs? A) 60-100 mg/kg B) 150-200 mg/kg C) 10-15 mg/kg D) 0.5-2 mg/kg

A) 60-100 mg/kg Explanation - The LD50 toxic dose of theobromine in dogs which many texts list is 100-150 mg/kg, and may be the correct answer if listed as a choice. HOWEVER, this means that about 50% of pets die at this level; therefore, obviously the true toxic dose would be lower than this. In some sensitive pets, toxic doses could as low as 20 mg/kg. On average, the toxic dose is likely somewhere between 60-100 mg/kg

The heart rate of an adult dog should range between which of the following? A) 70-160 bpm B) 24-50 bpm C) 180-240 bpm D) 120-200 bpm

A) 70-160 bpm Explanation - A dog heart rate ranges between 70-160 bpm. As with the cat, there may be variation. An adult cat typically has a resting heart rate between 150-220 bpm. Keep in mind that if the cat is very excited, the heart rate might be faster. Remember that puppies and kittens will have faster heart rates

A veterinarian hands you a prescriptions that reads: Cephalexin 250 mg, SIG: 3 cap PO BID x 14d What is correct amount of medication to dispense and what directions should be typed on the drug label? A) 84 capsules (250 mg each) of cephalexin. Give 3 capsules by mouth twice daily for 14 days. B) 42 capsules (75 mg each) of cephalexin. Give 3 capsules by mouth once daily for 14 days. C) 42 capsules (250 mg each) of cephalexin. Give 3 capsules by mouth twice daily for 14 days. D) 84 capsules (75 mg each) of cephalexin. Give 3 capsules by mouth twice daily for 14 days. E) 136 capsules (250 mg each) of cephalexin. Give 3 capsules by mouth three times daily for 14 days.

A) 84 capsules (250 mg each) of cephalexin. Give 3 capsules by mouth twice daily for 14 days. Explanation - The signatura (SIG) or transcription is the directions for the medication. PO stands for "Per Os" which means orally. BID means twice daily. So the correct instructions are: Give 3 capsules by mouth twice daily for 14 days. If 3 capsules are given twice daily, then 6 are given total per day. 6 X 14 days= 84 capsules

What is an osteosarcoma? A) A malignant bone tumor B) A benign bone tumor C) A benign tumor of the skull sutures D) A malignant tumor of the spleen E) A benign tumor originating from cartilage

A) A malignant bone tumor Explanation - An osteosarcoma is a malignant bone tumor. Osteosarcoma is the most common bone tumor in dogs and has a 90% metastatic rate. An osteoma is a benign bone tumor. Tumors of the skull sutures are rare and do not have a specific name; a benign bone tumor of the skull would be a skull osteoma. There are various types of cancerous splenic tumors with hemangiosarcoma (a tumor of blood vessels) being the most common. A benign tumor of cartilage is a chondroma

Which of the following is the best description of an epulis? A) A non-malignant oral tumor B) A gingival lesion caused by trauma C) A gingival lesion caused by inflammation D) A developmental tooth disorder

A) A non-malignant oral tumor Explanation - An epulis is a non-malignant tumor of the periodontal ligament. These tumors do not metastasize but can be locally invasive. The most aggressive form of this condition, an acanthomatous epulis, can invade and destroy bone

Which ICU patient on fluids needs to be watched closely for jugular distention? A) A patient with a heart murmur B) An aggressive patient C) A patient with history of seizures D) A patient in renal failure

A) A patient with a heart murmur Explanation - A patient with underlying cardiac disease is at the highest risk of pulmonary edema (fluid in the lungs) from fluid overload. An indication of possible fluid overload is jugular distention. A good way to monitor hydration status of patients like this in the ICU is via central venous pressure readings

What is a pneumocystogram? A) A radiograph of the bladder taken after having had air injected into it B) Fluoroscopy of the kidney and bladder taken after intravenous injection of positive contrast that is excreted via the kidneys C) A radiograph of the bladder taken after intravenous injection of positive contrast that is excreted via the kidneys D) A radiograph of the bladder taken after direct injection of positive contrast

A) A radiograph of the bladder taken after having had air injected into it Explanation - A pneumocystogram is a radiograph of the bladder taken after insertion of a urinary catheter, draining of the urine, and injection of air (negative contrast) into the bladder via the catheter. This procedure allows the bladder wall to be visible and makes certain types of bladder stones more easily viewable. A radiograph of the bladder taken after direct injection of positive contrast is called a cystogram or a positive-contrast cystogram. A radiograph of the bladder taken after intravenous injection of positive contrast that is excreted via the kidneys is called an intravenous pyelogram (IVP) or excretory urogram

When is the appropriate time to immerse surgical instruments in surgical milk? A) After being cleaned in the ultrasonic cleaner B) Immediately after surgery C) After a preliminary rinsing of the instruments D) Just prior to placing them in the ultrasonic cleaner

A) After being cleaned in the ultrasonic cleaner Explanation - Surgical instruments should be placed in surgical milk after being cleaned in the ultrasonic cleaner. Surgical milk is used to help lubricate instruments and protect them from rust. Placing an instrument in an ultrasonic cleaner after being in the surgical milk will result in removal of the surgical milk. For highly specialized instruments such as drills, it is recommended that the technician follow the recommendations of the manufacturer for the best cleaning and sterilization practices.

Gas exchange occurs between air and blood by diffusion through which structure? A) Alveoli B) Trachea C) Pleura D) Bronchi

A) Alveoli Explanation - The alveoli are the site of gas exchange. They are tiny air sacs in the lungs that exchange carbon dioxide and oxygen and are located at the last branching of the respiratory tract. They are lined with simple squamous epithelium. The oxygen diffuses through the epithelium and the capillary endothelium in the exchange. The upper airway is lined mostly with pseudostratified columnar epithelium A rabbit presents with a history of sneezing, nasal discharge, and conjunctivitis. The veterinarian suspects the rabbit has a

What is commonly done to get a mare to cycle out of season? A) Artificial lighting B) Administration of oxytocin C) Manual manipulation of the ovaries via rectal palpation D) Keep a stallion in the same barn

A) Artificial lighting Explanation - Most breeders prefer for a foal to be born early in the year. This is because all horses are considered 1 year old on January 1 no matter when they are born. Many mares are not cycling at the right time for this to occur, so artificial lighting is used to trick the mare that it is the breeding season. While keeping a stallion in the barn may be of benefit, horses are seasonally polyestrous, so they cycle when there are longer days, typically March-September. 16 hours of artificial light mimics this time period. Oxytocin is the hormone responsible for milk letdown. Palpation of the ovaries does not cause cycling

Which of the following methods involves collecting nematode larvae by allowing them to pass through a wire net or cheesecloth? A) Baermann technique B) Wisconsin technique C) McMaster technique D) Sheather's solution technique

A) Baermann technique Explanation - The Baermann technique involves placing a fecal sample in a warm water or saline solution over a cheesecloth. This stimulates larvae to travel through the cloth toward the bottom of a funnel. This technique is useful in collecting motile larvae. The Wisconsin technique and McMaster technique are both quantitative fecal count procedures, and Sheather's solution is a sugar solution used for fecal flotation

Ticks can serve as vectors to transmit several diseases in animals including Lyme disease. What is the agent that causes Lyme disease? A) Borrelia burgdorferi B) Ixodes pacificus C) Rickettsia rickettsii D) Neorickettsia helminthoeca

A) Borrelia burgdorferi Explanation - Borrelia burgdorferi is the name of the rickettsial organism that causes Lyme disease. Ixodes is a genus of tick that may transmit the disease. Rickettsia rickettsii is the causative agent of Rocky Mountain spotted fever. Neorickettsia helminthoeca is the causative agent of salmon poisoning

Mineralized debris on the tooth surface is known as which of the following? A) Calculus B) Dentin C) Plaque D) Cementum

A) Calculus Explanation - Calculus is mineralization of plaque on the teeth. Plaque is a mixture of bacteria, saliva, and food parts that cover the tooth in a thin film. Plaque can turn into calculus in as early as a week. Dentin is the hard, white portion of the tooth. Cementum is calcified connective tissue

Which species regurgitates food for remastication? A) Cattle B) Lagomorphs C) Cats D) Pigs E) Horses

A) Cattle Explanation - Cattle chew their "cud", which is food that they regurgitate for remastication. When a cow chews her cud, she regurgitates a food bolus into her mouth, which is then re-chewed and re-swallowed. While cattle are chewing their cud, they produce saliva which acts as a natural antacid that helps to buffer and protect the rumen. Cows sometimes spend up to 8 hours a day chewing their cud

A 2-year old male neutered Great Dane presents on emergency with a distended abdomen and a history of unproductive retching. A catheter must be placed to provide immediate fluid therapy. What is the best site for catheterization? A) Cephalic vein B) Jugular vein C) Dorsal pedal artery D) Lateral saphenous vein E) Femoral artery

A) Cephalic vein Explanation - A catheter should be placed in the cephalic vein. A large breed dog with a distended abdomen and history of unproductive retching would be highly suspicious of gastric dilatation-volvulus (GDV). Patients with GDV have poor perfusion to the hind limbs; therefore a saphenous catheter would not be the best choice. If a cephalic catheter cannot be achieved, then a saphenous catheter is better than not having one at all. A jugular vein may be used; however, placing a catheter there would be more time consuming than placing a cephalic catheter. Arterial catheterization would not be recommended as this catheter could not be used for rapid infusion of fluids

Many factors can contribute to insulin resistance in pets. Which of the following can lead to insulin resistance and type II diabetes? A) Chronic use of corticosteroids B) Increased drinking and urinating C) Loss of acinar cells of the pancreas D) High-protein diet

A) Chronic use of corticosteroids Explanation - Type II diabetes occurs mostly due to insulin resistance and can occur from things such as chronic use of corticosteroids, obesity, or pancreatitis. Acinar cells secrete digestive enzymes. Beta cells of the pancreas actually make insulin, and degeneration of these cells can lead to type I diabetes. Increased drinking and urinating is a clinical symptom of diabetes but doesn't cause insulin resistance. A high-protein, low-carbohydrate diet actually helps with glycemic control

You are asked to run an APTT and PT test. What anticoagulant is used for this? A) Citrate B) No anticoagulant because it is a clotting test C) Coumadin D) EDTA E) Heparin

A) Citrate Explanation - Sodium citrate is the anticoagulant found in the turquoise (light blue) top tube used to run an Activated Partial Thromboplastin Time and a Prothrombin Time. Citrate is a reversible anticoagulant used for coagulation testing

What type of placenta does a ruminant have? A) Cotyledonary B) Discoid C) Zonary D) Diffuse

A) Cotyledonary Explanation - Ruminants (not including camels) have a cotyledonary placenta. This means they have numerous smaller placentae instead of a single large contact area between the mother and fetus. Ruminant placental cotyledons attach to a caruncle to form a placentome. The ruminant placenta is epitheliochorial (they have all three layers: endometrium, connective tissue, and uterine endothelium). The cotyledon is the fetus side of the placenta. The caruncle is the maternal side of the placenta. The placentome is the cotyledon and caruncle together. Dogs and cats have a zonary placenta. The placenta forms a band of tissue surrounding the fetus. They have an endotheliocorial placenta (only has the uterine endothelium). Primates and rodents have a discoid placenta (a single discoid shaped placenta). They have a hemochorial placenta (no maternal layers, but instead direct contact of maternal blood with the chorion). Horses and Pigs have a diffuse placenta. The placenta is formed by the whole allantochorion surface. They also have an epitheliochorial placenta like ruminants

Which syndrome can occur in dogs when long-term corticosteroids such as prednisone are administered? A) Cushing's B) Addison's C) Hypothyroidism D) Hyperthyroidism

A) Cushing's Explanation - Cushing's syndrome, or hyperadrenocorticism, occurs from high levels of cortisol. It can be due to over-production of cortisol by the adrenal glands or can be iatrogenic from long-term or chronic corticosteroid administration. Symptoms of this include a thinning haircoat, polyuria and polydipsia, pot belly, thinning skin, and calcinosis cutis. Long-term corticosteroid administration can also lead to insulin resistance, leading to type II diabetes in some patients

The current accepted standard image format for digital radiographs is called DICOM (digital imaging and communications in medicine). DICOM files have many advantages over other types of image files. Which of the following is NOT true of DICOM images? A) DICOM image files can be sent to other veterinarians or to owners and can be viewed in almost any photo viewer B) DICOM images are less easily altered by image editing software than other types of images (i.e. JPG, TIFF) and are therefore more appropriate for a medical record. C) DICOM image files allow for storage, query/retrieval, and manipulation of images D) DICOM image files contain information about the patient and the type of study performed E) DICOM image files are arranged into images, studies, and series where each study may consist of one or more series and each series may consist of one or more images

A) DICOM image files can be sent to other veterinarians or to owners and can be viewed in almost any photo viewer Explanation - DICOM image files require special viewing software (i.e. eFilm or OsiriX) and cannot be opened in most photo viewing or editing software (i.e. Photoshop). This makes it difficult for the images to be altered, which maintains their integrity as a medical document. DICOM files contain information about the patient and study performed, and they can be stored, retrieved, and manipulated. Files are arranged into images, studies, and series where each study may consist of one or more series, and each series may consist of one or more images

A drug that is an anti-cholinergic agent will have which of the following effects? A) Decrease salivation B) Suppress the cough reflex C) Constrict the pupil of the eye D) Decrease heart rate

A) Decrease salivation Explanation - Anti-cholinergic agents are drugs that antagonize the effects of acetylcholine in the autonomic nervous system. Commonly used examples of anticholinergic agents include atropine and pilocarpine. They are commonly used prior to anesthesia because they increase the heart rate (most anesthetic drugs decrease heart rate) and decrease oral secretions, which can aid intubation

Which parasite would be flat and segmented in appearance? A) Dipylidium B) Toxocara C) Trichuris D) Isospora

A) Dipylidium Explanation - The tapeworm of Dipylidium is usually 6 or more inches in length. It is a flat, segmented worm. It attaches to the intestine via the rostellum which has hooks on it. It has 6 rows of teeth. The segments from the tail end of the worm are dropped off and passed in the feces (look like rice granules). These segments contain the eggs which are then shed into the environment.

A dog in respiratory distress should not be placed in which position for radiographs? A) Dorsal B) Left lateral C) Sternal D) Right lateral

A) Dorsal Explanation - It is contraindicated to place an animal in respiratory distress on its back. Doing so makes it more difficult for the patient to fully expand their lungs

Which antibiotic choice is NOT bactericidal? A) Doxycycline B) Baytril C) Cephalexin D) Gentamicin

A) Doxycycline Explanation - Tetracyclines (including doxycycline) are bacteriostatic, which means these antibiotics do not kill bacteria. Bacteriostatic antibiotics act to inhibit bacterial growth. Tetracyclines, the macrolides (like erythromycin), chloramphenicol, and trimethoprim sulfa are all bacteriostatic. Bacteriostatic antibiotics can be sufficiently effective if the host's immune system is strong. The other choices listed are bactericidal (they kill bacteria).

Which of the following signs likely means that parturition is imminent? A) Drop in body temperature B) Pruritus of the abdomen C) Poor appetite D) Vocalization

A) Drop in body temperature Explanation - A drop in the normal body temperature by 1 or 2 degrees means that labor will likely start within 12-24 hours on average. It is important to evaluate this based on multiple temperatures taken at the same time each day over a period of time (i.e., don't take the temperature after the bitch has been outside using the bathroom in the heat and compare it to temperatures taken in the morning at rest, etc).

The pancreas connects to which part of the intestine? A) Duodenum B) Jejunum C) Cecum D) Ileum

A) Duodenum Explanation - The pancreas connects to the duodenum (the first segment of the small intestine). It secretes digestive enzymes into the duodenum via the pancreatic duct

Which describes the proper order of intestinal segments? A) Duodenum, jejunum, ileum, colon B) Ileum, duodenum, jejunum, colon C) Jejunum, duodenum, ileum, colon D) Jejunum, ileum, duodenum, colon

A) Duodenum, jejunum, ileum, colon Explanation - The proper order of the GI tract is: esophagus, stomach, duodenum, jejunum, ileum, cecum, colon. The duodenum, jejunum, and ileum make up the small intestine. The cecum is the pouch that connects the small intestine to the colon (the large intestine).

The tooth crown is covered with which of the following? A) Enamel B) Cementum C) Pulp D) Dentin

A) Enamel Explanation - Enamel is the hard mineralized surface of the tooth. It is made up of hydroxyapatite, or calcium phosphate. The tooth itself is made up mostly of dentin. The dentin is covered by the enamel on the crown and by cementum on the roots. The pulp is the center part of a tooth that is made up of living connective tissue and is the softest part containing nerve fibers

Which drug is a fluoroquinolone? A) Enrofloxacin B) Cephalexin C) Carprofen D) Clindamycin

A) Enrofloxacin Explanation - Fluoroquinolones include antibiotics such as enrofloxacin (Baytril), ciprofloxacin (Cipro), and marbofloxacin (Zeniquin). The fluoroquinolones are bactericidal. If possible, they should not be used in young, growing animals due to potential for cartilage disturbances. In some cases, they also have the potential side effect of causing blindness in cats at higher doses and should be used with caution

In which species is it common to observe Rouleaux formation in a blood smear? A) Equine B) Canine C) Avian D) Bovine

A) Equine Explanation - It is very common for horses to have blood cells arranged like a stack of coins (Rouleaux formation). This is not typically seen in the other species listed and may be indicative of an underlying inflammatory process in those species. Rouleaux formation may be a normal physiologic finding in horses, cats, and pigs. Of these, it is by far the most common in horses

Which of these cell types are all typically found in circulation? A) Erythrocytes, leukocytes, thrombocytes B) Leukocytes, thrombocytes, metamyelocytes C) Rubriblasts, thrombocytes, erythrocytes D) Lymphoblasts, erythrocytes, leukocytes

A) Erythrocytes, leukocytes, thrombocytes Explanation - The cellular makeup of the blood includes erythrocytes (red blood cells), leukocytes (white blood cells), and thrombocytes (platelets)

Administration of which antimicrobial is MOST LIKELY to result in diarrhea in the adult horse? A) Erythromycin B) Ceftiofur C) Enrofloxacin D) Penicillin E) Gentacin

A) Erythromycin Explanation - Many antimicrobials can result in diarrhea in horses because of disruption of the normal intestinal flora. Of the drugs listed, erythromycin has the highest likelihood of contributing to the development of diarrhea. Erythromycin is administered to foals with Rhodococcus equi pneumonia but is rarely used in adult horses because of its common association with the development of diarrhea. All the other listed drugs can be administered to horses

What is the most likely period of time during the estrous cycle that a female will be receptive to mating? A) Estrus B) Anestrus C) Proestrus D) Diestrus

A) Estrus Explanation - Estrus is the main period of sexual receptivity. At this point, the uterus and uterine horns are primed for receiving an embryo. Proestrus is the period just prior to estrus and is under the influence of a hormone call follicular stimulating hormone (FSH). During this period FSH stimulates release of estrogen. During diestrus the corpus luteum (ruptured follicle) secretes hormones. If pregnancy does not occur, the corpus luteum will degenerate and stop secreting hormones. Anestrus is the period in which there is sexual inactivity

Why should atropine, a competitive antagonist of acetylcholine receptors, be used cautiously in horses (i.e. what is a major side effect of the drug)? A) Excessive use can result in intestinal stasis (colic) B) Excessive use can result in bradycardia C) Excessive use can result in diarrhea D) Excessive use can result in intense constriction of the pupil

A) Excessive use can result in intestinal stasis (colic) Explanation - Atropine is commonly used topically to dilate the pupil in horses with uveitis; with excessive administration, atropine can cause intestinal motility to decrease, possibly resulting in abdominal pain (colic) in the horse. Atropine is also used to increase the heart rate during general anesthesia

Which of the following species is an induced ovulator? A) Ferret B) Llama C) Mouse D) Pig

A) Ferret Explanation - Induced ovulator means that the animal does not ovulate until the act of breeding occurs. Cats and Ferrets are the most common induced ovulators.

What procedure performed in horses includes filing and smoothing their teeth? A) Floating B) Quidding C) Cribbing D) Raking

A) Floating Explanation - Horses' teeth grow throughout their lives. When a horse eats normally, grinding food between the back teeth, the abrasives wear the teeth down to keep them even and smooth. However, when a horse eats a softer diet like one containing a lot of alfalfa and grains, the teeth do not wear evenly and can form sharp, painful points. A horse with overgrown or sharp teeth may have difficulty eating, may hypersalivate, may start dropping food from its mouth, and can begin losing weight. Floating entails filing down the teeth using a dental rasp, or float, to make the teeth even and smooth again. Cribbing is the act of swallowing air or "wind sucking" and is thought to be behavioral. Sometimes cribbers will chew or suck on fences or other fixed objects so they can actually draw in air. Quidding is when a horse stores a bolus of food inside its mouth and spits it out or drops food from its mouth. The most common cause is bad teeth; the horse may need to have its teeth floated

The owner of an 8-year old German Shepherd dog calls to report that the dog may have ingested an unknown quantity of rat bait 12 hours ago. What would be the best course of action to advise over the phone? A) Have the owner bring the dog to the clinic immediately B) The new formulations of rat bait on the market are pet-friendly and no action needs to be taken C) Take a message and have the veterinarian call the owner back D) Have the owner monitor the dog's mucus membranes and bring the dog to the clinic if the mucus membranes become pale E) Have the owner induce vomiting at home

A) Have the owner bring the dog to the clinic immediately Explanation - Rat bait can cause prolonged clotting times or cerebral edema, and the dog should be seen immediately. It may not be advisable to induce vomiting without a veterinarian examination. Signs such as poor mucus membrane color only develop after a coagulopathy has developed, and by that time, intervention will be much less effective. The veterinarian won't be able to provide any treatment for rat bait ingestion over the phone. Rat bait is never pet-friendly

Purkinje fibers are found in which of the following organs? A) Heart B) Bladder C) Pancreas D) Stomach E) Duodenum

A) Heart Explanation - Purkinje fibers are conductive fibers within ventricular walls. They relay cardiac impulses to ventricular cells, which allow contraction.

Which setting is better for an x-ray of a part of the body with low contrast between tissues, like the abdomen? A) High kilovoltage (kVp); Low milliamp-seconds (mAs) B) Low kilovoltage (kVp); High milliamp-seconds (mAs) C) High kilovoltage (kVp); High milliamp-seconds (mAs) D) Low kilovoltage (kVp); Low milliamp-seconds (mAs)

A) High kilovoltage (kVp); Low milliamp-seconds (mAs) For soft tissues (like the abdomen) use a higher kVp and lower mAs settings. For bone, which has a lot more natural contrast because it is so hard, use a lower kVp and a higher mAs. Kilovoltage (kVp) controls the penetrating ability of an x-ray. Generally, increasing kVp increases film blackness and contrast. Usually, the higher the kVp, the higher the scale of contrast between tissues, and the better the quality of the image, because small differences in soft tissue density become more visible (more contrast-y).

The veterinarian is performing an orthopedic exam and says a dog has positive Ortolani sign. What corresponds with this? A) Hip dysplasia B) Cranial cruciate rupture C) Cervical instability D) Luxating patella

A) Hip dysplasia Explanation - Ortolani is the palpable sensation of gliding the femoral head in and out of the acetabulum and suggests joint laxity, most often seen in hip dysplasia. A "drawer sign" would correspond with a cranial cruciate rupture

Which species is unable to vomit? A) Horses B) Birds C) Pigs D) Cows

A) Horses Explanation - Horses are unable to empty their stomach by vomiting. Passing a nasogastric tube is important in cases of colic to prevent the stomach from filling or rupturing. Placing a nasogastric tube: The tube is placed in the ventral meatus of the nasal passage and gently advanced through the nasopharynx. The neck should be flexed to help the horse swallow the tube and prevent advancement of the tube into the trachea. Sucking back on the tube and getting negative pressure helps to confirm that the tube is in the esophagus

Which of the following is not a common cause of dystocia? A) Hypokalemia B) Decreased pelvic diameter C) Oversize fetus D) Uterine inertia

A) Hypokalemia Explanation - Hypokalemia is not considered a cause of dystocia. Hypocalcemia, on the other hand, can result in decreased ability to contract and will be a cause for dystocia. Oxytocin release will lead to contraction and depletion of calcium stores; inadequate release of oxytocin may result in uterine inertia. An oversized fetus, as is common in small breed dogs, may necessitate a planned cesarean delivery. Animals that have a decreased pelvic diameter will also be at risk of developing dystocia. A common cause of decreased pelvic diameter is previous trauma

You pull a 6 year old film out of a folder to compare to radiographs that you took today. The old film appears to have yellow-brown staining. What is the likely cause of this? A) Incomplete washing of the film B) Film was left in developer too long C) Storing the film at a hot temperature D) Incomplete drying of the film

A) Incomplete washing of the film Explanation - Yellow-brown staining over time is the result of incomplete removal of chemicals (usually from the fixer) during the final washing step in film development. A film that is not dried completely commonly has artifacts around the edges of the film

A dog has a high fever, is lethargic, and is diagnosed with peritonitis. What does this dog have? A) Inflammation of the internal abdominal lining B) Fluid in the pericardial sac around the heart C) A ruptured bladder D) Inflammation of the pleura

A) Inflammation of the internal abdominal lining Explanation - The peritoneum is a membrane that lines the abdominal cavity. It also covers most of the abdominal organs. The peritoneum helps support the organs and helps to facilitate blood and lymph flow to these organs. Peritonitis is inflammation ("itis") of the peritoneum. The most common causes of peritonitis include infection (such as a leak in the GI tract or a ruptured gall bladder). In cats, FIP causes a viral peritonitis. Fluid in the pericardial sac is termed pericardial effusion. The pleura is the tissue that protects the lungs

Which of the following correctly defines the term stomatitis? A) Inflammation of the oral mucosal surfaces B) Inflammation of the periodontal ligament and alveolar bone C) Inflammation of the tongue D) Inflammation of the region housing the tonsils

A) Inflammation of the oral mucosal surfaces Explanation - Stomatitis describes inflammation of the oral mucosal surfaces. It can be subdivided by region, such as buccal stomatitis (inflammation of the cheek mucosa). Inflammation of the tongue is known as glossitis. Inflammation of the glossopalatine folds around the area housing the tonsil is known as faucitis. Inflammation of the periodontal ligament and alveolar bone fall under the category of periodontitis

A 6-month old puppy has presented to the hospital with a history of severe straining to defecate. A fecal float identifies a severe parasitic infection. An abdominal ultrasound was performed, and the doctor identified that one of the loops of intestine was telescoping into another segment of intestine. What is this called? A) Intussusception B) Hydrometra C) Evisceration D) Strangulation

A) Intussusception Explanation - This question is describing an envagination of intestine into an adjacent segment of intestine. The part which prolapses into the other is called the intussusceptum, and the part which receives it is called the intussuscipiens. Evisceration refers to the removal of organs from body, usually from the abdomen. A hydrometra is the word used to describe fluid in the uterus. If the uterus were filled with pus, it would be referred to as a pyometra. Strangulation of a loop of intestine results in a point of constriction which then impedes circulation.

Which drug choice is a dissociative anesthetic and used for anesthetic induction and analgesia in veterinary medicine? A) Ketamine B) Propofol C) Xylazine D) Acepromazine

A) Ketamine Explanation - Ketamine is a dissociative anesthetic often used for induction of anesthesia. It is an NMDA receptor antagonist and also binds to opioid receptors, which provides analgesic effect. It is often given simultaneously with diazepam for anesthetic inductions.

The term pediculosis indicates a problem with which of the following? A) Lice B) Flies C) Worms D) Mites E) Fleas

A) Lice Explanation - Pediculosis refers to infestation with lice. It does not differentiate between biting and sucking lice. Myiasis is the term for infestation with fly larvae

Which of the following is NOT part of the axial skeleton? A) Limbs B) Skull C) Spine D) Ribs

A) Limbs Explanation - The axial skeleton forms the core body structure. The appendicular skeleton includes the appendages, or limbs

Where would you collect cerebrospinal fluid (CSF) from a standing sedated horse? A) Lumbosacral space B) Atlantooccipital space C) Temporohyoid space D) Cervical-thoracic space

A) Lumbosacral space Explanation - There are two sites that are routinely used to collect CSF from horses: the lumbosacral space and the atlantooccipital space. In the standing horse, the lumbosacral space is the only space that can be used. CSF is collected from the atlantooccipital space only in the anesthetized horse

Which of the following components of a digital radiography unit has the biggest influence on image quality? A) Monitor B) Server processor C) Workstation graphics card D) Workstation processor

A) Monitor Explanation - A PACS system consists of a server, a workstation, and a viewing monitor. The workstation receives the images and sends them for display on the monitor. The monitor is the most critical to image quality. The other components have a bigger influence on how quickly images may load. A high quality graphics card is more important for viewing movie loops of ultrasound or fluoroscopy images, which require more processing power but are inherently lower resolution

Which of the following is the most common factor that prevents a diagnostic radiograph in a horse? A) Motion B) Low line voltage C) Oversedation D) Bandages

A) Motion Explanation - Motion is a problem with any radiograph on any species. However, in horses, due to their fight or flight nature, getting them to stand still can be tricky

A 14-year old DSH has been prescribed total parenteral nutrition (TPN). What type of catheter would be most appropriate for TPN administration? A) Multi-lumen jugular catheter B) Single lumen PICC line in the hind leg C) 18 gauge saphenous catheter D) A catheter is not needed; TPN can be delivered subcutaneously E) 18 gauge cephalic catheter

A) Multi-lumen jugular catheter Explanation - TPN must be administered via a central vein due to its hypertonicity. A short saphenous or cephalic catheter would not reach a central vein. TPN is never administered subcutaneously. While a single lumen PICC could be used, a multi-lumen catheter would be preferred as blood samples for blood glucose monitoring could be more easily obtained

Dirofilaria immitis falls under which category of parasite? A) Nematode B) Protozoan C) Coccidian D) Trematode E) Cestode

A) Nematode Explanation - Dirofilaria immitis is the heartworm and is a nematode (a type of roundworm). The mosquito is the intermediate host. Trematodes are flukes and require a snail as an intermediate host. Protozoans are single celled organisms (microscopic). They may take on a cyst form or may be in trophozoite form (such as Giardia). Coccidians (such as Isospora) are a type of protozoan. Cestodes are tapeworms. Dipylidium, a type of tapeworm, requires a flea as an intermediate host

What is the medical term for declawing? A) Onychectomy B) Ovariohysterectomy C) Caudectomy D) Orchiectomy E) Enucleation

A) Onychectomy Explanation - The procedure consist of surgically removing or amputating the third phalanx of each toe. The majority of practitioners who perform this controversial operation will only do the procedure on the front paws. Caudectomy is tail amputation. Orchiectomy is removal of the testicles. Ovariohysterectomy is surgical removal of the uterus and ovaries. This sterilization procedure prevents pregnancy and is called a spay. Enucleation is surgical removal of the eye

Which of the following is a general term for giving birth that can be applied to all species? A) Parturition B) Pyometra C) Whelping D) Queening

A) Parturition Explanation - Parturition is the act of giving birth. Pyometra is a life-threatening uterine infection. Whelping is the term for giving birth in dogs. Queening is the term for giving birth in cats

Which species other than cattle is considered cloven hoofed? A) Pigs B) Camel C) Horse D) Rabbit

A) Pigs Explanation - A cloven hoof is a hoof split into two toes. This includes cattle, sheep, goats, deer, and pigs. Camels have two digits in their feet (III and IV) which are almost flat on the ground as part of a wide pad. Instead of hooves, the distal phalanges have nails on the upper surface.

An owner presents her diabetic cat and new glucometer with lancets. Where is the best place for her to collect a sample? A) Pinna B) Medial hindlimb C) Dorsal frontlimb D) Pad

A) Pinna Explanation - Lancets are best to use on superficial smaller veins such as veins on the ear. Pads are very thick and therefore difficult to use for sample collection

Which blood cells play an important role in clotting? A) Platelets B) Erythrocytes C) Leukocytes D) Metamyelocytes

A) Platelets Explanation - When a wound occurs, platelets act with fibrin to form a clot and stop bleeding. A metamyelocyte is a very immature white blood cell (that will eventually become a band) and is typically only seen in the bone marrow.

Periodontal pockets are measured by which dental instrument? A) Probe B) Scaler C) Curet D) Elevator

A) Probe Explanation - The probe is used to check pocket depth. The scaler is used to remove plaque and calculus from the teeth. In dentistry, the curet is a spoon-shaped instrument used for removing necrotic cementum and calculus from the teeth and can be used supragingival and subgingival. The elevator is used to wear down the periodontal ligament to aid in extracting teeth from the oral cavity

A female dog has some vaginal bleeding, swollen vulva, and attracts males but will not allow them to mount. What stage of the estrus cycle is she in? A) Proestrus B) Diestrus C) Estrus D) Anestrus

A) Proestrus Explanation - The correct answer is proestrus. During this phase, there is vulvar swelling, vaginal bleeding, and attraction of males but no mounting is allowed. When the female will stand to be mounted, she is in estrus

Which of these sedative/anesthetic drugs does not have an antagonist? A) Propofol B) Diazepam C) Hydromorphone D) Xylazine E) Medetomidine

A) Propofol Explanation - The correct answer is propofol. The reversal agents are: For diazepam: flumazenil For hydromorphone: naloxone or naltrexone For medetomidine: atipamezole For xylazine: yohimbine Acepromazine is another sedative that does not have an antagonist

Which anesthetic is short-acting, white-opaque in color, and can cause transient apnea on induction? A) Propofol B) Etomidate C) Thiopental D) Ketamine

A) Propofol Explanation - Propofol is an opaque-white short-acting anesthetic or sedative for intravenous use and is often used for anesthetic induction or sedation for short procedures. It can cause respiratory depression or transient apnea, especially when given too quickly.

Culture or draining lesions of which bacteria gives a characteristic sweet smell? A) Pseudomonas B) Nocardia C) E. coli D) Staphylococcus E) Campylobacter

A) Pseudomonas Explanation - Pseudomonas aeruginosa is a gram-negative bacterium that often has a sweet or fruity smell

A parrot presents because it is bleeding profusely from one of its feathers. What would likely be the best course of action to take after examination by the veterinarian? A) Pull the feather out B) Place a tight wrap around the feather shaft C) Do not manipulate the feather and allow it to clot on its own D) Cauterize the feather to stop bleeding

A) Pull the feather out Explanation - Blood feathers are immature feathers that still have a blood supply to the shaft. The shaft usually appears dark as opposed to white or clear as would be seen in a mature feather. If one is cut or breaks from trauma, it can bleed profusely and not clot. The bird can actually die of blood loss in some cases. Typically, the best course of action is to pull the blood feather out. This is typically done using a hemostat and giving a firm pull

A dog with bradycardia and a low albumin on a high rate of IV fluids is at risk for developing which of the following? A) Pulmonary edema B) Seizures C) Thromboembolism D) Syncope

A) Pulmonary edema Explanation - Albumin provides oncotic pressure which helps maintain fluid in the vasculature. If the oncotic pressure is decreased from low albumin and movement of blood through the body is slow due to a low heart rate (bradycardia), fluid can build up in the lungs (pulmonary edema

A urinalysis should ideally be performed within 30 minutes of collection. How should urine be stored prior to evaluation if it cannot be evaluated immediately? A) Refrigerate B) At room temperature C) In a 98 degree F water bath D) Freeze

A) Refrigerate Explanation - Urine should be stored in a refrigerator for up to 12 hours if necessary. Before evaluation the urine should be allowed to return to room temperature as this may otherwise affect the specific gravity. Freezing may destroy some cellular elements present but is okay to do if only evaluating chemical analytes

Which of the following findings in a urine sample is associated with renal tubular damage or disease? A) Renal casts B) Red blood cells C) Bilirubin D) White blood cells

A) Renal casts Explanation - Renal casts are suggestive of renal disease. There are several different types of casts including hyaline, cellular, and granular casts. The presence of a few hyaline casts may be a normal finding, but a large number of these should make the interpreter suspicious. Cellular casts can contain epithelium from the renal epithelium and are seen with acute tubular degeneration. Granular casts in large numbers may be indicative of acute renal disease. These contain fragments of cells or plasma proteins.

Where is the sinoatrial node located? A) Right atrium B) Left atrium C) Left ventricle D) Ventricular septum

A) Right atrium Explanation - The sinoatrial node provides automaticity to the heart and is located on the right atrium. The impulse then travels to the atrioventricular node which then conducts the impulse down the Bundle of His to the Purkinje fibers. This pattern of depolarization results in a very coordinated contraction of the heart, allowing for smooth blood flow.

Which breed should not receive ivermectin for treating Demodex? A) Sheltie B) Chihuahua C) Boxer D) Pit Bull

A) Sheltie Explanation - Ivermectin should not be given to collie-type breeds due to a possible MDR1 gene mutation that could cause the drug to be toxic. Breeds included for possible ivermectin toxicity include Collies, Shelties, and Australian Shepherds. The saying is "white feet, don't treat".

respiratory compromise What is the metallic element present on radiograph film that turns black when exposed and run through the radiograph development process? A) Silver B) Tungsten C) Gold D) Lead E) Zinc F) Magnesium

A) Silver Explanation - Films go into developer first which reduces exposed silver halide into elemental metallic silver which is black. Crystals that were unexposed do not undergo this process. Fixer then coverts the unexposed crystals into a soluble form so that they dissolve away, leaving a clear image where the film was unexposed

What organism is verminous arteritis associated with in the horse? A) Strongylus vulgaris B) Salmonella C) Cryptosporidium D) Parascaris equorum

A) Strongylus vulgaris Explanation - S. vulgaris is a large strongyle that causes arteritis and thrombosis of the vessels of the GI tract in the horse and eventually can result in segmental ischemia or infarction of the bowel wall. This, in turn, results in signs of colic in the horse. Fortunately, with the advent of modern anthelmintics, this form of colic is not common

Which of the following is true of a dog with polycythemia? A) The dog has an increased hematocrit B) The dog has a low platelet count C) The dog has an increased white blood cell count D) The dog has a low red blood cell count

A) The dog has an increased hematocrit Explanation - Polycythemia is an increase in the absolute number of circulating red blood cells (which results in an increased hematocrit or PCV). This is not the same as an increased hematocrit due to hemoconcentration from dehydration. A low red blood cell count is anemia. A low platelet count is thrombocytopenia. An increased white blood cell count is leukocytosis

What is the purpose of the condenser of a microscope? A) To focus light on the object being viewed B) To adjust the intensity of light reaching the object being viewed C) To filter out wavelengths of light before they reach the object being viewed D) To focus the lens on the object being viewed

A) To focus light on the object being viewed Explanation - The condenser of a microscope consists of two lenses that focus light on the object. It is adjusted by being raised or lowered below the stage. It may have a diaphragm that can control the amount of light but this is not the main purpose of the condenser. The rheostat of the light source is the best way to adjust light intensity.

A 7-year old Saint Bernard dog has a lesion of the left distal radius that is highly suspicious for a bone tumor (osteosarcoma). Prior to surgery, a bone scan is ordered and performed. What is the main reason for this study? A) To identify possible additional bone lesions (other than the left radius lesion) that may represent bony metastases B) To provide greater anatomic detail regarding the lesion in the left distal radius C) To confirm the diagnosis of osteosarcoma of the left distal radius D) To identify possible pulmonary metastatic lesions

A) To identify possible additional bone lesions (other than the left radius lesion) that may represent bony metastases Explanation - While the lungs are the most common site of metastasis of bone tumors in dogs, these tumors can also metastasize to other bony sites. Thoracic radiographs are generally sufficient to assess gross pulmonary metastasis but radiography is not typically practical for assessing all of the bones in the body. A bone scan is a nuclear medicine test that involves administration of a bone-seeking radioisotope (Technetium phosphate) that is taken up in areas of bone turnover. The test does not provide significant anatomic detail and is not confirmatory of osteosarcoma because it is not specific for cancer (osteoarthritis or osteomyelitis also are positive on bone scans). The test is used to identify additional bony sites that should be evaluated radiographically to assess for metastasis. Because this type of test is not routinely available at all practices and the overall incidence of bony metastasis is relatively low, it is usually reserved for cases where there is greater suspicion for bone metastasis. It would also be used to rule-out the presence of bone metastasis prior to performing complex or expensive limb-sparing surgeries

Schnauzers are notorious for having hyperlipidemia, and when their blood is spun down, the serum often appears milky. This is due to a high level of what in the serum? A) Triglycerides B) Chyme C) Leukocytes D) Lymph

A) Triglycerides Explanation - Hyperlipidemia is the term for elevated triglycerides or cholesterol

Which of the following are triple rooted teeth in dogs? A) Upper 4th premolars B) Lower 2nd premolars C) Upper 1st premolars D) Lower 1st molars

A) Upper 4th premolars Explanation - In the dog, the upper 4th premolars (also known as carnassial teeth), have three roots. The upper 1st and 2nd molars also have 3 roots. A dog has 3 incisors, 1 canine, 4 premolars and 2 molars on one side of the upper jaw and 3 incisors, 1 canine, 4 premolars and 3 molars on one side of the lower jaw, so the dental formula for the dog is: 2 (I 3/3 C 1/1 P 4/4 M 2/3)= 42

What is the dental formula for the adult horse (i.e. incisor-canine-premolar-molar)? A) Upper: 3-1-4-3 Lower: 3-1-3-3 B) Upper: 4-1-2-3 Lower: 4-1-2-3 C) Upper: 4-1-3-3 Lower: 3-1-3-3 D) Upper: 3-1-2-3 Lower: 3-1-2-3

A) Upper: 3-1-4-3 Lower: 3-1-3-3 Explanation - The upper quadrant of the horse's dental arcade has 3 incisors, 1 canine (primarily male horses), 4 premolars (including "wolf" tooth) and 3 molars; the lower arcade has 3 incisions, 1 canine, 3 premolars and 3 molars. Not all horses have the 1st upper premolar, the so-called wolf tooth. Generally, females do not have canine teeth

Which of the following may be a cause of post-renal azotemia? A) Urethral obstruction B) Primary loss of nephrons C) Severe dehydration D) Infection in the kidney

A) Urethral obstruction Explanation - Post-renal azotemia is an elevation in BUN and creatinine due to pathology located after the kidney (in the ureters, bladder, or urethra). A urethral obstruction means that the outflow from the bladder is obstructed. This could be from severe inflammation, uroliths, or neoplasia. This puts ascending pressure on the kidneys because the urine has no way of being voided. This may cause an elevation of renal values (BUN and creatinine) and is referred to as azotemia. Treatment of post-renal azotemia depends on relieving the obstruction and subsequent IV fluid diuresis. Pre-renal causes of azotemia include dehydration and lack of perfusion (as could occur in shock from hypotension). Renal azotemia is caused by a primary problem with the kidneys (acute or chronic renal failure

An incision made from xyphoid to pubis would be found where? A) Ventral midline B) Ventral cervical region C) Over the dorsal hip region D) Ventral chest over the sternum

A) Ventral midline Explanation - The xyphoid is the lowest part of the sternum. The pubis is one of the bones that makes up the hip. An incision from xyphoid to pubis implies a ventral midline incision as is customarily performed during abdominal exploratories.

A patient has marked pleural effusion, and there is concern that it may be due to heart failure. Which of the following radiographic views would allow the best visualization of the heart? A) Ventrodorsal B) Lateral oblique C) Medial oblique D) Dorso-ventral

A) Ventrodorsal Explanation - Oblique radiographs are not typically indicated in thoracic radiography. When a patient is positioned for a dorsoventral projection, fluid will accumulate in the dependent region and will frequently obscure the view of the heart. When a patient is positioned in ventral recumbency for a ventrodorsal view, the fluid will accumulate away from the heart, improving the ability to visualize the cardiac silhouette. NOTE: Although this is not what the question asked, if a patient is not stable and has significant respiratory compromise due to pleural effusion, being held in ventral recumbency may cause stress to the patient and increase breathing labor, potentially resulting in respiratory compromise

You are on a house call with the veterinarian to evaluate some goats that are having "night blindness" per the owner. Upon arrival you can see that the goats have a dull haircoat, are very thin, and have nasal discharge. What deficiency can cause these symptoms? A) Vitamin A B) Iodine C) Vitamin E D) Zinc

A) Vitamin A Explanation - Vitamin A deficiency causes a rough dry haircoat, thick nasal discharge, diarrhea, and can also lead to night blindness. Vitamin E can cause white muscle disease (a nutritional muscular dystrophy). Zinc deficiency can cause hypersalivation, chronic skin problems, testicular hypoplasia, and deformed hooves. Iodine deficiency most often causes a goiter

What is a symptom of myasthenia gravis? A) Weakness and muscle fatigue B) Muscle rigidity C) Hyperesthesia to touch D) Papules on the dermis

A) Weakness and muscle fatigue Explanation - In Myasthenia gravis, there are few acetylcholine receptor sites (AChR) on the muscles, and acetylcholine is broken down before it can fully cause muscle stimulus. This results in muscle weakness, and with this condition, animals experience severe muscle fatigue

How quickly after birth does a lamb need colostrum to be immunologically protected? A) Within first hour B) Within 15 minutes C) Within 24 hours D) Within 8 hours

A) Within first hour Explanation - Within the first hour after birth, a lamb must receive 6 to 8 ounces of colostrum to be protected

How many upper incisors does a goat have? A) Zero B) Four C) Two D) Six E) One

A) Zero Explanation - Goats have no upper front teeth (incisors); their upper front mouth is one big gumline. In the back on their mouth they do have both upper and lower teeth. Goats are born with teeth which are replaced by permanent teeth as they age. The dental formula for permanent teeth in the goat is: 2 ( I 0/3 C 0/1 P 3/3 M 3/3) = 32

Electrolytes (potassium, sodium, chloride) are measured in what units? A) mEq/L B) mg/dL C) mmol D) g/dL

A) mEq/L Explanation - Electrolytes (potassium, sodium, chloride) are measured in milliequivalents per liter (mEq/L)

Regarding lice parasites, the biting louse found in canines is known as which of the following? A)Trichodectes B) Hematopinus C) Damalinia D) Felicola

A)Trichodectes Explanation - Trichodectes canis is the chewing or biting louse in dogs. Hematopinus asini is the sucking louse in cattle. Damalinia bovis is the chewing louse in cattle (also called Bovicola). Felicola subrostratus is the chewing or biting louse in cats. Lice are ectoparasites; infestation with lice is known as pediculosis. Trichodectes canis can also serve as an intermediate host for Dipylidium caninum (although more commonly, the intermediate host is a flea

Icterus is a term that means which of the following? A)Yellow discoloration of the skin, eyes, and mucus membranes B) Straining to urinate C) Blood in the stool D) Small hemorrhages on the mucus membranes

A)Yellow discoloration of the skin, eyes, and mucus membranes Explanation - Icterus, or jaundice, means yellow discoloration of the skin and sclera. It is most often associated with liver disease from a poor flow of bile or biliary obstruction in the liver. It can also be caused by hepatitis or from red blood cell destruction. Small hemorrhages on the mucus membranes are petechiae. Straining to urinate is termed stranguria. Blood in the stool is hematochezia (bright red blood suggests large intestine bleeding) or melena (black/digested blood suggests small intestine bleeding

What is the normal amount of gastric reflux obtained from a healthy horse via nasogastric tube placement? A) 10-12 liters B) 1-3 liters C) 14-18 liters D) 5-8 liters

B) 1-3 liters Explanation - Normally, a healthy horse will have a small amount of gastric reflux, 1-3 liters. If you get back 8-12 or more liters of reflux, the horse likely has an obstructive intestinal disease or ileus of some sort

Which of the following is an abnormal finding in an intact male dog's urine sediment sample performed from a free catch? A) Fat droplets B) 10-15 WBC per high-powered field C) Bacteria D) Sperm

B) 10-15 WBC per high-powered field Explanation - The only abnormal finding in this sample is the excessive number of white blood cells present. Bacteria are not unexpected, as the sample obtained is a free catch, which is not a sterile collection method. Sperm can be seen in intact males. Fat droplets are not an abnormal finding in urine sediment

A dog refuses a rectal temperature. You take his temperature under the axillary region and get a reading of 99.5F. What is likely his core body temperature? A) 102.1F B) 101.5F C) 98.7F D) 99.5F

B) 101.5F Explanation - Typically a temperature taken in the ear or under a peripheral limb reads 1 to 2 degrees lower than rectal temperature, or core body temperature

An 11-year old intact male Golden Retriever presents with respiratory distress and a temperature of 106.7 F. The attending veterinarian has gone to talk to the client and has requested that the patient be cooled. To what temperature should the dog be cooled? A) 104.0 B) 103.0 C) 99.0 D) 102.0 E) 100.0

B) 103.0 Explanation - A patient will continue to cool despite cessation of active cooling, so a patient should not be cooled past the high end of the normal temperature range. Normal temperature range is 101-102.5F in dogs and cats

The gestation period for hamsters makes them a valuable research model. What is their gestation period? A) 56 days B) 15 days C) 21 days D) 30 days

B) 15 days Explanation - Hamsters have the shortest gestation period of all the rodents. Their rapid reproduction makes them valuable in the biotech field, saving on cost. In addition, it allows quick results for any reproductive study.

What is the average gestation for goats and sheep? A) 100 days B) 150 days C) 240 days D) 83 days

B) 150 days Explanation - This list of gestations should be committed to memory: Llama 1 year (350 days), Horse 11 months (330 days), Cow 9 months (280 days), Sheep/Goat 5 months (150 days), Pig 4 months (114 days), Dog/Cat 2 months (63 days), Ferret 1.5 months (42 days)

You are making serial dilutions of a sample in the laboratory. You start by making a 1:5 dilution of a serum sample. You then take this new solution and make a serial 1:5 dilution of it. What is the dilution ratio of this new solution? A) 1:5 B) 1:25 C) 1:20 D)1:1 E) 1:50

B) 1:25 Explanation - When calculating the final dilution ratio of a serial dilution, you must multiply the dilution factors. Here 1:5 is multiplied by 1:5 and the final dilution is (1 x 1):(5 x 5) = 1:25

A patient has come into the hospital after being stung by a bee. The patient's muzzle is severely swollen, and the doctor has ordered a dose of dexamethasone sodium phosphate to be given intravenously. The patient weight is 30 kg, and the dose is 0.5 mg/kg. The concentration of the dexamethasone is 4 mg/ml. How many milliliters is this patient to receive? A) 1.7 ml B) 3.75 ml C) 12 ml D) 8 ml

B) 3.75 ml Explanation - First calculate the amount of milligrams to be given. 30 kg x (0.5 mg/kg) = 15 mg Next calculate the amount of milliliters to be administered. 15 mg / (4 mg/ml) = 3.75 ml

If you have 25% mannitol and you wish to give 500 mg per kg to a 15 kg dog, how much should you give? A) 300 mls B) 30 mls C) 60 mls D) 3 mls E) 600 mls

B) 30 mls Explanation - The correct answer is 30 mls. 25% corresponds to 250 mg/ml strength. At a dose of 500 mg/kg, the total dose for a 15 kg dog should be 7,500 mg, which divided by 250 mg/ml equals 30 mls

Cats do not have first and second premolars on the mandible. Therefore, what is the number in the Triadan system for the premolar that is closest to the canine on the left mandible? A) 407 B) 307 C) 305 D) 404 E) 405

B) 307 Explanation - In the Triadan system, each tooth has a predictable number, even with teeth missing. Therefore, even though cats do not have PM 1 and 2 (numbered 305 and 306 on the left mandible), the third premolar which is the tooth closest to the canine is 307. In all species, the canine tooth ends in 04 and the first molar ends with 09

A client presents with her 8-week old puppy. She wants to know when she can be spayed. What is the ideal recommended age for spays and neuters? A) 8-10 months B) 5-6 months C) The younger the better, as young as 2 weeks D) 10-12 weeks cats

B) 5-6 months Explanation - The ideal age for this procedure is 5-6 months of age. This helps to decrease the anesthetic risk that may occur in younger animals but allows for the procedure before the first heat cycle. The first heat cycle typically occurs around 6-8 months of age. Most shelter facilities conduct the procedure earlier so that the pets may be adopted

A 20-kg patient is currently receiving 52 ml/hr of Lactated Ringer's Solution. The clinician has requested that 2 mg/kg/day of metoclopramide be added to the bag. Metoclopramide is available as a 5 mg/ml solution. How many milliliters do you need to add to a 1 liter bag? A) 45 ml B) 6.5 ml C) 3 ml D)0.8 ml

B) 6.5 ml Explanation - This is definitely a more complicated calculation question, but when broken down it is not insurmountable. A 20-kg patient will need 40 mg/day of metoclopramide (20 kg x 2 mg/kg/day) Since the fluid is being administered at an hourly rate, we need to figure out how much metoclopramide we are administering per hour. 40 mg/day / (24 hr) = 1.7 mg/hr To determine how much volume of metoclopramide is needed, the following math is performed: 1.7 mg/hr / (52 ml/hr) x (1000 ml)= 32.6 mg (Notice how the units cancel out.) 32.6 mg / (5 mg/ml) = 6.5 ml

A breeder calls and wants to know how long sperm may be viable to fertilize an egg after dogs have mated. What will you tell her? A) 24 hours B) 7 days C) 14 days D) 4 days

B) 7 days Explanation - Canine sperm may live or be able to fertilize an ovum for about 7 days

Socialization plays an important role in companion animal behavior. At what age is it vital that painful and/or traumatic experiences be avoided? A) 6 months B) 8-10 weeks C) 16-20 weeks D) 3-8 weeks

B) 8-10 weeks Explanation - Between the ages of 8 and 10 weeks puppies experience a fear period. Because of this, painful and traumatic situations should be avoided. Between the ages of 3 and 8 weeks puppies socialize best with other dogs. At 16-20 weeks puppies go through a period when they are curious and exploring new environments

Which anesthetic breathing circuit should be used for rabbit anesthesia? A) As long as you are using a precision vaporizer the breathing circuit does not matter B) A breathing circuit with minimal dead space C) A breathing circuit with plenty of dead space D) As long as you are using a nonprecision vaporizer the breathing circuit does not matter

B) A breathing circuit with minimal dead space Explanation - As with small animal anesthesia, as a general rule, any patient under 7 kilograms should be placed on a nonrebreathing circuit. These offer minimal dead space, less resistance to breathing, and provide faster changes in anesthetic depth. Precision vaporizers should always be used in small mammal anesthesia, but this does not change the need for minimal dead space.

What is a localized accumulation of pus in the body is known as? A) Seborrhea B) Abscess C) Vesicle D) Adenoma E) Cyst

B) Abscess Explanation - An abscess is a collection of pus that accumulates and is surrounded by inflammatory tissue. A cyst is a closed-off sac that may contain fluid, air, semi-solid material, etc. A vesicle is a small blister on the skin or mucosa, similar to a cyst. Adenoma is a benign glandular tumor. Seborrhea is a skin condition characterized by either greasy skin from excessive sebum (overproductive glands in the skin), or dry skin characterized by flaking scales

Which of the following terms describes the something that is closer to the root of a tooth relative to another structure? A) Coronal B) Apical C) Caudal D) Distal E) Buccal

B) Apical Explanation - The following terms are used in veterinary dentistry to describe different positions in the mouth and aspects of the tooth: Rostral - Any structure closer to the front of the head relative to another structure Caudal - Any structure closer to the back of the head relative to another structure Buccal - The tooth surface that faces the lips Labial/Vestibular- Same as buccal Facial - The surface of the tooth visible from the front (same as the vestibular surface but applies mainly to the incisor teeth) Lingual - The surface of the mandibular teeth that is adjacent to the tongue Palatal - The surface of the maxillary teeth that is adjacent to the palate Mesial - The portion of the tooth that is in line with the dental arcade and closest to the rostral midline of the dental arch Distal - The portion of the tooth that is in line with the dental arcade and closest to the most caudal aspect of the dental arch (the opposite side of mesial) Apical - The portion of the tooth closer to the tip of the root relative to another structure Coronal - The portion of the tooth closer to the crown relative to another structure

Assuming the daily maintenance fluid requirement is 60 ml/kg/day, how many milliliters per hour should a 4400 lb animal receive per minute to achieve this requirement? A) Approximately 120,000 ml/min B) Approximately 80 ml/min C) Approximately 600 ml/min D) Approximately 5000 ml/min

B) Approximately 80 ml/min Explanation - First, convert the body weight to kilograms. 4400 lbs / (2.2 lb/kg) = 2000 kg Next, calculate daily fluid requirement. 2000 kg x (60 ml/kg/day) = 120,000 ml/day Now, divide by 24 to figure out the hourly requirement. 120,000 ml/day / (24 hr/day) = 5000 ml/hr Lastly, divide by 60 to calculate the amount of fluid necessary per minute. 5000 ml/hr / (60 min/hr) = 83 ml/min

What is the fluid in the anterior chamber of the eye called? A) Lens B) Aqueous humor C) Cornea D) Vitreous humor E) Fundus

B) Aqueous humor Explanation - Aqueous humor is the fluid in the anterior chamber of the eye. It is produced by the ciliary body. The vitreous humor is the gel filling the area between the lens and retina.

What is the most common swine gastrointestinal parasite? A) Stephanurus dentatus B) Ascaris suum C) Erysipelothrix rhusiopathiae D) Trichuris suis

B) Ascaris suum Explanation - Ascaris suum is the large roundworm of pigs. This parasite undergoes hepato-tracheal migration (the egg with L2 is ingested; the larvae undergo hepatic migration and molt to L3, then migrate to lungs where they are coughed up and swallowed. They molt two more times and become adult roundworms in the intestine). Stephanurus dentatus is the swine kidney worm. Erysipelothrix is a bacterium which causes diamond skin disease in pigs. Trichuris suis is the swine whipworm which lives in the cecum; it is also fairly common but not as common as the roundworm

A canine hospitalized following a TPLO surgery unexpectedly cardiac arrests. Endotracheal intubation and chest compressions have been initiated. What two drugs are usually administered FIRST during CPCR? A) Mannitol and hypertonic saline B) Atropine and epinephrine C) Dexamethasone sodium phosphate and methylprednisolone D) Propofol and diazepam E) Dextrose and intravenous fluids

B) Atropine and epinephrine Explanation - Atropine is used in CPCR because it blocks signals from the vagus nerve and is used to treat bradycardia. Epinephrine is used because it increases cardiac output by causing peripheral vasoconstriction and causes bronchodilation. Mannitol and hypertonic saline are used to decrease intracranial pressure and are not the first drugs to be administered during CPCR. Dex SP and methylprednisolone are both steroids and do not increase cardiac function or output. Propofol and diazepam are drugs that might be given during status epilepticus, not during CPCR. Dextrose and fluids may be administered during CPCR but are not usually the first drugs of choice

Agglutination of blood and spherocytes seen on a blood smear are indicative of which type of disease? A) Rickettsial B) Auto-immune C) Bacterial D) Viral

B) Auto-immune Explanation - Agglutination occurs in immune-mediated disorders due to antibodies coating the red blood cells. This results in clumping of the erythrocytes. Spherocytes are often seen with Immune Mediated Hemolytic Anemia (IMHA)

Impulses are carried away from the neuronal cell body by which of the following structures? A) Afferent nerve B) Axon C) Spinous process D) Dendrite

B) Axon Explanation - The axon is the nerve cell projection that conducts the electrical impulses away from the neuronal cell body.

Assessing which of the following on serum chemistry is most appropriate in the assessment of liver function? A) Blood Urea Nitrogen (BUN) B) Bilirubin C) Creatinine D) Glucose

B) Bilirubin Explanation - Bilirubin is an indicator of liver function. However, bilirubin levels must be interpreted cautiously because factors unrelated to liver function can also cause elevated bilirubin; this is true for all of the choices listed. BUN and creatinine are primarily indicators of kidney function

Which technique is needed to produce quality intra-oral radiographs? A) Oblique angle technique B) Bisecting angle technique C) Occlusal angle technique D) Maxillary angle technique

B) Bisecting angle technique Explanation - In the bisecting angle technique, the film is placed next to the tooth; the primary x-ray beam is aimed perpendicular to the plane that bisects the angle created by the plane of the central tooth axis to the plane of the dental film

A dog is going to be euthanized due to aggressive behavior, and the owner wants the dog tested for rabies. The dog has not bitten anyone. What should be submitted? A) Urine B) Brain C) Serum D) Heart E) Salivary gland

B) Brain Explanation - Direct fluorescent antibody testing on the brain is the diagnostic test of choice for confirming rabies. This test detects viral antigens and should be conducted on two locations within the brain (brain stem and cerebellum). If a dog/cat bites a human, it must be quarantined for 10 days or euthanized for rabies testing

Which cranial nerve is known for three major branches? A) CN I B) CN V C) CN XI D) CN XII

B) CN V Explanation - Cranial nerve V (five) is the Trigeminal nerve and has three major branches: First branch: Ophthalmic nerve- sensory to the eye socket and its contents, the anterior nasal cavity, and the skin of the nose and forehead. Second branch: Maxillary nerve- sensory to maxilla, nasal cavity/sinuses, palate, and part of the face. Third branch: Mandibular nerve (the largest branch)- sensory fibers to the lower jaw, the floor of the mouth, the anterior two-thirds of the tongue, the lower teeth, and supplies motor fibers to the muscles of mastication (note: that this is the only branch that supplies motor function).

Endoscopy can be a beneficial diagnostic tool for which of the following conditions? A) Pancreatitis B) Chronic vomiting C) Nephrolithiasis D) Coughing E) Portosystemic shunt

B) Chronic vomiting Explanation - Endoscopy is a scope that is used in the upper gastrointestinal tract. It is used to take biopsies of the stomach and small intestine. It is very useful to examine for underlying disease grossly (via the endoscope's camera) and microscopically via the biopsy specimens. Colonoscopy is used to examine the large intestine. Bronchoscopy is used in the airways and would be useful in a case of chronic coughing

Which intestinal parasite is NOT considered zoonotic to humans? A) Giardia B) Coccidia C) Ancylostoma D) Toxocara

B) Coccidia Explanation - Coccidia are host-specific. Isospora (the coccida found in dogs and cats) is not infective to humans. The other parasites listed are zoonoticorganism

Which of the following is NOT a component of an incisional dehiscence secondary to infection? A) Swelling B) Cold to the touch C) Purulent discharge D) Pain

B) Cold to the touch Explanation - Incisional dehiscences are very serious, and the sooner they are identified the better. When dealing with an abdominal or thoracic incision, it is that much more important to recognize a problem early before there are life-threatening consequences as a result of incision failure. An infection is characterized by all the cardinal signs of inflammation (redness, pain, swelling, heat) as well as purulent discharge

What is the term given to the initial milk produced by a female after giving birth? A) Allantois B) Colostrum C) Chorion D) Amnion

B) Colostrum Explanation - The initial milk produced is known as colostrum because it contains a large amount of antibodies which provide the newborn with immediate immunity. Colostrum plays a very important role in providing immediate immunity to disease in certain species, depending on the type of placenta. For example, it is crucial for calves to receive colostrum, but this is not as important in dogs and cats. The amnion, allantois, and chorion are all fetal membranes around the embryo. The amnion is the closest layer to the fetus and forms a fluid filled sac in which waste material (amniotic fluid) is deposited. The middle layer is the allantois, and the outermost layer is the chorion. The chorion is attached to the endometrium

What is the most rostral structure of the eye? A) Pupil B) Cornea C) Lens D) Retina

B) Cornea Explanation - The cornea is the most rostral structure of the eye and is where light first enters. The retina is at the back of the eye and is the location of the photoreceptors. The lens is responsible for focusing light onto the retina and is located caudal to the pupil and iris

What is the term used to describe the junction between the skin and hoof of the horse's distal limb? A) Periople B) Coronet C) Laminae D) Transitional epithelium E) Frog

B) Coronet Explanation - The transition is called the coronet or coronary band; the hoof is formed by epithelial keratinization over a greatly modified dermis.

What is the shape of a bacillus such as the bacteria that causes anthrax, Bacillus anthracis? A) Spiral B) Cylindrical C) Round and organized in chains D) Circular

B) Cylindrical Explanation - A bacillus is a rod- or cylindrical- shaped bacterium. Circular or round bacteria are cocci

Which of the following is a term for a method of urine collection from a patient in a sterile manner? A) Cystotomy B) Cystocentesis C) Cystogram D) Free catch

B) Cystocentesis Explanation - Cystocentesis involves the sterile introduction of a needle into the bladder for collection of urine. A free catch is the opportunistic collection of urine from a voiding patient. This urine is not sterile because it becomes contaminated as it exits the urethra. A cystotomy is an incision into the bladder. A cystogram is a radiographic contrast study of the bladder

Which disease occurs when the body is unable to make or utilize insulin? A) Cushing's disease B) Diabetes C) Addison's disease D) Pancreatitis E) Hyperandrogenism

B) Diabetes Explanation - Diabetes mellitus occurs when the pancreas does not make insulin or when there is insulin resistance such that the body is unable to utilize insulin.

Which laboratory test is used to differentiate white blood cell types? A) Gram stain B) Diff-Quick stain C) Methylene blue wet mount D) Giemsa's stain

B) Diff-Quick stain Explanation - The Diff-Quick stain is used primarily to stain peripheral blood smears and bone marrow aspirates. It is the lab procedure most commonly used to differentiate white blood cell counts

Which laboratory test is used to differentiate white blood cell types? A) Methylene blue wet mount B) Diff-Quick stain C) Gram stain D) Giemsa's stain

B) Diff-Quick stain Explanation - The Diff-Quick stain is used primarily to stain peripheral blood smears and bone marrow aspirates. It is the lab procedure most commonly used to differentiate white blood cell counts

Which of the following parasites can a dog acquire by ingesting an infected flea during grooming? A) Toxoplasma gondii B) Dipylidium caninum C) Giardia lamblia D) Toxocara canis E) Ancylostoma caninum

B) Dipylidium caninum Explanation - The intermediate host of Dipylidium caninum is the flea; dogs are usually infected when they ingest a flea carrying the parasite while grooming

Coprophagy is the term used to describe an animal that is doing what? A) Regurgitating and then eating what was regurgitated B) Eating feces C) Digging holes D) Straining to defecate

B) Eating feces Explanation - An animal that is eating its own feces or feces of other animals is said to be coprophagic. Coporophagy is a normal behavior of rabbits and many rodents.

Which is a hinge joint connecting the humerus to the radius and ulna? A) Stifle B) Elbow C) Tarsus D) Shoulder

B) Elbow Explanation - The elbow joint is a hinge joint connecting the humerus to the radius and ulna. The shoulder joint is formed by the scapula and humerus. The tarsus and stifle are joints of the hindlimb. The tarsus connects the tibia/fibula to the metatarsus (the ankle); the stifle connects the femur to the tibia/fibula (the knee)

Which of the following medications should be used with caution in cats, as it could cause blindness in some cats? A) Metacam B) Enrofloxacin C) Amoxicillin D) Clindamycin

B) Enrofloxacin Explanation - Enrofloxacin (Baytril) is used in cats at no higher than a 5mg/kg per dose. Higher doses increase the risk for blindness in cats, and in general this drug should be used with caution because of this potential side effect

A 5-week old cat received emergency surgery for a laceration. What antibiotic should NOT be used in this patient? A) Cefazolin B) Enrofloxacin C) Amoxicillin D) Ticarcillin E) Clindamycin

B) Enrofloxacin Explanation - Young animals should not receive enrofloxacin (Baytril) due to risk of cartilage defects (mostly in young large breed dogs). This drug must be used with caution and at lower doses in cats because it may cause blindness

What is most common anticoagulant used for hematology? A) Sodium fluoride B) Ethylenediaminetetracetic acid C) Sodium citrate D) Heparin

B) Ethylenediaminetetracetic acid Explanation - The correct answer is Ethylenediaminetetracetic acid (EDTA). This is the anti-coagulant found in "purple top" tubes. Gray top tubes contain sodium fluoride and are most often used for accurate assessment of glucose levels. Blue top tubes contain sodium citrate and are used for coagulation testing

A patient presents with rodenticide toxicity and is hemorrhaging. The doctor decides the animal needs a transfusion. What blood product is most likely to help stop the patient's bleeding? A) Any blood product will do B) Fresh frozen plasma C) Packed red blood cells D) Stored plasma

B) Fresh frozen plasma Explanation - Rodenticide toxicity causes hemorrhage by inhibiting vitamin K1 clotting factors. Fresh frozen plasma provides coagulation factors. Fresh whole blood could be used as well because it also contains coagulation factors; however, this was not an option in this question. Packed red blood cells don't have any coagulation factors since the plasma has been removed. Stored plasma has plasma proteins only. Platelets or coagulation factors are no longer present at significant levels

For which of the following emergency situations would you most likely pass a stomach tube in a dog? A) Mesenteric torsion B) Gastric-dilatation-volvulus C) Intestinal intussusception D) Perforated stomach

B) Gastric-dilatation-volvulus Explanation - Of the answer choices provided, the only condition which results in life-threatening distension of the stomach is gastric-dilatation-volvulus. This condition results in abnormal positioning of the stomach such that blood flow to the stomach is disrupted and gas accumulation within the stomach occurs. The more gas that accumulates, the less blood is able to flow through the stomach walls. In time, this will cause necrosis of the stomach wall. Eventually, there is enough distension that the stomach begins to compress the caudal vena cava, and blood return to the heart is impeded. This results in shock, and if patients are not treated immediately they will die. One of the emergency procedures used to help stabilize the patient is the passing of a stomach tube. The tube is measured to the level of the stomach and then passed through the mouth. Decompression of the stomach will save the patient's life. However, the stomach should be anchored in a normal anatomical position by performing a gastropexy so that a GDV could never recur

A dog under anesthesia has an end-tidal CO2 (ETCO2) reading of 60 mm Hg. What should be done? A) Increase the anesthetic gas because the patient is light B) Give the patient a breath, or ventilate, to lower the ETCO2 C) Give a dose of atropine IV D) Increase the oxygen flow rate to the patient

B) Give the patient a breath, or ventilate, to lower the ETCO2 Explanation - If the patient has an ETCO2 greater than 55, the patient is hypoventilating. Giving the patient some breaths, or ventilating the patient, will help to lower this value. Increasing the oxygen flow rate would increase the amount of anesthetic flowing through the circuit. Atropine would increase the heart rate. Typically, a patient in a light plane of anesthesia would be hyperventilating and may have a lower ETCO2 (less than 30).

Which of the following nerve blocks can be used to decrease anesthetic requirements and provide analgesia for a dental extraction of a maxillary tooth? A) Inferior alveolar block B) Infraorbital block C) Auriculopalpebral block D) Mental nerve block

B) Infraorbital block Explanation - The infraorbital block or maxillary block are local anesthetic techniques used to decrease anesthetic requirements and provide analgesia for procedures involving the maxilla. The inferior alveolar block and mental nerve block are local anesthetic techniques for the mandible

Which route of drug administration usually allows for the most rapid onset of action? A) Intramuscular B) Intravenous C) Subcutaneous D) Oral

B) Intravenous Explanation - Drugs given IV have the most rapid onset of action, highest initial drug levels, and shortest duration of activity

Which is true of "red bag" during birth in the mare? A) It indicates a tear of the uterus, and a c-section needs to be performed immediately B) It is a protrusion of the chorioallantois and is an emergency C) It is the appearance of a normal placenta as the mare is giving birth D) It indicates a breech position of the foal, and assistance for delivery is needed

B) It is a protrusion of the chorioallantois and is an emergency Explanation - Red bag is a protrusion of the chorioallantois and indicates premature placental separation. It is an emergency situation in which the chorioallantois needs to be broken down manually to assist in delivery of the foal, or death of the foal can occur.

Castration is a common procedure performed in horses. How is a horse commonly positioned for the procedure? A) Dorsal recumbency with legs "hog" tied B) Lateral recumbency with upper rear leg tied around neck C) Dorsal recumbency with legs spread wide D) Dorsal recumbency with rear legs positioned cranially

B) Lateral recumbency with upper rear leg tied around neck Explanation - The most common position for a horse castration is lateral recumbency with the upper rear leg tied around the neck. It is extremely important to confirm that both testicles are descended prior to heavily sedating the patient for the procedure

Which vessel should be avoided when performing venipuncture in pigs? A) Tail vein B) Left jugular vein C) Auricular vein D) Right cranial vena cava

B) Left jugular vein Explanation - The phrenic nerve is located near the left exterior jugular vein, so the left side of the neck in general should be avoided in pigs. The right anterior vena cava is a commonly used site to draw blood from pigs. The auricular vein can be used to draw up to 5 mLs of blood. The tail vein and the orbital sinus near the medial canthus of the eye are other sites that can be used to draw small quantities of blood in the pig

When performing a lameness examination in a horse, perineural anesthesia is used to localize the site of lameness. The most common medication used to perform perineural anesthesia in a horse is what drug? A) Phenylbutazone B) Lidocaine C) Butorphanol D) Flunixin meglumine E) Ketamine

B) Lidocaine Explanation - Lidocaine is a local anesthetic that is used to perform perineural anesthesia during a lameness examination. Ketamine is a dissociative anesthetic; butorphanol is an opioid analgesic. Flunixin and phenylbutazone are systemically administered analgesics.

Which hormone is the trigger for ovulation and development of the corpus luteum? A) Follicle stimulating hormone (FSH) B) Luteinizing hormone (LH) C) Testosterone D) Progesterone

B) Luteinizing hormone (LH) Explanation - Luteinizing hormone is produced by the anterior pituitary gland. The LH surge is the trigger for ovulation and development of the corpus luteum. Progesterone levels increase after ovulation has already occurred

Which of the following retractors would you hand a surgeon if he/she needs to be able to retract the liver out of the way? A) Senn Rake retractor B) Malleable retractor C) Balfour retractor D) Gelpis retractor

B) Malleable retractor Explanation - Malleable retractors are bendable in multiple directions and have no sharp edges. This makes them ideal to use in areas such as the abdomen to gently push delicate organs out of the way. Senn rake retractors are small hand-held retractors that are usually used to retract small muscle bellies, tendons, or ligaments and thus would not be appropriate for the retraction of a liver lobe. Gelpis retractors are self-retraining retractors with sharp points. They should never be used to retract abdominal organs. Balfour retractors are only used to retract the body wall and would not be used inside the abdomen to retract a liver lobe

In dogs, which of the following are all permanent teeth with 2 roots? A) Maxillary premolars 2, 3, and 4, and molar 1 B) Mandibular premolars 2, 3, and 4, and molar 1 C) Maxillary molars 1 and 2 D) Mandibular premolar 1 and molars

B) Mandibular premolars 2, 3, and 4, and molar 1 2 and 3 Explanation - While exceptions do occur, the following are generally accepted and true. In dogs, all of the permanent incisors and canine teeth have 1 root. In the mandible, premolar (PM) 1 has one root. PM 2, 3, and 4 all have two roots. M 1 and 2 have two roots. M3 can have one or two roots. In the maxilla, PM 1 has one root. PM 2 and 3 have two roots, although PM 3 can sometimes have 3 roots. PM 4 and M1 and 2 all have 3 roots.

If hypochromasia is identified in a blood smear, which value on a complete blood count would one anticipate to be decreased? A) Red blood cell count B) Mean corpuscular hemoglobin concentration C) Red cell distribution width D) Mean corpuscular volume

B) Mean corpuscular hemoglobin concentration Explanation - Color in a red blood cell is attributed to the presence of hemoglobin. Cells that lack a normal amount of hemoglobin are said to lack pallor or color. Mean corpuscular hemoglobin concentration is a measure of the amount of hemoglobin present in the average red blood cell. Mean corpuscular volume is a measure of the average volume of a red blood cell. Red cell distribution width is a measure of the average variation in size present in a sample

After performing a blood smear in a patient it is determined that macrocytosis is present. Which value would be expected to be elevated based on this observation on a complete blood count? A) Hemoglobin B) Mean corpuscular volume C) Red blood cell number D) Red cell distribution width

B) Mean corpuscular volume Explanation - Macrocytosis is expected to result in an increase in mean corpuscular volume. Mean corpuscular volume is a measurement of the mean volume in a group of red blood cells. If there is a higher volume, then one can deduce that the cell is larger (macrocytosis). Red blood cell number as well as hemoglobin content should remain unaffected. Red cell distribution is a value that helps the interpreter determine if there is anisocytosis present

Sodium fluoride is the preferred anticoagulant for which of the following purposes? A) Assessment of most enzymatic activities B) Measurement of blood glucose C) Assessment of white blood cell morphology D) Assessment of the coagulation cascade

B) Measurement of blood glucose Explanation - Sodium fluoride is mainly used as a glucose preservative. With other anticoagulants, glucose metabolism may continue after the blood has been drawn from the patient and injected into the collection tube. Sodium fluoride is not as good as EDTA for assessing white blood cell morphology. Citrate is usually used for assessment of the coagulation cascade because its effects are reversible. Sodium fluoride interferes with metabolism and most enzymatic tests

A dog presents for his first day of heartworm treatment. He tested positive for heartworms last week, and the infection was confirmed by chest radiographs and finding microfilariae in his blood. Which drug will this dog receive for killing the heartworms? A) Selamectin B) Melarsomine C) Milbemycin D) Ivermectin

B) Melarsomine Explanation - Melarsomine (more commonly known as Immiticide) is the drug of choice for killing adult heartworms. Care must be used when giving this medication as death of a large worm burden may result in a severe anaphylactic reaction. The others listed are heartworm preventatives that are effective against the microfilariae only

Which cell structure is responsible for producing ATP? A) Lysosome B) Mitochondria C) Golgi complex D) Endoplasmic reticulum

B) Mitochondria Explanation - The mitochondria are responsible for producing the ATP necessary for all cellular functions. The endoplasmic reticulum acts as a transportation system for proteins to be moved as well as for protein synthesis. The Golgi complex modifies and packages compounds that came from the endoplasmic reticulum. They are then released into the cytoplasm for use within the cell. Lysosomes contain enzymes which digest intracellular bacteria and non-functional organelles

The small intestine is made up of four layers. Which of the following describes these layers from inside to outside? A) Serosa, submucosa, mucosa, muscularis B) Mucosa, submucosa, muscularis, serosa C) Muscularis, submucosa, mucosa, serosa D) Serosa, mucosa, submucosa, muscularis

B) Mucosa, submucosa, muscularis, serosa Explanation - The inner most layer of the intestine is the mucosa. It is lined with villi which help to absorb nutrients. The next layers working outward are the submucosa, muscularis, and serosa.

Which bacteria will not stain using the Gram staining method? A) Campylobacter B) Mycobacterium C) E.coli 0157 D) Salmonella

B) Mycobacterium Explanation - Mycobacteria are acid-fast bacteria that have a waxy coat on their cell walls (a lipoid capsule) and so they will not retain the crystal violet stain. The three types of acid-fast stains are Ziehl-Neelson, Kinyoun, and fluorochrome

What is the proper name for the "third eyelid"? A) Frenulum B) Nictitating membrane C) Tympanic membrane D) Ranula

B) Nictitating membrane Explanation - The nictitating membrane is the "third eyelid". The tympanic membrane is the "ear drum". A frenulum is a fold of tissue that prevents movement (such as with a persistent frenulum in the bull regarding the penis). Palpebra is the proper term in general for eyelid. A ranula is a mucocele usually found under the tongue region

Which of the following describes the best method for preparing a urine sediment sample for microscopic evaluation? A) Once sample is obtained, transfer a small amount with the aid of a pipette onto a microscopic slide and evaluate B) Once sample is obtained, centrifuge the sample and remove the supernatant. Using a pipette, transfer small amount of urine onto a microscopic slide and evaluate C) Once sample is obtained, heat it under a Bunsen burner until an adequate amount of supernatant has evaporated. Then, obtain a small sample with a pipette and transfer it to a microscopic slide for evaluation D) Once sample is obtained, allow it to sit in a refrigerated environment for 12 hours. Following this, centrifuge the sample, remove the supernatant, and transfer a small amount of urine onto a microscopic slide and evaluate

B) Once sample is obtained, centrifuge the sample and remove the supernatant. Using a pipette, transfer small amount of urine onto a microscopic slide and evaluate Explanation - The best method for preparing a urine sample for microscopic evaluation is by centrifuging the sample and removing the supernatant first. Once this is complete, a small amount of urine from the tip of the tube can be transferred to a slide for evaluation. Heating the urine sample will destroy cellular elements and is not recommended. Centrifuging the sample is important in order to obtain a useful sample to evaluate microscopically. The sample should then be evaluated at 10x (low power field) and then at 40x (high power field)

What is of the biggest concern when shipping pigs? A) Dehydration B) Overheating C) Stress-induced diarrhea D) Gastric dilatation

B) Overheating Explanation - Heat prostration is of greatest concern when moving and shipping porcine. The same is true when performing medical procedures on them. It is always best to handle swine in the early morning when the temperature is cooler. This is due to their large amount of body fat and lack of sweat glands. The same is true for sheep; they easily over-heat due to their thick coat

Several Heinz bodies have been identified on a blood smear of a sick dog. What is this an indication of? A) Viral hemolysis B) Oxidative damage C) Carbon monoxide exposure D) Cyanide poisoning

B) Oxidative damage Explanation - Heinz bodies are seen whenever oxidative damage occurs to the red blood cell. They can be seen with immune mediated hemolytic anemia or ingestion of certain toxins. For example, consumption of onions results in Heinz body anemia. Another example is the administration of methylene blue, which also causes oxidative damage to red blood cells. If a large enough amount is given, this can cause anemia

Which organ is responsible for producing most of the digestive enzymes used to breakdown food? A) Spleen B) Pancreas C) Stomach D) Liver

B) Pancreas Explanation - The pancreas produces enzymes such as trypsin, lipase, and amylase to breakdown food. These are packaged into zymogens, which are released and activated by trypsin.

What is another method, besides drawing from the tail vein, to blood draw in a laboratory rat? A) Cephalic vein B) Periorbital plexus C) Jugular vein D) Anterior vena cava

B) Periorbital plexus Explanation - The periorbital plexus can be used in sedated rats to collect small blood samples using a hematocrit tube. The other methods are not used in laboratory rats

Which of the following species have teeth that only grow for a limited time during development and do not continually erupt or grow throughout the life of the animal? A) Rabbits B) Pigs C) Horses D) Rats

B) Pigs Explanation - Cats, pigs, humans and other carnivores have all brachyodont teeth. This means that they have a small crown relative to the size of the roots, and the apex of the each tooth root is only open for a brief period during development and does not continue to erupt. Horses, rodents, and lagomorphs have hypsodont teeth, which means that they have a relatively large amount of crown that is beneath the gingival margin as well as a root structure that allows for continued eruption during most of the animal's lifetime. Hypsodont teeth can be further categorized as radicular hypsodont teeth, such as the cheek teeth of horses. These have apices that eventually close and stop growing, but continued eruption offsets attrition from occlusion. Aradicular hypsodont teeth (such as the incisors of rabbits) lack a true root structure, allowing for lifelong growth of the tooth itself

Which of the following lists the correct order of the phases in the estrous cycle? A) Anestrus, diestrus, estrus, metestrus, proestrus B) Proestrus, estrus, metestrus, diestrus, anestrus C) Metestrus, proestrus, estrus, anestrus, diestrus D) Estrus, proestrus, diestrus, metestrus, anestrus

B) Proestrus, estrus, metestrus, diestrus, anestrus Explanation - Proestrus is the time when ovarian follicles are starting to grow. It immediately precedes estrus. Estrus is the time of actual heat when follicles are mature and ovulation occurs or can be induced depending on species. During Metestrus, estrogen stimulation subsides and the corpus luteum starts to form. The uterine lining begins to secrete small amounts of progesterone. Diestrus is regression of the corpus luteum. Anestrus is resting of the sexual cycle

A dog is very weak in the hindend but is able to stand with support. The dog is being supported under the abdomen, and the paws are individually turned over so that the dorsal paw is touching the ground. The dog is not able to correct this posture. This is an evaluation of: A) Mentation B) Proprioception C) Spinal pain D) Malalignment of the vertebral column

B) Proprioception Explanation - Proprioception is a test to determine if the pet knows where the limbs are in space, or position awareness. The paw is first flipped under so that the dorsal part of the foot is touching the floor, and then the paw is released. The pet should return the paw to the normal positioning. If the paw remains flipped over, this indicates a proprioceptive deficit. It is most often a result of spinal cord disease or spinal cord compression from disk disease, neoplasia, etc

Upon gram staining, gram-positive bacteria will appear what color when visualized under the microscope? A) Pink B) Purple C) Blue D) Orange

B) Purple Explanation - Gram-positive bacteria stain purple because of the chemical and physical properties of the cell wall. Gram-negative bacteria would appear pink

Which of the following is true about red blood cells? A) Nucleated red blood cells are never seen in dogs B) Red blood cells are normally nucleated in birds C) Red blood cells have a lifespan of over 2 years D) Red blood cells are exclusively produced in the spleen

B) Red blood cells are normally nucleated in birds Explanation - Interestingly, red blood cells are normally nucleated in birds and reptiles. Nucleated red blood cells can be seen in other species whenever there is a demand on the body to rapidly release red blood cells into the bloodstream. If demand exceeds the bone marrow's production capabilities, the body will begin to release juvenile red blood cells, which still have a nucleus

When running a PCV, why are the red cells at the bottom and white cells at the top of the hematocrit tube? A) White cells are heavier B) Red cells are heavier C) Hemolysis makes red cells sink D) White cells are smaller E) Red cells have a low specific gravity

B) Red cells are heavier Explanation - Erythrocytes (red blood cells) have the highest specific gravity of all of the blood cells. That is why the red blood cells all end up at the bottom of the tube when a whole-blood tube is spun in a centrifuge. The white blood cells and platelets form the buffy coat which lies above the erythrocytes

Which of the following would be elevated in a pet with regenerative anemia? A) Hematocrit B) Reticulocyte count C) Hemoglobin concentration D) Lymphocyte count

B) Reticulocyte count Explanation - Reticulocytes are newly released red blood cells. These immature erythrocytes are indicative of a bone marrow response. An anemia is considered regenerative if the reticulocyte count is greater than 60,000. The hematocrit is low in a patient with anemia. Hemoglobin is typically low in a patient with anemia, especially if the pet has iron deficiency. Lymphocyte number can be variable but is not usually directly related to the anemia

The correct order of the ruminant digestive system in regards to the chambers of their stomach is: A) Rumen, reticulum, abomasum, omasum B) Rumen, reticulum, omasum, abomasum. C) Abomasum, omasum, rumen, reticulum D) Reticulum, rumen, omasum, abomasum

B) Rumen, reticulum, omasum, abomasum. Explanation - This is the order of the "stomachs" in ruminants such as cattle. The abomasum is the true glandular stomach. The rumen is the largest chamber. The reticulum, which has a honeycomb type lining, is positioned up against the diaphragm and is the chamber where a magnet can be placed to try and prevent Hardware Disease. If the cow swallows a metal object, it falls into the reticulum where the magnet can attach to it and prevent the object (such as a nail or wire) from piercing through the diaphragm into the heart.

Drugs that have no accepted medical use, are not considered safe, and have high potential for abuse are the most controlled by the DEA (Drug Enforcement Administration). Drugs under this category such as heroin are listed as what type of category? A) Schedule V controlled substance B) Schedule I controlled substance C) Schedule III controlled substance D) Schedule II controlled substance E) Schedule IV controlled substance

B) Schedule I controlled substance Explanation - Schedule I substances are the most controlled and are unlikely to be available at a veterinary clinic. Schedule II substances such as morphine and fentanyl are frequently the most tightly controlled substances in a veterinary hospital

The sac containing the testicles is known as which of the following? A) Glans B) Scrotum C) Cremaster D) Epididymis

B) Scrotum Explanation - The scrotal sac contains the male gonads (testes

A dorsoproximal-dorsodistal oblique radiograph of the carpus is frequently referred to as what type of radiograph? A) Cross-sectional view B) Skyline view C) Flexed lateral view D) Bisecting angle E) Horizontal beam

B) Skyline view Explanation - With a standing patient, a skyline view is taken with the carpus flexed and the x-ray cassette placed along the anterior aspect of the limb. The x-ray beam is aimed from proximally to distally and an oblique angle. This technique is used in equine radiography most frequently to view the distal radius, the proximal row of carpal bones, and the distal row of carpal bones where chip or slab fractures commonly occur, especially in racehorses. Sclerosis of the carpal bone can also be visualized here. This technique decreases the overlap of surrounding bones and structures in order to improve the ability to see lesions specifically in this area. A flexed lateral radiograph also involves flexing the carpus, but the beam is aimed from laterally and the cassette is placed medially. This technique allows the carpal bones to separate and makes the margins of the bone more clearly visible. A horizontal beam radiograph generally is used to confirm the presence of air or fluid in a body cavity (thorax, abdomen). A bisecting angle radiograph is a technique used to prevent distortion when it is not possible to place the film parallel to the structure of interest and perpendicular to the x-ray beam, commonly in dental radiography

Fasciola hepatica requires which intermediate host? A) Flea B) Snail C) Rat D) Mosquito

B) Snail Explanation - Fasciola hepatica is the liver fluke. It is typically found in wet environments where snails proliferate. Snails are the intermediate host for this parasite, and the most common definitive hosts are sheep and cattle. It is a flat worm, resembling a leaf.

Pasteurella infection, most commonly known as which of the following? A) Rabbit fever B) Snuffles C) Q fever D) Kennel cough

B) Snuffles Explanation - Snuffles is caused most often by Pasteurella multocida and results in upper respiratory symptoms and sometimes conjunctivitis in rabbits. It is highly contagious and generally treated with antibiotics. Tularemia is sometimes known as Rabbit fever. Q fever is caused by Coxiella burnetti

Which species is particularly aggressive when with her young, and extreme caution must be used? A) Cows B) Sows C) Ewes D) Mares

B) Sows Explanation - In general, pigs are a more aggressive species. The lactating sow is particularly dangerous; and when handling piglets, it is best to remove the sow to a separate area where she cannot hear her piglets

On a routine urinalysis an abundant amount of colorless prism-like crystals that have a coffin lid appearance are observed. What type of crystals are these? A) Cysteine B) Struvite C) Ammonium biurate D) Calcium oxalate

B) Struvite Explanation - Struvite crystals are clear/colorless and have a rectangular appearance. They are commonly described as having a "coffin lid" appearance. Ammonium biurate crystals are round and brownish in color. They have spicules coming off of them. Calcium oxalate crystals have a clear-colored square appearance with an X across the surface

In the horse, the condition known as Sweeney is caused by damage to which structure? A) Radial nerve B) Suprascapular nerve C) First cervical vertebrae D) Brachial plexus

B) Suprascapular nerve Explanation - Sweeney is the term for atrophy of the shoulder muscle in the horse. It is caused by damage to the suprascapular nerve. This nerve is responsible for innervation to the infraspinatus and supraspinatus muscles, which are found in the scapula (shoulder blade)

The veterinarian you work for is referring a Cocker Spaniel for an ear canal ablation procedure. What will this procedure entail? A) Opening of the tympanic membrane and flushing of the middle ear B) Surgical excision and closure of the external ear canal C) A deep flush and culture of the ear canal D) Ligation of the 8th cranial nerve

B) Surgical excision and closure of the external ear canal Explanation - Ear canal ablation is often performed in dogs with chronic stenotic, painful, and infected ears in which ancillary therapies have failed. It entails surgical removal and closure of the external ear canals

If a horse receives a cut on the leg from barbed wire and is not vaccinated, what is the protocol for preventing tetanus? A) Antibiotics B) Tetanus antitoxin within 24 hours C) No vaccine can be given after exposure, wash the wound with 2% chlorhexidine solution daily D) Tetanus toxoid 2 weeks after injury

B) Tetanus antitoxin within 24 hours Explanation - A horse that is potentially exposed and not vaccinated should receive the tetanus antitoxin within 24 hours of injury. This will provide some temporary immunity for about 2 weeks. If the wound is still present at that time, the vaccination is given again. A horse that actually has tetanus is given both tetanus toxoid and antitoxin

Ectopic pregnancy means which of the following? A) There is no fetus, just an empty amniotic sac B) The pregnancy is occurring outside of the uterus C) The fetus is getting resorbed by the body D) There is more than one fetus

B) The pregnancy is occurring outside of the uterus Explanation - Ectopic pregnancy occurs when a fertilized egg has implanted outside the uterus, most often in the fallopian tubes. Mostly these pregnancies are not viable and pose a bleeding risk which can lead to death

An animal with pleural effusion may require which procedure for stabilization? A) Abdominocentesis B) Thoracocentesis C) Orogastric tube D) Radiographs

B) Thoracocentesis Explanation - A large amount of fluid in the pleural space (around the outside of the lungs) will prevent the lungs from fully expanding. A chest tap (thoracocentesis) will allow the fluid to be drained from the pleural space and allow the patient to expand the lungs and breath easier. While radiographs may be needed, an unstable patient in respiratory distress should not have radiographs. The chest tap should occur first to stabilize the patient

What is the function of the screen of an X-ray cassette? A) Absorb scatter radiation to increase image clarity B) To decrease the exposure necessary to create an image on the film C) Allow use of higher kVp settings D) Protect the film from light

B) To decrease the exposure necessary to create an image on the film Explanation - The screen of an X-ray cassette, sometimes called intensifying screens, are the shiny white inner surfaces of the cassette. They are made of crystals that fluoresce, emitting light, when exposed to x-rays, producing the diagnostic image

Why do you use a heparin flush to flush an IV catheter? A) To reduce pain associated with flushing the catheter B) To prevent clot formation in the catheter C) To prevent phlebitis D) To prevent infection associated with the catheter

B) To prevent clot formation in the catheter Explanation - Heparin is an anticoagulant and helps to prevent clot formation in the IV catheter. A catheter should be flushed prior to and after giving IV medications to make sure it is functioning properly, and then flushed every 4 hours when not in use to help prevent clot formation. Phlebitis is inflammation of the vein; heparin does not prevent this

Which parasite is commonly transmitted transplacentally to puppies? A) Isospora B) Toxocara C) Trichuris D) Dipylidium

B) Toxocara Explanation - Toxocara canis (roundworms) commonly infect puppies via the transplacental route

The lining of the urinary bladder is composed of which cell type? A) Columnar cell B) Transitional cell C) Parietal cell D) Squamous cell

B) Transitional cell Explanation - Transitional cells are unique in that they allow the bladder to stretch as well as decrease in size

Ivermectin would not be used when treating which parasite? A) Mites B) Trematodes C) Ticks D) Nematodes

B) Trematodes Explanation - The correct answer is trematodes (flukes). Ivermectin is ineffective against flukes. Ivermectin has activity against ticks, mites, nematodes, and even lice. Ivermectin works by stimulating GABA which is an inhibitory neurotransmitter

During ultrasound of the bladder, a hyperechoic (white) linear structure is seen against the bladder wall that creates a black shadow obscuring all structures deep to it. What is this structure most likely? A) Bladder wall tumor B) Urolith C) Free fluid D) Round ligament of the bladder

B) Urolith Explanation - Sound is strongly reflected by mineral and gas resulting in a hyperechoic appearance. The structure most likely to be mineralized inside the bladder is a urolith. The bladder wall should be of intermediate echogenicity as are most bladder wall tumors. The round ligament of the bladder is usually not visible sonographically but also would not create such a dramatic acoustic shadow as described in the question. Free fluid appears anechoic (black). Air inside the bladder as might be seen in emphysematous cystitis could also be echogenic and cast a shadow but this was not an answer choice

The dog's abdominal region may also be referred to as which of the following? A) Proximal region B) Ventral region C) Dorsal region D) Anterior region

B) Ventral region Explanation - Cranial is toward the head; caudal is toward the tail. Ventral is toward the abdomen; dorsal is toward the backbone. Lateral is away from midline and medial is toward midline. Proximal is close to the spine or body while distal is away; these are typically used when describing limbs (ex. fracture of the proximal femur vs. fracture of the distal femur). Palmar is the bottom of the paw of the forelimb; plantar is the bottom of the paw of the hind limb. Anterior is toward the head and posterior is away from the head. In veterinary medicine, anterior and posterior are usually only used to describe distal extremities or areas of the head

The somatic nervous system is responsible for which of the following functions? A) Contraction of the intestines B) Voluntary motor movement C) Regulation of heart rate D) Secretion of saliva

B) Voluntary motor movement Explanation - Secretion of saliva, contraction of the intestines, and regulation of the heart rate falls under the responsibility of the autonomic nervous system. More specifically, the heart rate is influenced by the parasympathetic and sympathetic nervous systems, both of which are branches of the autonomic nervous system

When performing a direct fecal smear to look for "swimmers" or parasitic trophozoites, the feces cannot be older than which of the following? A) 5 minutes B) 12 hours C) 1 hour D) 24 hours if not refrigerated

C) 1 hour Explanation - Fresh feces are needed for a direct smear. The smear should be made and read within 1 hour of fecal collection for most accurate results. Also, serial samples are ideal. Sometimes 4 to 5 smears are needed to find an organism. A negative direct smear does not rule out a parasite

The Triadan numbering system is commonly used in human and veterinary dentistry to specify a tooth. What is the correct number of the permanent right maxillary canine tooth of a dog or cat? A) 501 B) 301 C) 104 D) 201 E) 404

C) 104 Explanation - In this system, the first numeral denotes the quadrant of the tooth and if it is permanent or deciduous. They are numbered starting at the right maxilla and moving counter-clockwise: 1 = Right maxilla (5 = deciduous) 2 = Left maxilla (6 = deciduous) 3 = Left mandible (7 = deciduous) 4 = Right mandible (8 = deciduous) The next two digits indicate how many teeth over from the midline. For example, in the dog: 701 is the deciduous mandibular left first incisor. 104 is the permanent right maxillary canine. 409 is the permanent mandibular right first molar

Pregnancy check in the mare can be performed via ultrasound as early as which of the following? A) 60 days B) 30 days C) 14 days D) 42 days

C) 14 days Explanation - Ultrasound is the method of choice for detecting pregnancy in the mare and can be done as early as 11 to 14 days (around 2 weeks).

A patient is hypoventilating and the end-tidal CO2 is rising. The doctor instructs you to give the patient a breath. The pressure on the manometer to which you ventilate the patient is: A) 60 cm H2O B) 5 cm H2O C) 20 cm H2O D) 40 cm H2O

C) 20 cm H2O Explanation - When inflating the lungs, the pressure should reach but not exceed 20 cm H2O

How many milliliters of 25% dextrose should be added to 1 L of 0.9% saline to make a 5% dextrose solution? A) 20 ml B) 100 ml C) 200 ml D) 50 ml

C) 200 ml Explanation - To answer this question, the following formula should be implemented: C(1)V(1) = C(2)V(2) C is the concentration V is the volume C(1) =0.25 V(1) = x C(2) = 0.05 V(2) = 1000 ml 0.25x = 0.05(1000) x = 200 ml 200 ml of 25% dextrose should be added to one liter in order to make a 5% solution

How many milliliters of dextrose should be added to a 60 ml syringe of 0.9% NaCl to make a 2.5% dextrose solution? The patient weighs 20 kg, and the dextrose in the hospital comes in a 50% concentration. A) 0.6 ml B) 1.5 ml C) 3 ml D) 36 ml E) 120 ml

C) 3 ml Explanation - To answer this question, use the following equation: C(1)V(1) = C(2)V(2) C is the concentration V is the volume C(1) =0.025 V(1) = 60 ml C(2) = 0.50 V(2) = x 0.025(60)= 0.50x x = 3 ml Note that the body weight is completely unnecessary to calculate the answer to this problem. 200 ml of 25% dextrose should be added to one liter in order to make a 5% solution.

Calculate the standard maintenance fluid rate for a 30-pound dog. How many milliliters per hour should the dog receive? A)56 mL/hr B) 12 mL/hr C) 34 mL/hr D) 42 mL/hr E) 50 mL/hr

C) 34 mL/hr Explanation - 30 pounds/ 2.2 kg= 13.6 kg 13.6 kg X 60 mL per day = 818 mL/kg per day 818 mL/ 24 hours = 34 mL/hr The simplest maintenance fluid calculation equation is 60 mL/kg/day

You perform a dental prophylaxis on a dog that has all of his teeth. How many adult teeth are in the dog's mouth? A) 32 B) 28 C) 42 D) 24

C) 42 Explanation - The adult dog has 42 teeth. Cats have 30 teeth. Most cats and dogs have all of their permanent teeth by 6 months of age

At what age should pigs be castrated? A) 4 weeks B) 8 months C) 5 - 14 days D) 4 months

C) 5 - 14 days Explanation - Piglets are castrated very young due to ease of handling

A cat is starting weekly subcutaneous fluid administration for renal failure at the clinic where you work. The client asks how long it will take for him to absorb the fluids after they are given. You tell him most cats will absorb the fluid within: A) 48 hours B) 1 hour C) 5-8 hours D) 24 hours

C) 5-8 hours Explanation - Subcutaneous fluids are most commonly absorbed within 5 to 8 hours of administration. However, fluids are absorbed when they are needed, and absorption is dependent on having adequate SQ blood flow, which can be decreased in severely hypothermic or dehydrated patients

The attending veterinarian asks you to administer 120 mg of enrofloxacin (Baytril) IV slow over 20 minutes to a dog that is hospitalized in your clinic. The strength of injectable enrofloxacin is 2.27%, how many milliliters of drug will you administer? A) 0.53 mls B) There is not enough information available to calculate the dosage C) 5.3 ml D) 52.8 mls

C) 5.3 ml Explanation - A 2.27% solution is 22.7 mg/ml. 120 mg divided by 22.7 mg/ml = 5.3 mls

What adverse side effect can occur in cats given metoclopramide? A) Cyanosis B) Dilated and fixed pupils C) Aggressive behavior D) Estrus behavior

C) Aggressive behavior Explanation - Rare side effects that can occur are nervous, jittery, or aggressive behaviors. The antidote for these types of behaviors is diphenhydramine (Benadryl). Head bobbing can also rarely occur as a side effect of Reglan (metoclopramide).

Which of the following terms describes variation in cell size? A) Agglutination B) Megalocytosis C) Anisocytosis D) Anisokaryosis

C) Anisocytosis Explanation - Anisocytosis is the term used to describe variation in cell size. Agglutination is a descriptive term for cells red blood cells that are adhering or clumping with each other. Anisokaryosis implies variation between cells in the amount of cytoplasm present. Megalocytosis implies enormous and abnormal cells, usually with multiple nuclei present

When is generally considered the optimal time to spay a dog? A) Anytime after the second estrus, but before the third B) Anytime after the third estrus C) At approximately 6 months of age D) At 2 years of age

C) At approximately 6 months of age Explanation - Spaying before the first estrus dramatically reduces the lifetime likelihood of developing mammary neoplasia. Dogs that are spayed after the first estrus have an 8% chance of developing mammary neoplasia. Dogs spayed after the second estrus have a 24% chance of developing mammary neoplasia. Performing a spay at the time of estrus can be done but is not recommended, as the uterus is highly vascularized at this time, and the chances of complications or excessive bleeding may be greater. It is usually recommended to perform a spay 3-4 months after estrus (if the dog went into estrus).

Which of the following correctly indicates where electrons are generated in an x-ray machine? A) At the filament of the anode B) At the target of the cathode C) At the filament of the cathode D) At the target of the anode

C) At the filament of the cathode Explanation - The cathode flament of the x-ray machine is where electrons are generated. The kVp applied directs those electrons to the anode, where they hit a tungsten target to produce x-rays

A cat is given dexmedetomidine, and the cat's heart rate has dropped significantly. The veterinarian asks you to give the reversal, which is the following is the appropriate reversal agent? A) Acepromazine B) Yohimbine C) Atipamezole D) Naloxone

C) Atipamezole Explanation - The reversal of medetomidine or dexmedetomidine (Domitor or Dexdomitor) is atipamezole (Antisedan). Naloxone reverses morphine and other opioids. Yohimbine reverses xylazine. Acepromazine is a tranquilizer and is not a reversal agent

Which joint allows rotational movement of the head from side to side (from right to left or left to right)? A) Coxofemoral B) Scapulohumeral C) Atlantoaxial D) Atlantooccipital

C) Atlantoaxial Explanation - The atlantoaxial joint is a pivot joint which allows rotation of the head from side to side. It is located between the atlas and axis. The atlantooccipital joint provides the up and down motion of the head and is located between the occipital condyles of the skull and C1 (atlas). The coxofemoral joint is the hip joint. The scapulohumeral joint is where the scapula joins with the proximal humerus

All of the following are typically included in a complete blood count EXCEPT for which of the following? A) Differential white blood cell count B) Erythrocyte indices C) Blood urea nitrogen levels D) Plasma protein concentration E) Hemoglobin concentration F) Platelet count and/or estimate

C) Blood urea nitrogen levels Explanation - A complete blood count provides a total red blood cell count, white blood cell count, and platelet count and/or estimate in addition to a packed cell volume or hematocrit and plasma protein concentration. It includes a blood smear evaluation to assess the differential white blood cell count as well as blood cell morphology. Hemoglobin concentration and erythrocyte indices of size are included. For anemic patients, a complete blood count should also include a reticulocyte count. Blood urea nitrogen is not part of a complete blood count and is considered a chemistry value

The main stimulus for ventilation is the partial pressure of which of the following gases? A) Nitrogen B) Oxygen C) Carbon dioxide D) Carbon monoxide

C) Carbon dioxide Explanation - Elevations of carbon dioxide are the main stimulus for respiration. Low levels of oxygen can also stimulate ventilation; however, carbon dioxide is a more potent stimulant

What is the vertebral formula for dogs and cats? A) Cervical 6, Thoracic 12, Lumbar 5, Sacral 13 B) Cervical 6, Thoracic 10, Lumbar 7, Sacral 7 C) Cervical 7, Thoracic 13, Lumbar 7, Sacral 3 D) Cervical 3, Thoracic 12, Lumbar 6, Sacral 7

C) Cervical 7, Thoracic 13, Lumbar 7, Sacral 3 Explanation - The dog and cat have 7 cervical, 13 thoracic, 7 lumbar, and 3 sacral vertebrae as a general rule (in rare cases the dog may have 6 lumbar vertebrae).

A subtotal colectomy in which a small amount of colonic contents spills into the abdomen in a cat is an example of which of the following? A) Clean-contaminated surgery B) Infected (dirty) surgery C) Contaminated surgery D) Clean surgery

C) Contaminated surgery Explanation - In order from cleanest to most at risk from infection, surgery can be categorized by the following system: Clean, clean-contaminated, contaminated, and infected surgery. These can also be referred to as category I, II, III, or IV surgeries respectively. A clean surgery is one where the surgical wound is made under aseptic conditions, no contaminated viscous is entered, and no breaks in sterile technique occur. Examples include castration, splenectomy, and hernia repair. A clean-contaminated surgery is one made under aseptic conditions that enter the oropharynx, respiratory, alimentary, or urogenital tracts, but there is no other source of contamination (i.e. no spillage of contents). Examples include lung lobectomy and gastrotomy. A contaminated surgery is one where there is either a spill of contaminated material, a break in sterile technique, or entry into a region with a high bacterial load such as the oral cavity or colon. An infected or dirty surgery is a procedure involving a site with known infection. Examples include ear canal ablation or removal of abscessed or necrotic tissue

Which of the following pairs of measurements are equivalent to each other? A) Nanometer and microliter B) Kilogram and liter C) Cubic centimeter and milliliter D) Kilogram and kiloliter E) Cubic foot and meter

C) Cubic centimeter and milliliter Explanation - A cubic centimeter is the same volume as a milliliter. A liter of water weighs one kilogram, but these terms are not interchangeable because a liter is a volume and a kilogram is a weight. A liter of a heavier fluid would weigh more than a kilogram. A cubic foot is a volume, and a meter is a distance

A fresh biopsy sample from a nasal lesion is pressed against a glass slide several times to leave an impression. The cells that are on the slide are stained with a Romanowsky stain and viewed under a microscope. Which of the following tests is this an example of? A) Histopathology B) Immunocytochemistry C) Cytology D) Immunohistochemistry E) Histology

C) Cytology Explanation - This is an example of cytology (referring to the examination of cells from body tissues or fluids). Cytology specimens may be collected by impression smear as described in this question or by other techniques including needle aspirate, aspiration of fluid (centesis), or by swab with a cotton-tipped applicator. Histology involves the examination of complete tissue and architecture and is usually performed on tissue collected by biopsy and processed intact. Histopathology is the same except that it involves the assessment of diseased tissue. Immunocytochemistry involves using special stains that are taken up by specific cellular components or markers on a cytologic specimen. Immunohistochemistry involves similar special stains applied to histologic sections

A deep skin scraping wherein the skin is scraped until capillary blood is seen is generally necessary to recover which of the following parasites? A) Psoroptes B) Chorioptes C) Demodex D) Cheyletiella

C) Demodex Explanation - Surface dwelling mites such as Sarcoptes, Cheyletiella, Psoroptes, and Chorioptes only require a superficial scraping to collect loose scales or crusts. A deeper scraping is necessary to recover Demodex mites which live deeper in the follicles

What is new methylene blue stain is used mostly for? A) Determining if bacteria are gram-negative B) Determining if bacteria are acid-fast C) Determining morphology of cells D) Determining if bacteria are gram-positive E) Staining yeast

C) Determining morphology of cells Explanation - New methylene blue (also NMB) is an organic staining agent used in diagnostic cytopathology and histopathology. It is useful for determining shape or structure of cells and especially helpful in examining immature erythrocytes. New methylene blue stain has many other uses such as evaluating vaginal smears during estrus and examining for blood parasites like heartworm microfilariae

Which of the following medications is commonly found over-the-counter (OTC)? A) Pimobendan B) Hydrocodone C) Diphenhydramine D) Doxycycline

C) Diphenhydramine Explanation - The other name for diphenhydramine is Benadryl and can be found over-the-counter. Hydrocodone is commonly used as a cough suppressant in veterinary medicine and is a controlled substance requiring a prescription. Doxycycline (an antibiotic) and Pimobendan (a heart medication) are by prescription only

Colic is a common condition in horses. Which of the following is not a common cause of colic? A) Enterolithiasis B) Intestinal volvulus C) Displaced abomasum D) Intestinal incarceration

C) Displaced abomasum Explanation - Horses are not ruminants and therefore do not have an abomasum. The abomasum is the fourth stomach in ruminants and functions in a similar fashion to a monogastric stomach. Enteroliths are stones that can form in the gastrointestinal tract that result in an obstruction. Intestinal incarceration is the trapping of intestines through a small opening. Finally, an intestinal volvulus occurs when a section of intestine spins about its axis and occludes its own blood supply

The Greyhound is an example of which of the following breeds? A) Mesaticephalic B) Brachycephalic C) Dolichocephalic D) Brachygnathic

C) Dolichocephalic Explanation - The Greyhound has a long and narrow head. The nose and cranium are parallel and are divided at eye level, where the cranium descends to nose level. The Greyhound is a classic example of a dolichocephalic breed. Examples of a brachycephalic breed include a Pug, English bulldog, and Pekingese. In the brachycephalics, the face is very short and the cranium wide. The eyes are more widely spaced. This type of breed often has brachygnathism, meaning they have an underbite. Mesaticephalic breeds have a skull with the cranium and nasal cavity about equal lengths. The Labrador is a good example of this type.

What is the most sensitive and preferred test for detecting Giardia? A) Direct smear B) Fecal flotation with zinc centrifugation C) ELISA D) PCR

C) ELISA Explanation - The most sensitive test for detecting Giardia is the ELISA. This will detect Giardia in greater than 95% of cases. The Fecal flotation with zinc centrifugation will only detect Giardia less than 70% of the time. A direct smear can be used to look for trophozoites but is not highly sensitive, and multiple smears may be needed to find the

You run a CBC on a dog that is heartworm positive. Which blood cell is commonly elevated in dogs with heartworm disease or other parasitic infections? A) Monocytes B) Bands C) Eosinophils D) Red blood cells E) Platelets

C) Eosinophils Explanation - Parasitic infections (such as heartworm, intestinal parasites, and fleas) often cause an elevated eosinophil count. Other diseases which may cause an elevation of eosinophils include allergies, chronic dermatitis, and feline asthma

What is Mycology the study of? A) Muscles B) Bacteria C) Fungi D) Parasites

C) Fungi Explanation - Mycology is the branch of biology that is the study of fungi. The study of muscles is myology (with no c). The study of bacteria is bacteriology. The study of parasites is parasitology

When cattle are placed in lateral recumbency during general anesthesia, how should the head be positioned to decrease the risk of aspiration? A) Head and oral cavity higher than the neck B) Head and oral cavity at the same level as the neck C) Head and oral cavity lower than the neck D) Head and oral cavity higher than the neck or at the same level as the neck

C) Head and oral cavity lower than the neck Explanation - During general anesthesia the esophageal sphincter relaxes, if regurgitation occurs the fluid can be aspirated. Positioning the head and oral cavity lower than the neck paired with a properly inflated endotracheal cuff can reduce this risk.

Oxidative damage to red blood cells commonly causes formation of which of the following? A) Spherocytes B) Schistocytes C) Heinz bodies D) Howell-Jolly bodies E) Basophilc stippling

C) Heinz bodies Explanation - Heinz bodies are round structures seen in red blood cells that represent denatured hemoglobin caused by oxidative damage. They are seen best when cells are stained with new methylene blue. A spherocyte is a dark-staining red blood cell with no central pallor that usually occurs from partial phagocytosis of the cell, as is seen with immune-mediated destruction. A schistocyte is a red blood cell fragment that is usually formed due to shearing from intravascular trauma; it is seen in cases of DIC or vascular neoplasms such as hemangiosarcoma. Howell-Jolly bodies are basophilic nuclear remnants seen in red blood cells; increased numbers are seen after splenectomy. Basophilic stippling appears as small, dark blue bodies in a red blood cell and represents residual DNA. It may indicate lead poisoning when seen in dogs

The surgeon clamps off a bleeding blood vessel. It could be said that the surgeon achieved which of the following? A) Hemolysis B) Fibrinolysis C) Hemostasis D) Coagulation

C) Hemostasis Explanation - Hemostasis is to stop bleeding or hemorrhage. Coagulation is the complex process by which blood forms clots which has not yet necessarily occurred in this example. Fibrinolysis is the process of breaking down of clot. Hemolysis is the rupturing of erythrocytes

A tiny Chihuahua puppy presents for seizures, and the veterinarian suspects the puppy has "water on the brain" due to his enlarged head size. What is the term for this? A) Brachycephalous B) Craniostosis C) Hydrocephalus D) Paracervical edema

C) Hydrocephalus Explanation - Hydrocephalus is a build-up of cerebrospinal fluid inside the skull, leading to brain swelling. Hydrocephalus means "water on the brain"

What causes Metabolic Bone Disease (MBD) in tortoises? A) A diet mostly of fresh greens B) UVB light C) Improper calcium/phosphorus ratio D) Sun exposure E) Warm temperature

C) Improper calcium/phosphorus ratio Explanation - If the dietary calcium is too low, it will cause malformation and softening of bones with deformed shells (called pyramiding). Typical tortoise diets are low in calcium and high in phosphorus, hence the need for most tortoises to receive a calcium supplement. Too much calcium or over-supplementation may cause zinc, copper, and iodine deficiency, and could lead to bladder stones. If the temperature is too low, it will hinder the ability to digest food properly. UVB light or natural sunlight is needed to absorb and metabolize calcium. There are other factors that contribute to MBD, but improper care and diet are the main factors

Where is the thymus located? A) In the brain B) Ventral neck region C) In the cranial mediastinum in the chest D) In the abdomen just above the kidneys

C) In the cranial mediastinum in the chest Explanation - The thymus is an immune organ located in the mediastinum. It extends from the thoracic inlet to the pericardium upon which it is molded. The thyroid gland is located in the ventral neck region. The adrenal glands are located just above the kidneys. The hypothalamus is located in the brain.


Set pelajaran terkait

Nursing 203 Myasthenia Gravis and Guillain Barre syndrome Exam 4

View Set

Fundamentals of Management (MGT 3120) - CH. 12

View Set

Barefoot in the Park Act Two Scene Two

View Set

FL: Post-Licensing: Real Estate Finance Practice Exam

View Set

Adult Health Prep U chapter 10, 26, 27, 46

View Set